You are on page 1of 206

CHAPTER 1

Partnership Formation and Operations

EXERCISES

Exercise 1 –1

1.a Campos, Capital 14,000


Allowance for Uncollectible Accounts 14,000

Goodwill 30,000
Campos, Capital 30,000

Accumulated Depreciation 6,000


Campos, Capital 6,500
Furniture and Fixtures 12,500

Campos, Capital 40,000


Cash 40,000

1.b Cash (P83,500 x 1/2) 41,750


Tomas, Capital 41,750
To record contributions of Tomas

Req. 2.
Campos and Tomas Partnership
Statement of financial Position
July 1, 2008

Assets Liabilities & Owners’ Equity


Cash P41,750 Accounts Payable P90,000
Accounts Rec’l P60,000 Campos, Capital 83,500
Less Allowance for Tomas, Capital 41,750
Uncol Accts. 24,000 36,000
Inventory 100,000
Furniture 7,500
Goodwill 30,000 ________
TOTAL LIABILITIES &
TOTAL ASSETS P215.250 OWNERS’ EQUITY P215,250

Exercise 1-2

1. Cash 90,000
Accounts Receivable 36,000
Merchandise Inventory 54,000
Equipment 25,000
Allowance for Uncollectible Accounts 2,000
Accounts Payable 21,000
Notes Payable 18,000
Bernal, Capital 164,000
AA 1 - Chapter 1 (2008 edition)
page 2

2. Cash 100,000
Camino, Capital 100,000

Exercise 1 –3

1. Cash 800,000
Land 540,000
Building 900,000
Legaspi, Capital 800,000
Sabino, Capital 1,440,000

2. Cash 800,000
Land 540,000
Building 900,000
Legaspi, Capital 1,120,000
Sabino, Capital 1,120,000

Exercise 1 - 4

1. Income Summary 238,000


Santos, Capital (P238,000 x 260/425) 145,600
Abad, Capital (P238,000 x 165/425) 92,400

2. Income Summary 238,000


Santos, Capital (P238,000 x 3,125/5,000) 148,750
Abad, Capital (P238,000 x 1,875/5,000) 89,250

Santos:
Jan. 1 – Mar. 31 P260,000 x 3 P780,000
Apr. 1 – Apr. 30 290,000 x 1 290,000
May 1 – July 31 360,000 x 3 1,080,000
Aug. 1 – Dec. 31 320,000 x 5 1,600,000
P3,750,000/12 P312,500

Abad:
Jan. 1 – May 31 P165,000 x 5 P825,000
June 1 – Aug. 31 215,000 x 3 645,000
Sept.1 – Dec. 31 195,000 x 4 780,000
P2,250,000/12 P187,500

3. Income Summary 238,000


Santos, Capital 147,750
Abad, Capital 90,250
Santos Abad Total
Interest on ave. capital P 18,750 P 11,250 P 30,000
Salaries to partners 150,000 100,000 250,000
Balance - equally ( 21,000) ( 21,000) (42,000)
Net Profit P 147,750 P 90,250 P238 000
AA 1 - Chapter 1 (2008 edition)
page 3

4. Income Summary 238,000


Santos, Capital 164,840
Abad, Capital 73,160
Santos Abad Total
Bonus to Santos
25% (P238,000 - B) P 47,600 P 47,600
Interest of 6% on excess
average investment
6% (P312,500 - P187,500) 7,500 7,500
Balance - 3:2 109,740 73,160 182,900
Net Profit P 164,840 P73,160 P238,000

5. Income Summary 238,000


Santos, Capital (P238,000 x 15/25) 142,800
Abad, Capital (P238,000 x 10/25) 95,200

Exercise 1 – 5
Sanchez and Gomez
Schedule of Distribution of Net Profit
December 31, 2008
Sanchez Gomez Total
6% interest on average capital P 6,246 P 14.440 P 20,686
10% bonus on net profit after interest 8,331 8,331
Salaries 20,000 30,000 50,000
Balance – 70%, 30% 17,488 7,495 24,983
Net Profit P52,065 P51,935 P104,000

Computation of average capital:


Sanchez, Capital Gomez, Capital
Jan. 1 P81,600 x 3 P 244,800 Jan. 1 P224,000 x 7 P1,568,000
Apr. 1 P111,600 x 9 1,004,400 Aug. 1 P264,000 x 5 1,320,000
P1,249,200 P2,888,000

Ave. capital (P1,249,200/12) P104,100 Ave. capital (P2,888,000/12) P240,667

Computation of bonus: P160,000 x 65%= P104,000– P20,686 x 10% = P8,331

2.
Sanchez and Gomez
Statement of Partners’ Capital
For the Year Ended December 31, 2008

Sanchez Gomez Total


Capital, January 1 P 81,600 P224,000 P305,600
Additional investment 30,000 40,000 70,000
Net profit 52,065 51,935 104,000
Drawings ( 41,600) ( 41,600) ( 83,200)
Capital, December 31 P122,065 P274,335 P396,400
AA 1 - Chapter 1 (2008 edition)
page 4

3. Sanchez Gomez Total


6% interest on average capital P 6,246 P14.440 P 20,686
10% bonus on net profit after interest 8,331 8,331
Balance – Salary ratio 34,083 40,000 74,983
Net Profit P48,660 P 55,340 P104,000

Exercise 1-6

1. Mercado Puzon Total


8% interest on beg capital P 48,000 P 54.000 P102,000
Salaries 225,000 112,500 337,500
Balance 3:2 ( 38,700) ( 25,800) ( 64,500)
Net Profit P234,300 P140,700 P375,000

2. Mercado Puzon Total


8% interest on beg capital P 48,000 P 54.000 P102,000
Balance – Salary ratio 182,000 91,000 273,000
Net Profit P230,000 P145,000 P375,000

3. Puzon P375,000 x 2/3 = P150,000; however, minimum guaranteed amount is P160,000


Mercado P375,000 – P160,000 = P215,000

Exercise 1 – 7

Net profit after salary, interest and bonus P374,000


Interest P200,000 x 10% P20,000
Salary P8,000 x 12 96,000 116,000
Net profit before interest and salary P490,000
Bonus rate x 25%
Amount of bonus to Lirio P122,500

Exercise 1 – 8

1. B = .25 x P500,000 = P125,000

2. B = .25 x P500,000 = P100,000


1.25

3. B = .25 (P500,000 - Tax)


T = .35 x P500,000 = P175,000
B = .25 (P500,000 – P175,000)
B = P 81,250

4. B = .25 (P500,000 - B - Tax)


B = .25 (P50,0000 - B - P175,000)
B = P81,250 - .25B
B = P81,250/1.25
B = P65,000
AA 1 - Chapter 1 (2008 edition)
page 5

Exercise 1 - 9

1.
Estrella Felipe Garcia Jimenez Total
Salary P40,000 P20,000 P 60,000
Bonus 6,000 4,000 10,000*
Interest 10,000 9,000 P 4,000 P 9,400 32,400
Balance 26,900 26,900 26,900 26,900 107,600
Total P82,900 P59,900 P30,900 P36,300 P210,000
*B = 5% (P210,000 – B) = P10,000
2.
Estrella Felipe Garcia Jimenez Total
Salary P40,000 P20,000 P 60,000
Interest 10,000 9,000 P 4,000 P 9,400 32,400
Balance ( 43,100) ( 43,100) ( 43,100) ( 43,100) (172,400)
Total P 6,900 (P 14,100) (P39,100) (P33,700) (P 80,000)

3.
Estrella Felipe Garcia Jimenez Total
Interest P10,000 P 9,000 P 4,000 P 9,400 P 32,400
Bonus 6,000 4,000 10,000
Salary 25,067 12,533 _______ ________ 37,600*
Total P41,067 P25,533 P 4,000 P 9,400 P 80,000
*P37,600 x 4/ 6 = P25,067; P37,600 x 2/ 6 = P12,533

Exercise 1-10

1. Fees Earned 750,000


Joseph, Capital 50,000
Luis, Capital 200,000
Operating Expenses 100,000
Income Summary 500,000

2. Income Summary 500,000


Joseph Capital 150,000
Luis, Capital 250,000
Nicolas, Capital 100,000

Exercise 1 – 11

1.
Benito Cabral Duenas Total
Capital balances before payment
of cash P120,000 P100,000 P100,000 P320,000
Required capital balances based on
on profit and loss ratio 128,000 112,000 80,000 320,000
Cash received (paid) (P 8,000) (P 12,000) P 20,000 -
AA 1 - Chapter 1 (2008 edition)
page 6

Journal entry on the partnership books

Duenas, Capital 20,000


Benito, Capital 8,000
Cabral, Capital 12,000

2.
Benito Cabral Duenas Total
Capital balances before additional
cash investment P120,000 P100,000 P100,000 P320,000
Required capital balances based on
lowest possible cash investment* 160,000 140,000 100,000 400,000
Required additional cash investment P 40,000 P 40,000 - P 80,000
* P120,000/40% = P300,000; P100,000/35% = P285,174; P100,000/25% = P400,000

Journal entry on the partnership books

Cash 80,000
Benito, Capital 40,000
Cabral, Capital 40,000

3.
Benito Cabral Duenas Total
Capital balances P120,000 P100,000 P100,000 P320,000
Required capital 120,000 105,000 75,000 300,000
Additional investment(withdrawals) -------- 5,000 (P 25,000) P 20,000

Duenas, Capital 25,000


Cash 20,000
Cabral, Capital 5,000

Exercise 1 – 12
Enriquez and Flores
Schedule Showing Adjustments in Capital
For the Year Ended December 31, 2008

Reported net profit P400,000


Adjustments:
Equipment purchased charged to expense P200,000
Depreciation on equipment ( 20,000)
Overstatement of 2008 ending inventory ( 24,000)
P156,000
x 65% 101,400
Corrected net profit P501,400
AA 1 - Chapter 1 (2008 edition)
page 7

Distribution of 2008 net profit


Enriquez Flores Total
Salaries P120,000 P120,000 P240,000
Interest 30,000 45,000 75,000
Balance 51,000 34,000 85,000
P201,000 P199,000 P400,000

Distribution of 2008 corrected net profit


Salaries P120,000 P120,000 P240,000
Interest 30,000 45,000 75,000
Balance 111,840 74,560 186,400
P261,840 P239,560 P501,400
Adjustments P 60,840 P 40,560 P101,400

2. Equipment 200,000
Enriquez, Capital 60,840
Flores, Capital 40,560
Accumulated Depreciation 20,000
Inventory 24,000
Income Tax Payable 54,600

PROBLEMS
Problem 1 – 1

1. a. Merchandise, Inventory 60,000


Ruiz, Capital 60,000

b. Ruiz, Capital 30,000


Allowance for Uncollectible Accounts 30,000

c. Interest Receivable 1,500


Ruiz, Capital 1,500
P150,000 x 6% x 2/12 = P1,500

d. Ruiz, Capital 7,500


Interest Payable 7,500
P300,000 x 10% x 3/12 = P7,500

e. Accumulated Depreciation 180,000


Ruiz, Capital 60,000
Furniture and Fixtures 240,000

f. Office Supplies 5,000


Ruiz, Capital 5,000

g. Cash 524,500
Santos, Capital 524,500
AA 1 - Chapter 1 (2008 edition)
page 8

Ruiz and Santos


Statement of Financial Position
December 1, 2008

Assets
Cash P 764,500
Notes Receivable 150,000
Accounts Receivable P900,000
Less Allowance for Uncollectible Accounts 90,000 810,000
Interest Receivable 1,500
Merchandise Inventory 300,000
Office Supplies 5,000
Furniture and Fixtures 480,000
Total Assets P2,511,000
Liabilities and Capital
Notes Payable P300,000
Accounts Payable 630,000
Interest Payable 7,500
Total Liabilities P 937,500
Ruiz, Capital P1,049,000
Santos, Capital 524,500
Total Capital 1,573,500
Total Liabilities and Capital P2,511,000

Problem 1-2

1. Cash 518,000
Merchandise Inventory 1,152,000
Tomas, Capital 1,670,000

2. Accounts Receivable 1,792,000


Merchandise Inventory 256,000
Office Equipment 160,000
Goodwill 198,000
Allowance for Uncollectible Accounts 160,000
Accounts Payable 576,000
Vicente, Capital 1,670,000

Tomas and Vicente


Statement of Financial Position
June 1, 2008

Assets
Cash P 518,000
Accounts Receivable P1,792,000
Less Allowance for Uncollectible Accounts 160,000 1,632,000
Inventories 1,408,000
Office Equipment 160,000
Goodwill 198,000
Total Assets P3,916,000
AA 1 - Chapter 1 (2008 edition)
page 9

Liabilities and Capital


Accounts Payable P 576,000
Tomas, Capital P1,670,000
Vicente, Capital 1,670,000 3,340,000
Total Liabilities and Capital P3,916,000

Problem 1 – 3

1. Merchandise Inventory 3,000


Goodwill 3,000
Accumulated Depreciation 900
Allowance for Uncollectible Accounts 1,000
Equipment 2,000
Rosas, Capital 3,900

Cash 5,000
Accounts Receivable 46,000
Merchandise Inventory 108,000
Equipment 12,000
Furniture and Fixtures 9,000
Goodwill 3,000
Allowance for Uncollectible Accounts 4,000
Accounts Payable 54,000
Perlas, Capital 125,000

2. Cash 5,000
Accounts Receivable 46,000
Merchandise Inventory 108,000
Equipment 12,000
Furniture and Fixtures 9,000
Goodwill 3,000
Allowance for Uncollectible Accounts 4,000
Accounts Payable 54,000
Perlas, Capital 125,000

Cash 7,000
Accounts Receivable 49,000
Merchandise Inventory 75,000
Equipment 7,000
Goodwill 3,000
Allowance for Uncollectible Accounts 5,000
Accounts Payable 36,000
Rosas, Capital 100,000

Problem 1 – 4
1. Cash 900,000
Inventories 1,500,000
Equipment 3,000,000
Notes Payable 1,050,000
Serrano, Capital 4,350,000
AA 1 - Chapter 1 (2008 edition)
page 10

Cash 600,000
Land 6,000,000
Mortgage Payable 1,950,000
Torres, Capital 4,650,000

Torres, Capital 150,000


Serrano, Capital 150,000

Purchases 900,000
Accounts Payable 900,000

Accounts Payable 720,000


Cash 720,000

Mortgage Payable 300,000


Interest Expense 120,000
Cash 420,000

Notes Payable 225,000


Interest Expense 75,000
Cash 300,000

Accounts Receivable 3,450,000


Sales 3,450,000

Cash 3,150,000
Accounts Receivable 3,150,000

Selling and General Expenses 870,000


Cash 630,000
Accumulated Depreciation 150,000
Accrued expenses 90,000

Serrano, Drawing 351,000


Torres, Drawing 351,000
Cash 702,000

Income Tax 204,750


Income Tax Payable 204,750

Inventories, end 600,000


Sales 3,450,000
Inventories, beginning 1,500,000
Purchases 900,000
Selling and General Expenses 870,000
Interest Expense 195,000
Income Tax 204,750
Income Summary 380,250
AA 1 - Chapter 1 (2008 edition)
page 11

Income Summary 380,250


Serrano, Capital 192,150
Torres, Capital 188,100

Serrano Torres Total


Interest on beginning capital P180,000 P180,000 P360,000
Salaries 150,000 100,000 250,000
Remainder – 60%, 40% ( 137,850) ( 91,900) ( 229,750)
Net Profit P192,150 P188,100 P380,250

Serrano, Capital 351,000


Torres, Capital 351,000
Serrano, Drawing 351,000
Torres, Drawing 351,000

Serrano and Torres Partnership


Statement of Recognized Income and Expenses
For the Year Ended December 31, 2008

Sales P3,450,000
Cost of Goods Sold:
Inventories, beginning P1,500,000
Purchases 900,000
Cost of Goods Available for Sale P2,400,000
Less Inventories, end 600,000 1,800,000
Gross Profit P1,650,000
Selling and General Expenses 870,000
Operating Income P 780,000
Interest Expense 195,000
Net Profit before Income Tax P 585,000
Income Tax 204,750
Net Profit P 380,250

Serrano and Torres Partnership


Statement of Financial Position
December 31, 2008

Assets
Current Assets:
Cash P1,878,000
Accounts Receivable (P3,450,000 – P3,150,000) 300,000
Inventories 600,000 P 2,778,000
Property, Plant and Equipment:
Land P6,000,000
Equipment P3,000,000
Less Accumulated Depreciation 150,000 2,850,000 8,850,000
Total Assets P11,628,000
AA 1 - Chapter 1 (2008 edition)
page 12

Liabilities
Current Liabilities:
Accounts Payable (P900,000 – P720,000) P180,000
Accrued Expenses 90,000
Income Tax Payable 204,750 P 474,750
Long-term Liabilities:
Notes Payable (P1,050,000 – P225,000) P 825,000
Mortgage Payable (P1,950,000 – P300,000) 1,650,000 2,475,000
Total Liabilities P 2,949,750

Capital
Serrano, Capital P4,341,150
Torres, Capital 4,337,100
Total Capital
8,678,250
Total Liabilities and Capital P11,628,000

Problem 1 - 5

1. P2,700,000 (P200,000 + P1,100,000 + P500,000 + P1,500,000 – P600,000 = P2,700,000)


2 P2,600,000. (P2,500,000 + P2,700,000) / 2 = P2,600,000

Problem 1 – 6

1. Income Summary 700,000


Bernabe, Capital 505,800
Burgos, Capital 194,200
Bernabe Burgos Total
Interest on beg. capital P 28,800 P 35,200 P 64,000
Balance – 3:1 477,000 159,000 636,000
Net Profit P 505,800 P 194,200 P700 000

2. Income Summary 700,000


Bernabe, Capital 284,000
Burgos, Capital 416,000
Bernabe Burgos Total
Salaries P 140,000 P 260,000 P400,000
Interest on end capital 48,000 60,000 P108,000
Balance – Equally 96,000 96,000 96,000
Net Profit P 284,000 P 416,000 P700 000

3. Income Summary 700,000


Bernabe, Capital 394,150
Burgos, Capital 305,850
Bernabe Burgos Total
Salaries P 90,000 P 170,000 P260,000
Interest on average. cap 49,000 50,800 99,800
Balance – 3:1 255,150 85,050 340,200
Net Profit P 394,150 P 305,850 P700 000
AA 1 - Chapter 1 (2008 edition)
page 13

Bernabe:
Jan. 1 – May 31 P360,000 x 5 P1,800,000
June 1 – Oct. 31 460,000 x 5 2,300,000
Nov, 1 – Dec. 31 400,000 x 2 800,000
P4,900,000/12 P408,333

Burgos:
Jan. 1 – June 30 P440,000 x 6 P2,640,000
July 1 – Oct. 31 360,000 x 4 1,440,000
Nov.1 – Dec. 31 500,000 x 2 1,000,000
P5,080,000/12 P423,333

4. Income Summary 700,000


Bernabe, Capital 267,567
Burgos, Capital 432,433
Bernabe Burgos Total
Salaries P 100,000 P 200,000 P300,000
Interest on average. cap 40,833 42,333 83,166
Balance – 40%, 60% 126,734 190,100 316,834
Net Profit P 267,567 P 432,433 P700 000

5. Income Summary 700,000


Bernabe, Capital 329,360
Burgos, Capital 370,640
Bernabe Burgos Total
Salaries P 100,000 P 100,000 P220,000
Interest on beg. cap 28,800 35,200 64,000
Bonus 43,600 43,600
Balance – 2:3 156,960 235,440 392,400
Net Profit P 329,360 P 370,640 P700 000
B = 10%(NI –S – I)

Problem 1 – 7
Sandy Tammy Manny Total
1. 6% interest on capital P 16,800 P 12,000 P 7,200 P 36,000
Salaries 48,000 40,000 88,000
Balance – 5:3:2 ( 74,500) ( 44,700) ( 29,800) (149,000)
Net Profit P(57,700) P 15,300 P 17,400 P(25,000)

2. 6% interest on capital P 16,800 P 12,000 P 7,200 P 36,000


Salaries 48,000 40,000 88,000
Balance – 5:3:2 ( 32,000) ( 19,200) ( 12,800) ( 64,000)
Net Profit P( 15,200) P 40,800 P 34,400 P 60,000

3. 6% interest on capital P 16,800 P 12,000 P 7,200 P 36,000


Salaries 48,000 40,000 88,000
Bonus 13,500 13,500
Balance – 5:3:2 56,250 33,750 22,500 112,500
Net Profit P 73,050 P107,250 P 69,700 P250,000
AA 1 - Chapter 1 (2008 edition)
page 14

B = (P250,000 – P36,000 – P88,000 – P72,000)25% = P13,500

Problem 1 - 8

1. Delmar Pilar Total


5% interest on capital P 2,500 P 1,500 P 4,000
Salaries 12,000 8,000 20,000
20% bonus on net profit 22,100 22,100
Balance – capital ratio 40,250 24,150 64,400
Net Profit P76,850 P33,650 P110,500

2. Sales 480,000
Cost of Goods Sold 210,000
Operating Expenses 100,000
Income Taxes 59,500
Income Summary 110,500

Income Summary 110,500


Delmar, Capital 76,850
Pilar, Capital 33,650

Delmar, Capital 6,000


Pilar, Capital 10,000
Delmar, Drawing 6,000
Pilar, Drawing 10,000

Delmar and Pilar Company


Statement of Changes in Partners’ Capital
For the Year Ended December 31, 2008

Delmar Pilar Total


Capital balances, January 1, 2008 P 50,000 P30,000 P 80,000
Add Distribution of net income for 2008:
Interests P 2,500 P 1,500 P 4,000
Salaries 12,000 8,000 20,000
Bonus 22,100 22,100
Balance - capital ratio 40,250 24,150 64,400
Total share in net profit P 76,850 P33,650 P110,500

Total P126,850 P63,650 P190,500


Less Drawings 6,000 10,000 16,000
Capital balances, December 31, 2008 P120,850 P53,650 P174,500
AA 1 - Chapter 1 (2008 edition)
page 15

Problem 1 - 9

Ramos, Gonzales and Martinez


Statement of Changes in Partners’ Capital
For Three Years Ended December 31, 2008

Ramos Gonzales Martinez Total


Capital, January 1, 2006 P 80,000 P 48,000 P 40,000 P 168,000
Distribution of net loss (Sch. 1) ( 2,000) ( 1,520) ( 2,000) ( 5,520)
Withdrawals (12,000) (14,480) (16,000) (42,480)
Capital, December 31, 2006 P 66,000 P 32,000 P 22,000 P120,000
Distribution of net profit (Sch. 2) 7,960 8,320 7,720 24,000
Withdrawals (13,960) (16,320) (17,720) (48,000)
Capital, December 31, 2007 P 60,000 P 24,000 P12,000 P 96,000
Distribution of net profit (Sch. 3) 21,840 18,840 18,120 58,800
Withdrawals (20,400) (24,000) (21,200) (65,600)
Capital, December 31, 2008 P 61,440 P 18,840 P 8,920 P 89,200

Schedule 1 - Distribution of 2006 net loss

Ramos Gonzales Martinez Total


Salaries to partners P 9,600 P 12,000 P 12,000 P 33,600
Interest of 6% on beg. Capital 4,800 2,880 2,400 10,080
Balance – equally (16,400) (16,400) (16,400) P 49,200
Net income P( 2,000) P( 1,520) P( 2,000) P( 5,520)

Schedule 2 - Distribution of 2007 net profit

Ramos Gonzales Martinez Total


Salaries to partners P 9,600 P 12,000 P 12,000 P 33,600
Interest of 6% on beg. Capital 3,960 1,920 1,320 7,200
Balance – equally ( 5,600) ( 5,600) ( 5,600) (16,800)
Net income P 7,960 P 8,320 P 7,720 P 24,000

Schedule 3 - Distribution of 2008 net profit

Ramos Gonzales Martinez Total


Salaries to partners P 9,600 P 12,000 P 12,000 P 33,600
Interest of 6% on beg. Capital 3,600 1,440 720 5,760
Bonus - 20% (P58,800 - P39,360 - B) 3,240 3,240
Balance – equally 5,400 5,400 5,400 16,200
Net income 21,840 18,840 18,120 58,800
AA 1 - Chapter 1 (2008 edition)
page 16

Problem 1 -10
Robles, Bernal and Reyes
Statement of Partners’ Capital
For the Year Ended December 31, 2008

Robles Bernal Reyes Total


Capital balances before closing
the nominal accounts P120,000 P ( 2,000) P20,000 P138,000
Add Distribution of net profit:
Drawing allowance 20,000 14,000 10,000 44,000
Interest on average capital 7,200 240 560 8,000
Balance - 60%, 30%, 10% 58,800 29,400 9,800 98,000
Total P206,000 P41,640 P40,360 P288,000
Deduct Cash distribution 122,720 26,480 149,200
Capital, December 31, 2008 P 83,280 P41,640 P13,880 P138,800

P206,000 / 60% = P343,333; P40,360 / 10% = P403,600


P41,640 / 30% = P138,800 (required total capital)

Problem 1 - 11
Chavez, Roman, and Valdez
Statement of Changes in Partners’ Capital
January 1 to November 1, 2008

Chavez Chavez Roman Valdez


Loan Capital Capital Capital Total
Beginning balances P 80,000 P 80,000 P 80,000 P240,000
Loan from Chavez P 60,000 60,000
Transfer of equipment to Valdez ( 16,000) ( 16,000)
Balances P 60,000 P 80,000 P 80,000 P 64,000 P284,000
Distribution of loss on realization*
Salary to Valdez 24,000 24,000
Int. to Chavez for 7 months 2,100 2,100
Balance divided equally* ( 76,700) ( 76,700) ( 76,700) ( 230,100)
Balances P 60,000 P 5,400 P 3,300 P 11,300 P 80,000
Dist. of cash in final settlement 60,000 5,400 3,300 11,300 80,000

*Total partners’ equity as shown above P284,000


Less Cash available for distribution 80,000
Loss on realization P204,000
Less Salary and interest 26,100
Total loss to be divided equally P230,100
AA 1 - Chapter 1 (2008 edition)
page 17

Problem 1 - 12
Canlas, David, Estrella and Fajardo
Statement of Changes in Partners’ Capital Accounts
For the Year Ended December 31, 2008

Canlas David Estrella Fajardo Total


Investment P309,000 P159,000 P327,000 ------- P 795,000
Net profit 237,700 186,230 140,310 P 24,010 588,250
Total P546,700 P345,230 P467,310 P 24,010 P1,383,250
Less: Excess rent (P225 x 6) P 13,500 P 13,500
Withdrawals P 78,000 P 66,000 87,000 P 37,500 268,500
Uncollectible accounts 18,000 6,750 24,750
P 96,000 P 72,750 P100,500 P 37,500 P 306,750
Capital, December 31 P450,700 P272,480 P366,810 P(13,490) P1,076,500

Supporting computations:

Revenue from fees P 900,000


Expenses:
Total expenses, excluding depreciation and uncollectible
accounts (P290,000 - P13,500) P 276,500
Depreciation [(P195,000 x 10%) + (P75,000 x 5%) 23,250
Doubtful accounts (P24,000 x 50%) 12,000 311,750
Net profit P588,250

Distribution of net income

Canlas David Estrella Fajardo Total


20% of gross fees from respective
clients P 66,000 P 36,000 P 33,000 P135,000
20% of fees after April 1 after
expenses but before bad debts P24,010* 24,010
Balance -Canlas-40%, David-35%,
Estrella-25% 171,700 150,230 107,310 429,240
Total P237,700 P186,230 P140,310 P24,010 P588,250

After April 1
Revenues P 180,000
Expenses before uncollectible accounts (P276,500 + P23,250) x 180 / 900 59,950
P120,050
20%
Share of Fajardo P 24,010

Problem 1-13

1. Equipment 13,500
Accumulated Depreciation 1,350
Profit and Loss 12,150
AA 1 - Chapter 1 (2008 edition)
page 18

2. Profit and Loss 4,375


Interest Payable 4,375
P87,500 x 6% x 10/12 = P4,375

3. Profit and Loss 159,025


Abaya, Capital 63,700
Reyes, Capital 95,505
Abaya Reyes Total
Salaries P 39,000 P 58,500 P 97,500
Bonus [25% x (NI – B – S) ] 12,305 12,305
Balance – equally 24,610 24,610 49,220
Total P 63,610 P 95,415 P159,025

4. Abaya, Capital 36,000


Reyes, Capital 62,500
Abaya, Drawing 36,000
Reyes, Drawing 62,500

Problem 1-14

1. Jaime = 5/10 x 80% = 40% Soriano = 2/10 x 80% = 16%


Madrid = 3/10 x 80% = 24% Matias = 20%

2. Corrected net income = P250,000 – (P12,000 – P31,000 – P20,000 + P15,000 + P9,000 x


65%) = P240,250

Jaime = P240,250 x 40% = P96,100 Soriano = P240,250 x 16% = P38,440


Madrid = P240,250 x 24% = P57,660 Matias = P240,250 x 20% = P48,050

MULTIPLE CHOICE

1. D
2. D
3. A
4. C Abena Buendia
Total (60%) (40%)
Abena – MV – Cost (P90,000 – P60,000) P30,000 P18,000 P12,000
Buendia – MV – Cost (P60,000 – P70,000) ( 10,000) ( 6,000) ( 4,000)
Actual P20,000 P12,000 P 8,000
Inequity ( 20,000) ( 30,000) 10,000
P 0 (P18,000) P18,000

5. A
6. C
7. B
8. B Molina’s contribution (P190,000 – P60,000) P130,000
Nuevo’s tangible contribution 100,000
Total capital contributions P230,000
AA 1 - Chapter 1 (2008 edition)
page 19

x 60%
Capital credit of Molina P 138,000
Contribution of Molina 130,000
Bonus to Molina P 8,000

9. Roxas = P596,250 - P5,550 = P590,700


Bernardo = P335,000 - P4,050 - P9,000 = P321,950

10. Roxas = (P590,700 + P321,950) x 60% = P547,590


Bernardo = (P590,700 + P321,950) x 40% = P365,060

11. Roxas = P650,000 – P590,700 = P59,300


Bernardo = P400,000 – P321,950 = P78,050

12. Bruno = P150,000 - P90,000 = P60,000

13. Total assets = Total liab. + Total capital


= P25,000 + P300,000 = P325,000

14. Cash contribution = (P248,850 x 1/3) – P50,000 = P32,950

15 Total capital = (P158,400 + P17,500 – P5,000 – P5,000) ÷ 2/3 = P248,850

16. Required capital of Esteban (P287,500 x 60%) P172,500


Non-cash contributions of Esteban (P125,000 – P30,000) 95,000
Cash contribution P 77,500

17. Contribution of Diaz P115,000


Contribution of Esteban (P125,000 – P30,000 + P50,000) 145,000
Total partnership capital P260,000

18. C P115,000 + P95,000 = P210,000/2 P105,000

19. A Net increase (decrease) in capital (P120,000)


Add Withdrawals 260,000
Total (P140,000)
Less Additional investments 50,000
Profit share P 90,000
Profit share percentage ÷ 30%
Total partnership net profit P300,000

20. C
21. B
22. C Net profit (exclusive of salary, interest and bonus) P 93,500
Salary (P2,000 x 12) 24,000
Interest (P50,000 x 5%) 2,500
Net profit after deduction of bonus P120,000
Bonus = .20 (P120,000 + Bonus) = P24,000 + .20 Bonus
= P24,000/.80 = P30,000
AA 1 - Chapter 1 (2008 edition)
page 20

23. D
24. C Alberto Bustos Cancio Total
10% x P1,000,000 P 100,000 P 100,000
20% x P1,500,000 300,000 300,000
5% (P1M – P400,000) P30,000 P30,000 60,000
Balance – equally 680,000 680,000 680,000 2,040,000
Net income P1,080,000

25. A Ramos Campos Ocampo Total


Interest P24,000 P12,000 P 8,000 P 44,000
Salaries 60,000 40,000 100,000
Balance – equally ( 70,000) (70,000) ( 70,000) ( 210,000)
P14,000

26. C Sison Torres Velasco Total


Bonus - 10%(P44,000 - B) P 4,000 P 4,000
Interest on capital
in excess of P100,000 P 1,000 1,000
Salaries to partners P10,000 12,000 22,000
Balance - 4:4:2 6,800 6,800 3,400 17,000
P19,400 P44,000

27. B Sison Torres Velasco Total


Bonus - 10%(P22,000 - B) P 2,000 P 2,000
Interest on capital
in excess of P100,000 P 1,000 1,000
Salaries to partners P10,000 12,000 22,000
Balance - 4:4:2 (1,200) (1,200) (600) (3,000)
P13,400 P22,000

28. D Sison Torres Velasco Total


Bonus - 10%(P22,000 - B) P 2,000 P 2,000
Interest on capital
in excess of P100,000 P 1,000 1,000
Balance – Salary ratio P8,636 10,364 19,000
P12,364 P22,000

29. A Average capital of Tamayo Average capital of Vidal


P100,000 x 6 = P 600,000 P225,000 x 9 = P2,025,000
160,000 x 6 = 960,000 155,000 x 3 = 465,000
P1,560,000/12 P2,490,000/12
P130,000 P207,500

Average capital of Banson - P150,000

Total int. on ave. capital= (P130,000 + P207,500 + P150,000) 10%


= P48,750
AA 1 - Chapter 1 (2008 edition)
page 21

30. D Interest on ave. capital P 48,750


Salaries to partners 144,000
Balance - divided equally 9,000
P 201,750

31. B Total capital before net income


(P475,000 + P60,000 – P70,000) P465,000
Add Net profit 201,750
Total capital, Dec. 31, 2008 P666,750

32. D Andal Briones Camba Total


Int. on average capital P 47,250 P 23,865 P 16,235 P 87,350
Salaries to partners 122,325 82,625 204,950
Balance - equally (139,308) (139,308) (139,308) (417,924)
Net increase (decrease) P 30,267 P(115,443) P( 40,448) P(125,624)

33. C Net income = Net sales - CGS - Depr. - Oper. exp. Others)
= P228,000 - P123,000 - P7,500 - P58,100 x 65% P25,610

Mariano Lucas total


Salary to partner for 10 mos. P10,000 P 10,000
Bonus to managing partner 1,561 1,561
Balance – based on orig. cap. 8,781 P 5,268 14,049
TOTAL share in profit P 20,342 P 5,268 P 25,610

34. A TOTAL share in profit P 20,342 P 5,268 P 25,610


Add Capital, beginning 125,000 75,000 200,000
TOTAL P145,342 P 80,268 P 225,610
Less Withdrawals 20,000 30,000 50,000
Capital, end P125,342 P 50,268 P 175,610

35. D Belen Lorna Ursula Edna Total


36. A Interest P 5,000.00 P 2,500.00 P 2,500.00 P2,000.00 P12,000.00
37. A Salaries 10,000.00 6,000.00 16,000.00
Balance 10,000.00 10,000.00 6,666.67 6,666.67 33,333.34
Add’l profit
for Edna ________ ________ _________ 3,333.33 3,333.33
Net profit P25,000.00 P18,500.00 P9,166.67 P12,000.00 P64,666.67

38. B Puno Quirino Romero Total


Salaries P40,000 P36,000 P13,650 P 89,650
Bonus 13,000 13,000
Interest 1,000 750 4,600 6,350
Balance 7,000 7,000 7,000 21,000
Total P61,000 P43,750 P25,200 P130,000
AA 1 - Chapter 1 (2008 edition)
page 22

Computation of average capital:


Puno, capital
Jan. 1 – P10,000 x 3 P 30,000
Apr 1 - 9,000 x 3 27,000
July 1 - 11,000 x 3 33,000
Oct. 1 - 10,000 x 3 30,000
P120,000 / 12 P10,000
Quirino, capital
Jan. 1 – P 6,000 x 6 P 36,000
July 1 - 10,000 x 3 30,000
Oct. 1 - 8,000 x 3 24,000
P 90,000 / 12 P 7,500

Romero, capital
Jan. 1 – P40,000 x 3 P120,000
Apr. 1 - 38,000 x 3 114,000
July 1 - 53,000 x 6 318,000
P552,00 / 12 P46,000

Let X = Net Income


P40,000 + 10% X + P1,000 + 1/3 (X – P89,650 – 10% X – P6,350 = P61,000
P40,000 + 10% X + P1,000 + 1/3 (90% X – P96,000) = P61,000
P40,000 + 10% X + P1,000 + 30% X – P32,000 = P61,000
10% X + 30% X = P61,000 – P40,000 – P1,000 + P32,000
40% X = P52,000
X = P130,000

39. D Legarda- 5/10 x 80% = 40% Sotto - 2/10 x 80% = 16%


Madrigal-3/10 x 80% = 24% Pimentel - 20%

40. C Share of Legarda = P25,000 – ( P1,200 - P3,100 - P2,000 + P1,500 + P


900 x 65%) = P24,025 x 40% = P9,610

41. C Serrano Toledo


2008 Net income
(P50,000 – P8,000) P42,000
Salary to Serrano ( 36,000) P36,000
Remainder P 6,000
Divided equally ( 6,000) 3,000 P 3,000

Understatement in 2007 NI P8,000


Divided 60:40 ( 8,000) 4,800 3,200

Income allocation P43,800 P 6,200


1

CHAPTER 2
Partnership Dissolution

EXERCISES

Exercise 2 – 1

1. Sales, Capital 140,000


Rosales, Capital 140,000

2. P280,000 + P320,000 + P200,000 = P800,000

Exercise 2 –2

1. Total capital (P3,000,000 / 80%) P3,750,000


Capital interest of Fidel x 20%
Cash to be contributed by Fidel P 750,000

2. Cash 750,000
Fidel, Capital 750,000

Exercise 2 – 3
1.
Centeno, Capital 40,000
Corales, Capital 40,000
2.
Other Assets 80,000
Cortes, Capital 50,000
Centeno, Capital 20,000
Claudio, Capital 10,000
P140,000/ ¼ = P560,000 – (P200,000 + P 160,000 + P120,000)

Cortes, Capital P200,000 + P50,000 x 1/4 62,500


Centeno, Capital P160,000 + P20,000 x 1/4 45,000
Claudio, Capital P120,000 + P10,000 x 1/4 32,500
Corales, Capital 140,000
3.
Cash 230,000
Cortez, Capital 32,812
Centeno, Capital 13,125
Claudio, Capital 6,563
Corales, Capital 177,500
AC CC Bonus_
old (3/4) P532,500 P480,000 P52,500
new (1/4) 177500 230,000 (52,500)
P710,000 P710,000 P---0---
AA 1 - Chapter 2 (2008 edition)
page 2

Exercise 2 – 4
1.
Conde, Capital 90,000
Cuenco, Capital 60,000
Catral, Capital 150,000
2.
Other Assets 360,000
Conde, Capital 270,000
Cuenco, Capital 90,000

Conde, Capital P270,000 + 270,000 x 1/3 180,000


Cuenco, Capital P180,000 + P90,000 x 1/3 90,000
Catral, Capital 270,000
3.
Cash 270,000
Conde, Capital 67,500
Cuenco, Capital 22,500
Catral, Capital 180,000
AC CC Bonus_
old (3/4) P540,000 P450,000 P90,000
new (1/4) 180,000 270,000 (90,000)
P720,000 P720,000 P---0---
4.
Cash 270,000
Other Assets 360,000
Conde, Capital 270,000
Cuenco, Capital 90,000
Catral, Capital 270,000
AC CC Asset Re
old (3/4) P 810,000 P450,000 P360,000
new (1/4) 270,000 270,000
P1,080,000 P720,000 P360,000
5.
Cash 270,000
Conde, Capital 67,500
Cuenco, Capital 22,500
Catral, Capital 360,000

Exercise 2-5
1a. Bonus Method

Cash 180,000
Alba, Capital 6,000
Medel, Capital 9,000
Almeda, Capital 195,000
AC CC Bonus_
old (3/4) P585,000 P600,000 P(15,000)
new (1/4) 195,000 180,000 15,000
P780,000 P780,000 P---0---
AA 1 - Chapter 2 (2008 edition)
page 3

1b. Revaluation of Assets Method (AC = P180,000 ÷ 1/4 = P720,000)

Alba, Capital (P60,000 x 40%) 24,000


Medel, Capital (P60,000 x 60%) 36,000
Other Assets (P780,000 - P720,000) 60,000
To record revaluation of assets

Cash 180,000
Almeda, Capital 180,000

2. Alba Medel Almeda


Balances under the bonus method P194,000 P391,000 P195,000

Balances under the asset rev. method P200,000 P400,000 P200,000


Additional depreciation ( 6,666) ( 6,667) ( 6,667)
Balances after depreciation P193,334 P393,333 P193,333

Net advantage to Medel using the asset revaluation method P 2,333

Exercise 2 - 6

1. Garces, Capital 60,000


Kalaw, Capital 60,000
P120,000 x 1/2 = P60,000

2. Cash 60,000
Other Assets (P400,000 – P320,000) 80,000
Kalaw, Capital 40,000
Garces, Capital (P100,000 x 3/8) 37,500
Hilario, Capital (P100,000 x 3/8) 37,500
Juan, Capital (P100,000 x 2/8) 25,000
Total agreed capital P400,000
Total capital contribution 320,000
Asset revaluation P 80,000

Interest acquired from Garces P 60,000


Cash invested in the partnership 60,000
Total P 120,000
Capital credit of Kalaw 100,000
Bonus to old partners P 20,000

Exercise 2 – 7
Bonus method
Sabado Galman Estacio Total
Capital before admission of Estacio P1,000,000 P800,000 P1,800,000
Contribution of Estacio P500,000 500,000
Bonus to old partners 24,000 16,000 ( 40,000)
Capital after admission of Estacio P1,024,000 P816,000 P460,000 P2,300,000
AA 1 - Chapter 2 (2008 edition)
page 4

Asset Revaluation method


Sabado Galman Estacio Total
Capital before admission of Estacio P1,000,000 P800,000 P1,800,000
Contribution of Estacio P500,000 500,000
Adjustment of fixed assets to fair value 120,000 80,000 200,000
Capital after admission of Estacio P1,120,000 P880,000 P500,000 P2,500,000

Exercise 2 – 8
1. Bonus method
Noble Calma Reyes Naval Total
Capital balances before admission
of new partners P64,000 P136,000 P200,000
Contributions of new partners P110,000 P120,000 230,000
Bonus to old partners 10,950 25,550 ( 24,000) ( 12,500)
Capital balances after admission
of new partners P74,950 P161,550 P86,000 P107,500 P430,000

Cash 130,000
Equipment 100,000
Noble, Capital 10,950
Calma, Capital 25,550
Reyes, Capital 86,000
Naval, Capital 107,500

2. Asset Revaluation method


Noble Calma Reyes Naval Total
Capital balances before admission
of new partners P64,000 P136,000 P200,000
Adjustment of assets to FV 9,000 21,000 30,000
Contributions of new partners P110,000 P120,000 230,000
Capital balances after admission
of new partners P73,000 P157,000 P110,000 P120,000 P460,000

Cash 130,000
Equipment 84,000
Inventory 14,000
Land 80,000
Building 48,000
Noble, Capital 9,000
Calma, Capital 21,000
Reyes, Capital 110,000
Naval, Capital 120,000
Exercise 2 - 9
1a. Bonus Method
Songco, Capital 200,000
Bueno, Capital 60,000
Manzano, Capital 40,000
Cash/Payable to Songco 300,000
AA 1 - Chapter 2 (2008 edition)
page 5

1b. Asset Revaluation Method

Songco, Capital 200,000


Other Assets (P10,000 ÷ 1/6) 600,000
Bueno, Capital (P50,000 x 3/5) 300,000
Manzano, Capital (P50,000 x 2/5) 200,000
Cash/ Payable to Songco 300,000

2. The bonus method will be preferred by Manzano


Bonus Method Asset Rev
Capital of Manzano after retirement P260,000 P500,000
Additional depreciation 300,000
Capital of Manzano after additional depreciation P260,000 P200,000

Net advantage to Manzano with the use of the bonus P60,000


method

Exercise 2 – 10
1.
Delfin, Capital 400,000
Damian, Capital 200,000
Dencio, Capital 200,000
2.
Delfin, Capital 400,000
Cash 320,000
Damian, Capital 40,000
Dencio, Capital 40,000

3.
Other Assets 180,000
Delfin, Capital 400,000
Cash 460,000
Damian, Capital 60,000
Dencio, Capital 60,000
P460,000 – P400,000 = P60,000/ 1/3 = P180,000

Exercise 2 – 11
1. Guzman, Capital January 1 P100,000
Drawing (16,000)
Share in net profit 24,000
Interest of Guzman upon retirement P108,000

Other Assets 40,000


Guzman, Capital 108,000
Cash 120,000
Jorge, Capital 12,000
Lopez, Capital 16,000
P120,000 – P108,000 = P12,000/ 30% = P40,000
AA 1 - Chapter 2 (2008 edition)
page 6

2.
Guzman, Capital 108,000
Jorge, Capital 5,143
Lopez, Capital 6,857
Cash 120,000

Exercise 2 – 12

1. Building 200,000
Villa, Capital 60,000
Belen, Capital 40,000
Marcos, Capital 80,000
Cordero, Capital 20,000

Belen, Capital 140,000


Cash 140,000

2. Villa, Capital 15,000


Belen, Capital 100,000
Marcos, Capital 20,000
Cordero, Capital 5,000
Cash 140,000

Exercise 2 - 13

1. Galang, Capital 12,000


Henio, Capital 8,000
Israel, Capital 140,000
Cash 160,000

2. Israel, Capital 140,000


Galang, Capital 140,000

3. Israel, Capital 140,000


Cash 130,000
Galang, Capital 6,000
Henio, Capital 4,000

4. Other Assets 48,000


Israel, Capital 140,000
Cash 148,000
Galang, Capital 24,000
Henio, Capital 16,000

5. Israel, Capital 140 000


Galang, Capital 60,000
Henio, Capital 40,000
Cash 120,000
Other Assets 120,000
AA 1 - Chapter 2 (2008 edition)
page 7

6. Israel, Capital 140,000


Henio, Capital 140,000

PROBLEMS

Problem 2 - 1

1. Locsin, Capital (P240,000 x 1/4) 60,000


Montes, Capital (P120,000 x 1/4) 30,000
Nava, Capital 90,000
2. Locsin, Capital (P240,000 x 1/3) 80,000
Montes, Capital (P120,000 x 1/3) 40,000
Nava, Capital 120,000
3. Other Assets 180,000
Locsin, Capital (P180,000 x 3/4) 135,000
Montes, Capital (P180,000 x 1/4) 45,000
P540,000 – P360,000 = P180,000

Locsin, Capital [(P240,000 + P135,000) 1/3] 125,000


Montes, Capital [(P120,000 + P45,000) 1/3] 55,000
Nava, Capital 180,000
4. Cash 180,000
Locsin, Capital (P90,000 x 3/4) 67,500
Montes, Capital (P90,000 x 1/4) 22,500
Nava, Capital 270,000
AC CC Bonus
old (1/2) 270,000 360,000 (90,000)
new (1/2) 270,000 180,000 90,000
540,000 540,000 -----

5. Cash 180,000
Other Assets 180,000
Nava, Capital 180,000
Locsin, Capital (P60,000 x 3/4) 135,000
Montes, Capital (P60,000 x 1/4) 45,000
AC CC Asset Rev
old (3/4) 540,000 360,000 180,000
new (1/4) 180,000 180,000 -----
720,000* 540,000 180,000
*180,000 ÷ 1/4 = 720,000

6. Cash 240,000
Nava, Capital 180,000
Locsin, Capital (P60,000 x 3/4) 45,000
Montes, Capital (P60,000 x 1/4) 15,000
AA 1 - Chapter 2 (2008 edition)
page 8

7. Cash 240,000
Locsin, Capital 54,000
Montes, Capital 18,000
Nava, Capital 312,000

8. Cash 150,000
Locsin, Capital (P22,500 x 3/4) 16,875
Montes, Capital (P22,500 x 1/4) 5,625
Nava, Capital (P510,000 x 1/4) 127,500

9. Cash 165,000
Other Assetsl (P660,000 – P525,000) 135,000
Locsin, Capital (P135,000 x 3/4) 101,250
Montes, Capital (P135,000 x 1/4) 33,750
Nava, Capital (P660,000 x 1/4) 165,000

10 Cash 144,000
Locsin, Capital (P24,000 x 3/4) 18,000
Montes, Capital (P24,000 x 1/4) 6,000
Nava, Capital (P504,000 x 1/3) 168,000

Problem 2 - 2
1.
a. Ponce, Capital (P300,000 x ½) 150,000
Anton, Capital 150,000

b. Ponce, Capital (P300,000 x ¼) 75,000


Salva, Capital (P200,000 x ¼) 50,000
Victa, Capital (P100,000 x ¼) 25,000
Anton, Capital 150,000

c. Cash 220,000
Ponce, Capital 7,500
Salva, Capital 4,500
Victa, Capital 3,000
Anton, Capital 205,000
AC CC Bonus
Ponce P307,500 P300,000 P 7,500
Salva 204,500 200,000 4,500
Victa 103,000 100,000 3,000
Anton 205,000 220,000 ( 15,000)
P820,000 P820,000 ------
2.
a. Other Assets 360,000
Ponce, Capital 180,000
Salva, Capital 108,000
Victa, Capital 72,000
P960,000 – P600,000 = P360,000
AA 1 - Chapter 2 (2008 edition)
page 9

Ponce, Capital 240,000


Anton, Capital 240,000

b. Other Assets 120,000


Ponce, Capital 60,000
Salva, Capital 36,000
Victa, Capital 24,000
P180,000/ 25% = P720,000 – P600,000 = P120,000

Ponce, Capital 90,000


Salva, Capital 59,000
Victa, Capital 31,000
Anton, Capital 180,000

c. Other Assets 60,000


Ponce, Capital 30,000
Salva, Capital 18,000
Victa, Capital 12,000
P220,000/ 25% = P880,000 – P820,000 = P60,000

Cash 220,000
Anton, Capital 220,000

Problem 2-3
1.a Cash 90,000
Cabral, Capital 22,500
Corpus, Capital 18,000
Carlos, Capital 4,500
Other Assets 45,000
Camus, Capital 90,000
AC CC Asset Rev
old (3/4) 630,000 675,000 (45,000)
new (1/4) 90,000 90,000 -----
720,000* 765,000 (45,000)

b. Cash 90,000
Cabral, Capital 2,813
Corpus, Capital 2,250
Carlos, Capital 562
Camus, Capital 95,625
AC CC Bonus
old (1/2) 669,375 675,000 (5,625)
new (1/2) 95,625 90,000 5,625
765,000 765,000 -----

2.a Cabral, Capital 40,500


Corpus, Capital 27,000
Carlos, Capital 16,875
Camus, Capital 84,375
AA 1 - Chapter 2 (2008 edition)
page 10

b. Other Assets 45,000


Cabral, Capital 22,500
Corpus, Capital 18,000
Carlos, Capital 4,500
P90,000/ 1/8 = P720,000 – P675,000 = P45,000

Cabral, Capital 43,312


Corpus, Capital 29,250
Carlos, Capital 17,438
Camus, Capital 90,000

Problem 2 - 4
1. a. Inventories 5,625
Accumulated Depreciation – Equipment 7,500
Allowance for Doubtful Accounts 3,450
Accrued Liabilities 2,925
Roces, Capital (P6,750 x 60/100) 4,050
Lapuz, Capital (P6,750 x 40/100) 2,700

b. Cash 46,875
Doria, Capital 46,875
P187,500/80% = P234,375 x 20% = P46,875
c. Lapuz, Capital 13,388
Roces, Capital 13,388
Roces = (P234,375 x 50%) – P103,800 = P13,388
Lapuz = (P234,375 x 30%) - P83,400 = (P13,388)
2.
Roces, Lapuz and Doria
Statement of Financial Position
April 1, 2008

ASSETS LIABILITIES and PARTNERS’ CAPITAL


Cash P 82,875 Payables P66,750
Receivables P69,000 Accrued Liabilities 2,925
Less Allow. for DA 3,450 65,550 Roces, Capital P117,188
Inventories 129,375 Lapuz, Capital 70,312
Equipment P52,500 Doria, Capital 46,875 234,375
Less Acc. Depr. 26,250 26,250 ________
TOTAL LIABILITIES and
TOTAL ASSETS P304,050 PARTNERS’ CAPITAL P304,050

Problem 2 -5
Roldan Angeles Lazaro Moreno Total
Bal.before admission of Moreno P150,000 P180,000 P300,000 P630,000
Transfer of 1/6 int. to Moreno (30,000) P 30,000
Investment of Moreno 150,000 150,000
Asset revaluation 6,000 6,000 8,000 20,000
Bonus to old partners 6,000 6,000 8,000 (20,000)
Capital balances after admission
of Moreno P162,000 P162,000 P316,000 P160,000 P800,000
AA 1 - Chapter 2 (2008 edition)
page 11

2. Roldan 30% x 75% = 22.5%


Angeles 30% x 75% = 22.5%
Lazaro 40% x 75% = 30%
Moreno 25%

Problem 2 – 6

1. Lazo, Capital 19,000


Madrid, Capital 19.000
Buildings 8,000
Allowance for Doubtful Accounts 20,000
Allowance for Valuation of Investments 10,000

Lazo, Capital 60,000


Madrid, Capital 45,333
Nuguid, Capital 105,333
(P200,000 – P19,000 + P19,000 – P20,000) 1/3 = P60,000
(P150,000 – P19,000 + P19,000 – P14,000) 1/3 = P45,333

2. Lazo Madrid Total


Capital balances before admission of Nuguid P199,000 P155,000 P354,000
Revaluation of assets ( 19,000) ( 19,000) ( 39,000)
Capital balances after revaluation P180,000 P136,000 P316,000
Fraction of interest transferred to Nuguid x 1/3 x 1/3 x 1/3
Interest transferred to Nuguid P 60,000 P 45,333 P105,333
Gain on transfer 31,138 23,529 54,667
Cash distribution to partners P 91,138 P 68,862 P160,000

3. Lazo Madrid Nuguid


Capital balances before admission of Nuguid P199,000 P155,000
Revaluation of assets ( 19,000) ( 19,000)
Interest transferred to Nuguid ( 60,000) ( 45,333) 105,333
Balances P120,000 P 90,667 P105,333
Share in net profit 18,000 18,000 18,000
Drawings ( 15,000) ( 12,000) ( 28,000)
Capital balances, December 31, 2008 P123,000 P 96,667 P 95,333

4. Cash 66,000
Accounts Receivable 40,000
Investments 20,000
Accounts Payable 41,000
Osorio, Capital 85,000

Lazo, Capital 5,000


Madrid, Capital 5,000
Nuguid, Capital 5,000
Osorio, Capital 15,000
P315,000 + P85,000 = P400,000 x 1/4 P100,000 – P85,000 = P15,000
AA 1 - Chapter 2 (2008 edition)
page 12

Problem 2 - 7

1. Montero, Capital 100,000


Concio, Capital (P8,000 x 3/5) 4,800
Domino, Capital (P8,000 x 2/5) 3,200
Cash 108,000

2. Montero, Capital 100,000


Concio, Capital (P10,000 x 3/5) 6,000
Domino, Capital (P10,000 x 2/5) 4,000
Cash 90,000

3. Montero, Capital 100,000


Concio, Capital (P60,000 x 3/6) 30,000
Domino, Capital (P60,000 x 2/6) 20,000
Cash 90,000
Other Assets (P10,000 ÷ 1/6) 60,000

4. Montero, Capital (P6,000 x 1/6) 1,000


Concio, Capital (P6,000 x 3/6) 3,000
Domino, Capital (P6,000 x 2/6) 2,000
Equipment [(P60,000 x 40%) – P18,000] 6,000

Montero, Capital (P100,000 – P1,000) 99,000


Equipment 18,000
Cash 81,000

Problem 2-8

1. Damaso Dangwa Datu


Capital, January 1, 2008 P120,000 P 70,000 P 80,000
Share in net loss ( 9,600) ( 6,400) ( 16,000)
Drawings ( 24,000) ( 24,000) ( 24,000)
Capital balances, December 31, 2008 P 86,400 P 39,600 P 40,000

2. Dangwa, Capital 39,600


Dmaso, Capital 14,400
Datu, Capital 24,000
Cash 30,000
Inventory 48,000
P39,600 – P30,000 = P9,600 / 20% = P48,000

3. a Other Assets 42,000


Dangwa, Capital 39,600
Cash 48,000
Damaso, Capital 12,600
Datu, Capital 21,000
P48,000 – P39,600 = P8,400/ 20% = P42,000
AA 1 - Chapter 2 (2008 edition)
page 13

Dangwa, Capital 39,600


Damaso, Capital 3,150
Datu, Capital 5,250
Cash 48,000

Problem 2 - 9

1. Cash 120,000
Luna, Capital 2,000
Matias, Capital 2,000
Noble, Capital 2,000
Guzman, Capital 126,000
AC CC Bonus
Old P294,000 P300,000 P( 6,000)
New 126,000 120,000 6,000
P420,000 P420,000 ----

2. Cash 60,000
Luna, Capital 20,000
Matias, Capital 20,000
Noble, Capital 20,000
Other Assets 60,000
Guzman, Capital 60,000
AC CC Asset Rev
Old P240,000 P300,000 (P60,000)
New 60,000 60,000
P300,000 P360,000 (P60,000)

3. Matias, Capital 36,000


Guzman, Capital 36,000
P120,000 x 30% = P36,000

4. Luna, Capital 80,000


Matias, Capital 8,000
Noble, Capital 8,000
Cash 96,000

5. Luna, Capital 80,000


David, Capital 80,000

6. Luna, Capital 80,000


Matias, Capital 40,000
Noble, Capital 40,000
AA 1 - Chapter 2 (2008 edition)
page 14

Problem 2 -10
Canda, Pardo and Andres
Statement of Changes in Partners’ Equity
For the Period January 1, 2006 to January 1, 2009

Canda Pardo Andres Total


Original capital, January 1, 2006 P 62,500 P 25,000 P 12,500 P 100,000
Corrected 2006 net profit 26,375 10,550 5,275 42,200
Drawings (15,000) ( 7,800) ( 5,200) ( 28,000)
Capital, January 1, 2007 P 73,875 P 27,750 P 12,575 P 114,200
Corrected 2007 net profit 10,875 4,350 2,175 17,400
Drawings (15,000) ( 7,800) ( 5,200) ( 28,000)
Capital, January 1, 2008 P 69,750 P 24,300 P 9,550 P 103,600
Corrected 2008 net loss ( 6,750) ( 2,700) ( 1,350) ( 10,800)
Drawings (10,000) ( 5,200) ( 5,200) ( 20,400)
Capital, January 1, 2009 P 53,000 P 16,400 P 3,000 P 72,400

Schedule of computation of corrected net profit


2006 2007 2008
Reported net profit (loss) P 44,000 P 18,500 P (10,500 )
Understatement of accrued expenses 2006 ( 400 ) 400
2007 ( 500 ) 500
2008 ( 650 )
Understatement of accrued revenues 2006 250 ( 250 )
2007 100 ( 100 )
2008 150
Overstatement of inventories 2006 ( 1,500 ) 1,500
2007 ( 2,000 ) 2,000
2008 ( 2,000 )
Understatement of depreciation exp. ( 150 ) ( 350 ) ( 200 )
Corrected net profit (loss) P 42,200 P 17,400 P (10,800 )

2. a. Revenue Receivable 150


Canda, Capital 2,000
Pardo, Capital 800
Andres, Capital 400
Expenses Payable 650
Merchandise Inventory 2,000
Accumulated Depreciation 700

b. Canda, Capital (P3,000 x 625/1000) 1,875


Pardo, Capital (P3,000 x 250/1000) 750
Andres, Capital (P3,000 x 125/1000) 375
Furniture (P4,500 - P1,500) 3,000

c. Andres, Capital 2,625


Furniture 1,500
Cash 1,125
AA 1 - Chapter 2 (2008 edition)
page 15

Problem 2 -11

Abelar and Berces


Statement of Changes in Partners’ Equity
For the Period January 1, 2007 to January 15, 2009

Abelar Berces Custodio Total


Capital balances before closing the
books, December 31, 2007 P 50,000 P 30,000 P 80,000
Net profit for 2007 (Sch 1) 6,600 7,400 14,000
Drawing (8,200) (6,800) (15,000)
Capital, December 31, 2007 P 48,400 P 30,600 P 79,000
Admission of Custodio (Sch. 2) (7,800) (5,200) P 33,000 20,000
Net loss for 2008 (5,250) (3,750) (6,000) (15,000)
Drawings (7,500) (5,000) (6,800) (19,300)
Capital, December 31, 2008 P 27,850 P 16,650 P 20,200 P 64,700
Loss on realization on Jan. 15, 2009 (16,520) (11,800)` (18,880) (47,200)
Final cash distribution P 11,330 P 4,850 P 1,320 P 17,500

Schedule 1 - Distribution of 2007 net profit

Abelar_ Berces Total


Salaries P 9,000 P 9,000 P 18,000
Balance - 60%, 40% (2,400) (1,600) ( 4,000)
Total P 6,600 P 7,400 P 14,000

Schedule 2 - Admission of Custodio

Total capital contribution (P79,000 + P20,000) P 99,000


Interest to be credited to Custodio 1/3__
Capital credit of Custodio P 33,000
Capital contribution of Custodio 20,000
Bonus to Custodio from Abelar and Berces P 13,000

MULTIPLE CHOICE

1. B
2. A
3. B P264,000 – [(P278,000 + P418,000 + P192,000) x 1/5] = P86,400

4. A Lima = P100,000 x 80% = P80,000


Mitra = P 50,000 x 80% = P40,000

5. A Asset revaluationP60,000/20% = P300,000 - P150,000 P150,000


Lima = [P100,000 + (P150,000 x 75%)] x 80% P170,000
Mitra = [P 50,000 + (P150,000 x 25%)] x 80% P 70,000
Nova P 60,000
AA 1 - Chapter 2 (2008 edition)
page 16

6. D Felix Elias Total


Original investment P 24,000 P 48,000 P 72,000
Net profit 5,430 10,860 16,290
Drawings ( 5,050) ( 8,000) ( 13,050)
Capital bal . before transfer to Desta P 24,380 P 50,860 P 75,240
Required capital based on orig. capital
ratio after transfer to Desta of 1/4 int. 18,810 37,620 56,430
Capital to be transferred to Desta P 5,570 P 13,240 P 18,810
Excess cash to be dist. based on orig.
capital ratio (P30,000 - P18,810) 3,730 7,460 11,190
Distribution of cash to Felix and Elias P 9,300 P 20,700 P 30,000

7. D
8. C
9. C P90,000 – P75,000 = P15,000
10. A Capital of Mison prior to admission of Zamora P105,000
Share in the bonus from Zamora [(P90,000 – P75,000) 1/2) 7,500
Capital of Mison in the new partnership P112,500

11. C AC CC Asset Rev


Voltaire P180,000 P150,000 P30,000
Asuncion 210,000 180,000 30,000
Leonor 195,000 195,000
P585,000 P525,000 P60,000

12. D P195,000 – (P525,000 x 1/3 = P175,000) = P20,000


13. A Old partners’ capital contribution P600,000
Percentage of interest of old partners ÷ 75%
Total agreed capital of the new partnership P800,000
Percentage of interest of Sison x 25%
Capital credit of Sison P200,000
Bonus to Sison 70,000
Cash to be contributed by Sison P130,000

14. C Rivera Sanchez Torres


Capital balances before adm. of Vidal P504,000 P252,000 P 84,000
Asset revaluation
(P180,000/20% ) – P840,000 = P60,000 36,000 18,000 6,000
Adjusted capital balances P540,000 P270,000 P 90,000
Percentage of unsold interest x 80% x 80% x 80%
Capital balances after adm. of Vidal P432,000 P216,000 P 72,000

15. D Total capital of the new partnership (P840,000/75%) P1,120,000


Percentage of interest x 25%
Amount to be invested by Vidal in the partnership P 280,000

16. B Agreed capital P330,000


Capital contribution = P95,000 + P80,000 + P60,000 + P80,000 = 315,000
Asset revaluation P 15,000
AA 1 - Chapter 2 (2008 edition)
page 17

17. A P80,000 + P12,000 – P70,000 = P22,000

18. C Capital balance before admission of Manalo P 80,000


Interest sold to Manalo (P80,000 x 15%) (12,000)
Share in the recorded asset revaluation (P15,000 x 3/10) 4,500
Share in the bonus from Manalo
[(P80,000 + P12,000) - P70,000] x 3/10 6,600
Capital balance after admission of Manalo P 79,100

19. B Juan Cosme Luna Magno


Capital balances, April 30, 2008 P 360,000 P 225,000 P 135,000
1/6 Interest transferred to Magno ( 60,000) ( 37,500) ( 22,500) P 120,000
Balances P 300,000 P 187,500 P 112,500 P 120,000
Cash transfers to equalize investment ( 100,000) 12,500 87,500
Balances P 200,000 P 200,000 P 200,000 P 120,000
Distribution of net profit -equally 3,150 3,150 3,150 3,150
Withdrawals ( 1,500) ( 2,000) ( 1,500) ( 2,000)
Capital, June 30, 2008 P 201,650 P 201,150 P 201,650 P121,150

20. A Agreed capital = (P201,650 + P201,150 + P201,650) ÷ 3/4 = P805,933


Interest of Magno 1/4___
Required capital credit of Magno P201,483
Capital balance of Magno before investing cash 121,150
Cash to be invested by Magno P 80,333

21. A Galang Hizon Isleta


Asset revaluation method:
Capital contributions of partners P600,000 P480,000 P500,000
Asset revaluation 252,000 168,000
Additional depreciation ( 140,000) ( 140,000) ( 140,000)
Capital balances P712,000 P508,000 P360,000
Bonus method:
Capital contributions of partners P600,000 P480,000 P500,000
Bonus to old partners from new partner 63,000 42,000 ( 105,000)
Capital balances P663,000 P522,000 P395,000
Net advantage of bonus method to Isleta P 35,000

22. A Campos Centeno


Capital balance P641,976 P728,352
Uncollectible accounts ( 20,000) ( 35,000)
Worthless inventories ( 5,500) ( 6,700)
Other assets written off ( 2,000) ( 3,600)
Adjusted capital P614,476 P683,052

23. C Total capital P614,476 +P683,052 P1,297,528


Total liabilities 967,590
P2,265,118
AA 1 - Chapter 2 (2008 edition)
page 18

24. D Total capital P1,297,528 / 80% P1,621,910


Interest of Coronel x 20%
Contribution of Coronel P 324,382

25. D Campos Centeno


Capital balances P614,476 P683,052
Required capital P1,297,528/2 648,764 648,764
Cash paid (received) P 34,288 (P34,288)

26. B Campos Centeno Coronel


Capital balances P614,476 P683,052 P324,382
Cash paid (received) 34,288 (34,288)
Net profit 130,000 130,000 65,000
Drawings (50,000) (65,000) (28,000)
P728,764 P713,764 P361,382

27. C The capital balances would be the same as the balances prior to sale of interest.
28. C
29. D
30. D P4,000 x 2/5 = P1,600
31. D P3,000 / 40% = P7,500
32. A P12,000/3 = P4,000

33. C Yumul Yason Ylagan


Interest before retirement P103,000 P 77,000 P180,000
Adjustment of assets to FMV 12,000 12,000 24,000
P115,000 P 89,000 P204,000
Retirement of Yumul (115,000) ( 2,000) ( 4,000)
Capital balance of Ylagan P200,000

34. A Amount paid to retiring partner P28,000


Capital of retiring partner
Total capital before retirement P110,000
Total capital after retirement 90,000 20,000
Asset revaluation to retiring partner P 8,000
Fraction of interest of retiring partner ÷ 2/10
Total asset revaluation P40,000
CHAPTER 3
Partnership Liquidation

EXERCISES

Exercise 3 - 1

Aguilar Benito Casimiro David


Capital balances before liquidation P 11,000 P 10,300 P 13,700 P 9,000
Loan from partners 2,000
Total partners’ interest P 13,000 P 10,300 P 13,700 P 9,000
Loss on realization (P46,000 – P12,000) (13,600) ( 10,200) ( 6,800) ( 3,400)
Balances P( 600) P 100 P 6,900 P 5,600
Additional loss to partners 600 ( 300) ( 200) ( 100)
Balances -------- P ( 200) P 6,700 P 5,500
Additional loss to partners 200 ( 133) ( 67)
Distribution of cash to partners --------- --------- 6,567 5,433

Exercise 3 - 2

Duque Espino Felipe Total


Original investments P 50,000 P 22,500 P 20,000 P 92,500
Net income for 2007 15,000 7,500 7,500 30,000
Drawings in 2007 ( 15,000) ( 10,000) ( 10,000) ( 35,000)
Total partners’ interest before dissolution P 50,000 P 20,000 P 17,500 P 87,500
Net assets distributed to partners ( 32,500) ( 16,250) ( 16,250) ( 65,000)
Balances P 17,500 P 3,750 P 1,250 P 22,500
Loss to partners distributed 2:1:1 ( 11,250) ( 5,625) ( 5,625) ( 22,500)
Cash settlement among partners P 6,250 P( 1,875) P( 4,375) ------

Exercise 3 - 3

1. Guarin, Capital 1,500


Receivable from Guarin 1,500
To offset receivable from Guarin against his capital.

2. Salary Payable to Henson 500


Henson, Capital 500
To include salary payable to Henson to his interest.

3. Henson, Capital (P24,500 x 40%) 9,800


Guarin, Capital (P24,500 x 60%) 14,700
Loss from Liquidation 24,500
To distribute loss from liquidation to partners.

4. Henson, Capital (P9,500 + P500 - P9,800) 200


Guarin, Capital (P18,000 - P1,500 - P14,700) 1,800
Cash 2,000
AA1 - Chapter 3 (2008 edition) page
2

Exercise 3 - 4
1. Ibarra Javier Katindig
Original investment P 60,000 P 54,000 P 16,000
Net loss for six months* (18,000) (12,000) ( 6,000)
Loss on realization (P121,000 - P49,000 = P72,000) (36,000) (24,000) (12,000)
Balances P 6,000 P 18,000 P( 2,000)
Additional loss to partners ( 1,200) ( 800) 2,000
Cash distribution to Ibarra ( 4,800)

* Total capital, March 1 (P60,000 + P54,000 + P16,000) P130,000


Net assets, Aug. 31 (P5,000 + P121,000 - P32,000) 94,000
Net loss P 36,000

2. Book value of other assets P121,000


Total loss on realization
Capital balance of Katindig after dist. of net loss P 10,000
Excess of personal liabilities over personal assets ( 5,000)
Maximum amount of loss that can be absorbed by Katindig P 5,000
Fractional share of Katindig 1/6__ ( 30,000)
Cash that must be realized on sale of other assets P 91,000

Exercise 3 – 5
1. Book value of other assets (P459,000 – P3,000) P456,000
Cash realized:
Accounts receivable [P180,000 – (P60,000 x 20%)] P168,000
Merchandise inventory 75,000
Prepaid advertising 2,400
Machinery and equipment (P120,000 x 60%) 72,000 317,400
Loss on realization P138,600

Lesaca – Manalo Partnership


Statement of Liquidation
December 31, 2008

Other Liabilities Capital


Cash Assets AP NP Lesaca Manalo
Balances before liquidation P 3,000 P456,000 P60,000 P258,000 P90,000 P 51,000
Sale of assets and distribution 317,400 ( 456,000) ( 55,440) ( 83,160)
of loss
Balances P320,400 P60,000 P258,000 P34,560 (P32,160)
Payment of liabilities ( 320,400) ( 59,400) ( 258,000) ( 1,200) ( 1,800)
Balances P600 P33,360 (P33,960)
Additional investment by 12,000 12,000
Manalo
Balances P 12,000 P 600 P33,360 (P21,960)
Payment of liabilities ( 600) ( 600)
Balances P 11,400 P33,360 (P21,960)
Additional loss to Lesaca ( 21,960) 21,960
Payment to Lesaca P 11,400 P11,400
AA1 - Chapter 3 (2008 edition) page
3

Exercise 3 – 6
Nocum Oliva Pascua Quinto
Capital balances before liquidation P180,000 P300,000 P240,000 (P 33,000)
Restricted interest – possible loss
Non-cash assets P600,000
Liquidation expenses 9,000
Unrecorded liabilities 15,000
Total P624,000 ( 156,000) ( 156,000) ( 156,000) ( 156,000)
Balances P 24,000 P144,000 P 84,000 (P189,000)
Restricted interest – possible loss to
Nocum, Oliva and Pascua for the
deficiency of Quinto ( 63,000) ( 63,000) ( 63,000) 189,000
Balances (P 39,000) P 81,000 P 21,000 -
Restricted interest – possible loss to
Oliva and Pascua for the deficiency of
Nocum 39,000 ( 19,500) ( 19,500)
Safe payment - P 61,500 P 1,500 -
AA1 - Chapter 3 (2008 edition) page
4

Exercise 3 - 8
Rama, Sison and Toledo
Cash Priority Program

PAYMENTS
Rama Sison Toledo Rama Sison Toledo
Capital balances P30,000 P70,000 P40,000
Add Loan balances 20,000 20,000 30,000
Total partners’ interest P50,000 P90,000 P70,000
Profit and loss ratio 40% 40% 20%
Loss absorption balance P125,000 P225,000 P350,000
Allocation I – Cash to Toledo
reducing LAB to an amount
reported for Sison
(P125,000 x 20%) (125,000) P25,000
Balances P125,000 P225,000 P225,000
Allocation II - Cash to Sison &
Toledo reducing LAB to an amount
reported for Rama
P100,000 x 40% (100,000) P40,000
P100,000 x 20% (100,000) 20,000
Balances P125,000 P125,000 P125,000 P40,000 P45,000
Allocation III - Further cash
distribution may be made in the
P & L ratio

Exercise 3-9
1. Julian, Lagman and Magno
Cash Priority Program
January 1, 2008
PAYMENTS
Julian Lagman Magno Julian Lagman Magno
Capital balances before liquidation P 36,000 P 54,000 P18,000
Add Note payable to Magno 14,000
Total partners’ interest P 36,000 P 54,000 P 32,000
Profit and loss ratio 3/10 3/10 4/10
Loss absorption balances P120,000 P180,000 P80,000
Allocation I – Cash to Lagman reducing
LAB to an amount reported for Julian
(P60,000 x 3/10) (60,000) P18,000
Balances P120,000 P120,000 P80,000
Allocation II – Cash to Julian & Lagman
reducing LAB to an amount reported for
Magno (P40,000 x 3/10) ( 40,000) (40,000) P12,000 12,000
Balances P80,000 P80,000 P80,000 P12,000 P20,000 -
Allocation III – Further cash distributions
may be made in the P & L ratio
AA1 - Chapter 3 (2008 edition) page
5

2. Julian, Lagman and Magno


Statement of Liquidation
January to March, 2008
Other NP to PAYMENTS
Cash Assets Liabilities Magno Julian Lagman Magno
Balances before liquidation P12,000 P146,000 P36,000 P14,000 P36,000 P54,000 P18,000
January:
Sale of assets and dist. Of
loss 30,000 ( 38,000) ( 2,400) ( 2,400) ( 3,200)
Payment of liquidation
expenses ( 3,600) ( 1,080) (1,080 (1,440)
Payment of liabilities ( 36,000) (36,000)
Distribution of cash to
partners (sch. 1) ( 2,400) (2,400)
Balances P108,000 P14,000 P32,520 P48,120 P13,360
February:
Sale of assets and
distribution of gain 44,000 (35,000) 2,700 2,700 3,600
Payment of liquidation
expenses (8,400) (2,520) (2,520) (3,360)
Distribution of cash to
partners (sch. 2) (35,600) (10,000) (25,600)
Balances P73,000 P14,000 P22,700 P22,700 P13,600
March:
Sale of assets and
distribution of loss 36,000 (73,000) (11,100) (11,100) (14,800)
Balances P36,000 P14,000 P11,600 P11,600 P(1,200)
Offset of loan against
deficiency ( 1,200) 1,200
Final payment to partners (P36,000) (P12,800) (P11,600 (P11,600
) )

Schedule 1
Installment Liquidation
January 31, 2008

Amount Julian Lagman Mango


Cash available P2,400
Allocation I – Payable to Lagman P2,400 P2,400

Schedule 2
Installment Liquidation
February 29, 2008

Amount Julian Lagman Mango


Cash available P2,400
Allocation I – Balance
Payable to Lagman P2,400 P2,400
Allocation II – Payable to Julian and
Lagman P20,000 P10,000 10,000
P10,000 P25,600 -
AA1 - Chapter 3 (2008 edition) page
6

3. Journal entries
January Cash 30,000
Julian, Capital 2,400
Lagman, Capital 2,400
Magno, Capital 3,200
Other Asset 38,000

Julian, Capital 1,080


Lagman, Capital 1,080
Magno, Capital 1,440
Cash 3,600

Liabilities 36,000
Cash 36,000

Lagman, Capital 2,400


Cash 2,400

February Cash 44,000


Other assets 35,000
Julian, Capital 2,700
Lagman, Capital 2,700
Magno, Capital 3,600

Julian, Capital 2,520


Lagman, Capital 2,520
Magno, Capital 3,360
Cash 8,400

Julian. Capital 10,000


Lagman, Capital 25,600
Cash 35,600

March Cash 36,000


Julian, Capital 11,100
Lagman, Capital 11,100
Magno, Capital 14,800
Other assets 73,000

Note Payable to Magno 1,200


Magno, Capital 1,200

Note Payable to Magno 12,800


Julian, Capital 11,600
Lagman, Capital 11,600
Cash 36,000
AA1 - Chapter 3 (2008 edition) page
7

Exercise 3 - 10
U, V and W Co.
Cash Priority Program
PAYMENTS
Urbe Villa Waldo Urbe Villa Waldo
Capital balances P 11,200 P13,000 P 5,800
Profit and loss ratio 4/7 2/7 1/7
Loss absorption balance P 19,600 P 45,500 P 40,600
Allocation I - Cash to Villa reducing
LAB to an amount reported for
Waldo (P4,900 x 2/7) ( 4,900) P 1,400
Balances P 19,600 P 40,600 P 40,600
Allocation II - Cash to Villa & Waldo
reducing LAB to an amount
reported for Urbe
P21,000 x 2/7 ( 21,000) 6,000
P21,000 x 1/7 (21,000) P 3,000
Balances P 19,600 P 19,600 P 19,600 P 7,400 P 3,000
Allocation III - Further cash distribution
may be made in the P & L ratio

2. Book value of assets P 30,000


Loss on realization:
Capital balance of Urbe prior to realization P 11,200
Cash to be received by Urbe 10,000
Share of Urbe in the loss on realization P 1,200
Fractional share of Urbe 4/7_ 2,100
Cash to be realized of the sale of assets P 27,900

3. Allocation III - P3,200 ÷ 4/7 = P5,600 x 1/7 P 800


Allocation II 3,000
Total cash received by Waldo P 3,800

4. Book value of assets P 30,000


Total cash available
Allocation I P 1,400
Allocation II - P1,800 - P1,400 = P400 ÷ 2/3 600 2,000
Loss on liquidation P 28,000

Exercise 3 – 11
Partnership Books
1. Inventories 90,000
Capital Adjustment Account 90,000

2. Accumulated Depreciation 160,000


Equipment 80,000
Capital Adjustment Account 80,000

3. Goodwill 56,000
Capital Adjustment Account 56,000
P980,000 – P924,000 = P56,000
AA1 - Chapter 3 (2008 edition) page
8

4. Capital Adjustment Account 226,000


Belen, Capital (3/4) 169,500
Bgnes, Capital (1/4) 56,500

5. Colored Co. Stocks 980,000


Allowance for Uncollectible Accounts 12,000
Accounts Payable 104,000
Accounts Receivable 124,000
Inventories 296,000
Equipment 520,000
Goodwill 156,000

6. Belen, Capital 563,500


Bagnes, Capital 458,500
Cash 42,000
Colored Co. Stocks 980,000

New Corporation’s Books


1. Authorized to issue 50,000 shares of P50 par value Ordinary Share Capital.

2. Cash 700,000
Ordinary Share Capital 500,000
PIC in Excess of Par 200,000

3. Accounts Receivable 124,000


Inventories 296,000
Equipment 520,000
Goodwill 156,000
Allowance for Doubtful Accounts 12,000
Accounts Payable 104,000
Ordinary Share Capital 700,000
PIC in Excess of Par 280,000
AA1 - Chapter 3 (2008 edition) page
9

Problem 3 – 2 (Case 3 – cont.)

Calma, Daza and Esteban


Schedule of Cash Distribution to Partners

Calma Daza Esteban


Capital balances before cash distribution P 27,000 P ( 3,000) P 46,000
Add loan balance 8,000
Total partners’ interest P 27,000 P ( 3,000) P 54,000
Restricted interest - possible loss to Calma and Esteban
in the ratio of 2:1 if Daza fails to pay his deficiency ( 2,000) 3,000 ( 1,000)
Free interests - amounts to be paid to partners P 25,000 - P 53,000
Payment to apply on:
Loan P 8,000
Capital P 25,000 45,000
Cash distribution P 25,000 - P 53,000
AA1 - Chapter 3 – Partnership Liquidation (2005)
Suggested Answers page

Problem 3 – 2 (Case 4 – cont.)

Calma, Daza and Esteban


Schedule of Cash Distribution to Partners

Calma Daza Esteban


Capital balances before cash distribution P 9,000 P (21,000) P 37,000
Add loan balance 8,000
Total partners’ interest P 9,000 P (21,000) P 45,000
Restricted interest - possible loss to Calm and Esteban in
the ratio of 2:1 if Daza fails to pay his deficiency (14,000) 21,000 ( 7,000)

Balances P( 5,000) - P 38,000


Restricted interest - possible loss to Esteban if Calma fails
to pay his deficiency 5,000 - ( 5,000)
Free interests - amounts to be paid to partners - - -
Payment to apply on:
Loan P 8,000
Capital 25,000
Cash distribution - - P 33,000
Chapter 3 – Partnership Liquidation
Suggested Answers page

Problem 3 - 3

1. a. Cash 48,000
Accumulated Depreciation 25,000
Fuentes, Capital (P72,000 x 5/15) 24,000
Goco, Capital (P72,000 x 5/15) 24,000
Herrera, Capital (P72,000 x 3/15) 14,400
Isla, Capital (P72,000 x 2/15) 9,600
Merchandise Inventory 55,000
Accounts Receivable 60,000
Store Fixtures 30,000

b. Accounts Payable 76,000


Cash (P28,000 + P48,000) 76,000

c. Fuentes, Capital 4,500


Herrera, Capital 2,700
Isla, Capital 1,800
Goco, Capital 9,000

d. Fuentes, Capital 1,500


Isla, Capital 600
Herrera, Capital 2,100

e. Fuentes, Loan 2,000


Isla, Loan 3,000
Fuentes, Capital 2,000
Isla, Capital 3,000

f. Cash 6,000
Fuentes, Capital 1,000
Herrera, Capital 5,000

g. Accounts Payable 4,000


Cash 4,000

h. Isla, Loan 2,000


Cash 2,000

2. a. Accounts Payable 4,000


Fuentes, Capital 4,000

b. Isla, Loan 2,000


Fuentes, Capital 3,000
Herrera, Capital 5,000
Chapter 3 – Partnership Liquidation
Suggested Answers page

Problem 3 – 3 (cont.)

3. a. Accounts Payable 4,000


Herrera, Capital 4,000

b. Isla, Capital 2,000


Fuentes, Capital 1,000
Herrera, Capital 1,000

Schedule to support the entries in Requirement 1

L O A N C A P I T A L
Fuentes Isla Fuentes Goco Herrera Isla
Balances before liquidation P 2,000 P 5,000 P27,000 P15,000 P10,000 P 9,000
Distribution of loss ( 24,000) ( 24,000) ( 14,400) ( 9,600)
Balances P 2,000 P 5,000 P 3,000 P( 9,000) P( 4,400) P( 600)
Additional loss for the
deficiency of Goco ( 4,500) 9,000 ( 2,700) ( 1,800)
Balances P 2,000 P 5,000 P( 1,500) - P( 7,100) P( 2,400)
Additional loss for the
deficiency of Herrera ( 1,500) 2,100 ( 600)
Balances P 2,000 P 5,000 P( 3,000) - P( 5,000) P( 3,000)
Offset against debit balance
in capital account ( 2,000) ( 3,000) 2,000 - 3,000
Balances - P 2,000 P( 1,000) P( 5,000) -
Additional investment by
partners 1,000 5,000
Payment to Isla - P 2,000 - - - -
Chapter 3 – Partnership Liquidation
Suggested Answers page

Problem 3 -5

JKLM Trading Co.


Schedule To Accompany Statement of Liquidation
Amounts to be Paid to Partners
February 28, 2008

Jocson Kaimo Legarda Manabat


Capital balances before dist. of cash P 19,128 P 88,992 P 101,532 P 22,878
Add Loan balances 15,000
Total partners’ interest P 34,128 P 88,992 P 101,532 P 22,878
Restricted interest - possible loss if
nothing is realized on remaining assets ( 25,494) ( 38,241) ( 38,241) ( 25,494)
Balances P 8,634 P 50,751 P 63,291 P( 2,616)
Restricted interest - additional possible
loss if Manabat is unable to pay his
deficiency (20:30:30) ( 654) ( 981) ( 981) 2,616
Free interest - payments to partners P 7,980 P 49,770 P 62,310 -

Payment to apply on
Loan P 7,980
Capital P 49,770 P 62,310 -
Total cash distribution P 7,980 P 49,770 P 62,310 -

JKLM Trading Co.


Schedule To Accompany Statement of Liquidation
Amounts to be Paid to Partners
March 31, 2008

Jocson Kaimo Legarda Manabat


Capital balances before dist. of cash P 18,348 P 38,052 P 38,052 P 22,098
Add Loan balance 7,020
Total partners’ interest P 25,368 P 38,052 P 38,052 P 22,098
Restricted interest - possible loss if
nothing is realized on remaining assets ( 16,524) ( 24,786) ( 24,786) (16,524)
Free interest - payment to partners P 8,844 P 13,266 P 13,266 P 5,574

Payment to apply on:


Loan P 7,020
Capital 1,824 P 13,266 P 13,266 P 5,574
Total cash distribution P 8,844 P 13,266 P 13,266 P 5,574
AA1 -Chapter 3 – Partnership Liquidation
Suggested Answers page

Problem 3 – 6
QRS Partnership
Schedule to Accompany Statement of Liquidation
Amounts to be Paid to Partners
July 31, 2008
Quizon Roman Silva
Balances before cash distribution P116,250 P159,750 P151,500
Add Loan balance 150,000
Total partners’ interest P116,250 P309,750 P151,500
Restricted interest – possible loss of P480,000
on remaining unsold assets and cash
withheld of P30,000 ( 255,000) ( 153,000) ( 102,000)
Balances ( P138,750) P156,750 P 49,500
Restricted interest – possible loss of P138,750
to Roman and Silva 138,750 ( 83,250) ( 55,500)
Balances - P 73,500 (P 6,000)
Restricted interest – possible loss to Roman ( 6,000) 6,000
Payment to Roman to apply on loan P 67,500

QRS Partnership
Schedule to Accompany Statement of Liquidation
Amounts to be Paid to Partners
August 31, 2008
Quizon Roman Silva
Balances before cash distribution P 93,000 P145,800 P142,200
Add Loan balance 82,500
Total partners’ interest P 93,000 P228,300 P142,200
Restricted interest – possible loss of P375,000
on remaining unsold assets and cash
withheld of P30,000 ( 202,500) ( 121,500) ( 81,000)
Balances ( P109,500) P106,800 P 61,200
Restricted interest – possible loss of P109,500
to Roman and Silva 109,500 ( 65,700) ( 43,800)
Payment to Roman to apply on loan and to Silva
to apply on capital - P 41,100 P 17,400
AA1 -Chapter 3 – Partnership Liquidation
Suggested Answers page

Problem 3 - 7
Requirement 1
Tabora, Ureta and Veloso
Cash Priority Program
January 1, 2008
PAYMENTS
Tabora Ureta Veloso Tabora Ureta Veloso
Capital balances P120,000 P 90,000 P 40,000
Loan balances 45,000 30,000 13,000
Total partners’ interest P165,000 P120,000 P 53,000
Profit and loss ratio 50% 30% 20%
Loss absorption balance P330,000 P400,000 P265,000
Allocation I - Cash to Ureta to
reduce LAB to amount
reported for Tabora ( 70,000) P21,000
Balances P330,000 P330,000 P265,000
Allocation II - Cash to Tabora
and Ureta to reduce LAB to
amount reported for Veloso ( 65,000) ( 65,000) P32,500 19,500
Balances P265,000 P265,000 P265,000 P32,500 P40,500
Allocation III - Further cash
distribution may be made
based on P & L ratio

Requirement 2
Amount Tabora Ureta Veloso
January:
Cash available P15,000
Allocation I - payable to Ureta 15,000 P15,000
February:
Cash available P40,000
Allocation I - Bal. payable to Ureta 6,000 P 6,000
Allocation II - Payable to Tabora and
Ureta in the ratio of 50:30 P34,000 P21,250 12,750
P21,250 P18,750
March:
Cash available P90,000
Allocation II - Balance 18,000 P11,250 P 6,750
Allocation III - Based on P & L ratio P72,000 36,000 21,600 P14,400
P47,250 P28,350 P14,400
April:
Cash available P30,000
Allocation III - Based on P & L ratio 30,000 P15,000 P 9,000 P 6,000
AA1 -Chapter 3 – Partnership Liquidation
Suggested Answers page

Problem 3 – 8 (cont.)
Requirement 1
January: a. Cash 112,000
Accounts Receivable 112,000

b. Neri, Capital 2,200


Ordan, Capital 1,100
Pacia, Capital 1,100
Cash 4,400

c. Accounts Payable 38,000


Cash 38,000

d. Pacia, Loan 9,000


Pacia, Capital 7,000
Cash 16,000

February: a. Cash 36,000


Accounts Receivable 36,000

b. Neri, Capital 1,400


Ordan, Capital 700
Pacia, Capital 700
Cash 2,800

c. Accounts Payable 39,000


Cash 38,000
Neri, Capital 500
Ordan, Capital 250
Pacia, Capital 250

d. Salary Payable to Neri 6,000


Neri, Capital 1,400
Ordan, Capital 3,700
Pacia, Capital 8,700
Cash 19,800

March: a. Cash 35,000


Neri, Capital 4,000
Ordan, Capital 2,000
Pacia, Capital 2,000
Accounts Receivable 43,000

b. Neri, Capital 2,000


Ordan, Capital 1,000
Pacia, Capital 1,000
Cash 4,000
AA1 -Chapter 3 – Partnership Liquidation
Suggested Answers page

c. Neri, Capital 39,500


Ordan, Capital 19,750
Pacia, Capital 19,750
Cash 79,000

Problem 3 - 9

Requirement 1 Wilson, Yuson and Zapata


Cash Distribution Schedule
June 30, 2008
P AY M E N T S
Wilson Yuson Zapata Wilson Yuson Zapata
Capital balances P 67,000 P 45,000 P 31,500
Receivable from partners 12,000 7,500
Total partners’ interest P 55,000 P 45,000 P 24,000
Profit and loss ratio 50% 30% 20%
Loss absorption balance P110,000 P150,000 P120,000
Allocation I - Cash to
Yuson to reduce LAB
to amt. reported for Zapata 30,000 P 9,000
Balances P110,000 P120,000 P120,000
Allocation II - Cash to Zapata
and Yuson to reduce LAB
to amt. reported for Wilson 10,000 10,000 3,000 P 2,000
Balances P110,000 P110,000 P110,000
Allocation III - Based on P & L
ratio (P6,000 + P100,000 -
P17,000 = P89,000 - P14,000) P37,500 22,500 15,000
TOTALS P37,500 P 34,500 P17,000
AA1 -Chapter 3 – Partnership Liquidation
Suggested Answers page

Problem 3 – 9 Requirement No 2
Wilson, Yuson and Zapata
Cash Distribution Schedule
July 1 - September 30, 2008

Liabilities Wilson Yuson Zapata


Capital balances before liquidation P 17,000 P 55,000 P 45,000 P 24,000
July - Dist. of loss on sale of assets (1) (4,750) (2,850) (1,900)
Liquidation expenses (500) (300) (200)
Payment of liabilities (17,000)
Payment to partners (2) (6,500)
Balances - P 49,750 P 35,350 P 21,900
Aug.- Liquidation expenses (750) (450) (300)
Equipment taken by Zapata (10,000)
Gain on transfer of eqt. to Zapata (3,000) (1,800) (1,200)
Payment to Yuson (3) (4,000)
Balances - P 52,000 P 32,700 P 12,800
Sept- Dist. of loss on sale of assets (4) (10,000) (6,000) (4,000)
Liquidation expenses (500) (300) (200)
Final distribution to partners - P41,500 P26,400 P8,600

(1) (P22,000 + P14,000) - (P16,500 + P10,000) = P9,500 loss on sale of assets


(2) (P6,000 + P26,500 - P1,000 - P17,000) - P8,000 cash withheld = P6,500 cash dist. Req. 1
(3) schedule of cash distribution below8,000 – 1,500 – 2,500 +4,000
(4) (P99,000 - P4,000 BV of equipment taken by Zapata) - P75,000 = P20,000 loss on sale

Schedule of Cash Distribution


August 31, 20068
Wilson Yuson Zapata Wilson Yuson Zapata
Capital balances after dist.
of equipment to Zapata P 52,000 P 36,700 P 12,800
Profit and loss ratio 50% 30% 20%
Loss absorption balance P104,000 P122,333 P 64,000
Allocation I - Cash to
Yuson to reduce LAB to
amt. reported for Wilson 18,333 P 5,500
Balances P104,000 P104,000 P 64,000
Allocation II - Cash to Wilson
& Yuson to reduce LAB to
amount reported for Zapata 40,000 40,000 P20,000 12,000
Balances P 64,000 P 64,000 P 64,000 P20,000 P 17,500
Allocation III – P & L ratio
AA1 -Chapter 3 – Partnership Liquidation
Suggested Answers page

Problem 3 - 9 - Requirement 3
Amount Wilson Yuson Zapata
Cash available in September P76,500
Allocation I – Balance 1,500 P 1,500
Allocation II 32,000 P 20,000 12,000
Balance - Allocation III P43,000 21,500 12,900 P 8,600
P 41.500 P 26,400 P 8,600
Problem 3 -10
Arceo, Basco and Cervo
Statement of Changes in Partners’ Capital
For the Period January 1, 2006 to May 31, 2008

Arceo Basco Cervo Total


2006:
Original investment P50,000 P30,000 P 80,000
Distribution of net income (sch. 1) 15,200 12,800 28,000
Drawings (7,000) (6,000) (13,000)
Balance, December 31 P58,200 P36,800 P 95,000
2007:
Investment of Cervo (sch. 2) (9,100) (4,900) P54,000 40,000
Distribution of net loss 4,200 3,000 4,800 12,000
Drawings (4,900) (3,900) (4,200) (13,000)
Balances, December 31 P40,000 P25,000 P45,000 P110,000
2008:
Distribution of cash in Feb. (sch. 3) (5,000) (5,000) (10,000)
Distribution of cash in April (sch. 4) (7,000) (5,000) (8,000) (20,000)
Balances P28,000 P20,000 P32,000 P 80,000
Sale of assets & distribution of loss
in May (sch. 5) (17,500) (12,500) (20,000) (50,000)
Final cash distribution P 10,500 P 7,500 P 12,000 P 30,000

Schedule 1 - Distribution of 2006 net income


Arceo Basco Total
Salaries P10,000 P10,000 P20,000
Remainder – 65%:35% 5,200 2,800 8,000
Total P15,200 P12,800 P28,000

Schedule 2 - Admission of Cervo


Total capital before admission of Cervo P 95,000
Investment of Cervo 40,000
Total capital P135,000
Interest acquired by Cervo 40%
Capital credit of Cervo P 54,000
Investment of Cervo 40,000
Bonus to Cervo from old partners (shared 65%:35%) P 14,000
AA1 -Chapter 3 – Partnership Liquidation
Suggested Answers page

Schedule 3 - Cash distribution in February

Arceo Basco Cervo


Capital balances before dist. of cash P40,000 P25,000 P45,000
Restricted interest - possible loss if nothing
is realized on remaining assets (P100,000) 35,000 25,000 40,000
Free interest - amount to be paid to partners P 5,000 P ------ P 5,000

Schedule 4 - Cash distribution in April

Arceo Basco Cervo


Capital balances before dist. of cash P 35,000 P25,000 P40,000
Restricted interest - possible loss if nothing
is realized on remaining assets (P80,000) 28,000 20,000 32,000
Free interest - amount to be paid to partners P 7,000 P 5,000 P 8,000

Schedule 5 - Loss on realization of assets in May

Capital balances equal to net assets P80,000


Cash realized on sale of assets 30,000
Loss on realization P50,000

Problem 3-11
Partnership Books
1. Inventories 60,000
Prepaid Expenses 3,000
Goodwill 243,000
Accrued Expenses 6,000
Leony, Capital 200,000
Espie, Capital 100,000

2. Rover Corp. Stocks 4,500,000


Accounts Payable 600,000
Accrued Expenses 6,000
Allowance for Uncollectible Accounts 120,000
Cash 450,000
Accounts Receivable 660,000
Inventories 1,350,000
Prepaid Expenses 3,000
Furniture and Equipment 2,520,000
Goodwill 243,000

3. Leony, Capital 2,600,000


Espie, Capital 1,900,000
Rover Corp. Stocks 4,500,000
AA1 -Chapter 3 – Partnership Liquidation
Suggested Answers page

Corporation’s Books
1. Cash 450,000
Accounts Receivable 660,000
Inventories 1,350,000
Prepaid Expense 3,000
Furniture and Equipment 2,520,000
Goodwill 243,000
Allowance for Uncollectible Accounts 120,000
Accounts Payable 600,000
Accrued Expenses 6,000
Ordinary Share Capital 4,500,000

2. Land 3,600,000
Cash 1,500,000
Pre-Operating Expenses 450,000
Ordinary Share Capital 4,800,000
PIC in Excess of Par 750,000

Rover Corporation
Statement of Financial Position
July 1, 2008

Assets Liabilities and Shareholders’ Equity


Cash P 1,950,000 Accounts Payable P 600,000
Accounts Receivable (net of Allow Accrued Expenses 6,000
of P120,000) 540,000 Total Liabilities P 606,000
Inventories 1,350,000 Shareholders’ Equity
Prepaid Expenses 3,000 Ordinary Share Capital P9,300,000
Land 3,600,000 PIC in Excess of Par 750,000
Furniture and Equipment 2,520,000 Retained Earnings (deficit) (450,000)
Goodwill 243,000 Total Shareholders’ Equity P9,600,000
Total Assets P10,206,000 Total liabilities and SH equity P10,206,000

MULTIPLE CHOICE

1. D
2. D
3. C
4. C Share on loss on realization
(P39,000 + P4,800 – P33,000) P10,800
Percentage ownership of Imperial ÷ 20%
Total loss on realization P54,000

5. A Total capital P70,000


Cash available 28,000
Loss on realization P42,000
AA1 -Chapter 3 – Partnership Liquidation
Suggested Answers page

6. B Gueco Tiangco Bacelon


Capital bal. before liquidation P 40,000 P 25,000 P 5,000
Loss on realization ( 21,000) (14,000) ( 7,000)
Balances P 19,000 P 11,000 P( 2,000)
Add’l loss to Gueco & Tiangco
for the deficiency of Barcelon ( 1,200) ( 800) 2,000
Cash distribution to partners P 17,800 P 10,200 P ---0---

7. D Total capital (P360,000 + P72,000) P432,000


Total liabilities 84,000
Total loss on liquidation P516,000

8. A Alarcon Baretto Coronel


Capital balances P 100,000 P 80,000 P 300,000
Drawing ( 60,000) ( 40,000) (20,000)
Distribution of net income 24,000 24,000 24,000
Loss on liquidation (172,000) (172,000) (172,000)
Balances P(108,000 P(108,000) P 132,000
Additional loss to partners 108,000 ( 54,000) ( 54,000)
Cash to be distributed to P 78,000
Coronel

9. C Total capital P40,000


Loans from partners 7,500
Total partners’ interest P47,500
Cash available to partners (P37,500 – P28,500) 9,000
Total loss on realization P38,500

10 C Doria Elma
Capital balances before liquidation P 24,500 P 15,500
Loan balances 4,000 3,500
Total partners’ interest P 28,500 P 19,000
Loss on realization ( 23,100) ( 15,400)
Balances – cash to be paid to partners P 5,400 P 3,600

11 C Total assets = Total capital + Total liabilities


= P60,000 + P 3,000 P 63,000
Less Cash = P3,000 + P22,200 – P23,200 ___2,000
Book value of noncash assets P 61,000

12 C P61,000 – P23,200 = P37,800 x 3/21 P 5,400


AA1 -Chapter 3 – Partnership Liquidation
Suggested Answers page

13 B Jurado Katindig Lazaro Marcelo


Capital balances P 1,000 P25,000 P25,000 P 9,000
Loss on realization ( 5,400) ( 7,200) ( 10,800) ( 14,400)
P( 4,400) P(17,800) P14,200 P( 5,400)
Additional loss 4,400 3,920 ( 5,880) 5,400
Payment to Lazaro P 8,320

14 C Total credits equal debits (P130,000 + P44,000 +


P90,000) P264,000
Less Cash 40,000
Book value of other assets P224,000
Loss on realization [(P50,000 + P17,600 – P55,200)/40%] 31,000
Cash received from sale of other assets P193,000

15 A
16 A
17 B Esper Ester Ethel Elmer
Capital balances P 50,000 P50,000 P50,000 P 75,000
Loss on realization (112,000) ( 56,000) ( 56,000) ( 56,000)
P(62,000) P(6,000) P(6,000) P19,000
Additional loss (3,000) 6,000 ( 1,500) (1,500)
Amt to be rec.from the part. P 17,500
200,000
P217,500

18 D Urbe Viray
Initial investment P 137,500,000 P 137,500,000
Purchases ( 1,237,500,000) ( 495,000,000)
Sales 1,339,250,000 462,000,000
Interest ( 2,200,000) ( 1.375,000)
Dividends 1,100,000 2,750,000
Cash held P 238,150,000 P 105,875,000
Equal share 172,012,500 172,012,500
Cash received (paid) (P 66,137,500) P 66,137,500

19 C Delia Erma Flora


Capital balances before liquidation P480,000 P135,000 P165,000
Loss on liquidation (P180,000) ( 72,000) ( 90,000) ( 18,000)
Cash to be received by Delia P408,000

20 A Delia Erma Flora


Capital balances before liquidation P480,000 P135,000 P165,000
Loss on liquidation (P540,000) ( 216,000) ( 270,000) ( 54,000)
P264,000 (P135,000) P111,000
Add’l loss to Delia & Flora ( 108,000) 135,000 ( 27,000)
Cash to be received by Flora P 84,000
AA1 -Chapter 3 – Partnership Liquidation
Suggested Answers page

21 D Delia Erma Flora


Capital balances before liquidation P480,000 P135,000 P165,000
Loss on liquidation (P180,000) ( 72,000) ( 90,000) ( 18,000)
Balances P408,000 P 45,000 P147,000
Possible loss if remaining
inventories are not sold (192,000) ( 240,000) ( 48,000)
Balances P216,000 (P 195,000) P 99,000
Add’ loss to Delia & Flora ( 156,000) 195,000 ( 39,000)
Distribution of cash to partners P 60,000 - P 60,000
22 D
23 B
24 A Estrada Fortuna Gener
Balances before liquidation P 40,000 P 65,000 P 48,000
Loss on sale of assets - P40,000 ( 16,000) (16,000) ( 8,000)
Possible loss if nothing is realized
on remaining assets - P90,000 ( 36,000) (36,000) (18,000)
Balances P( 12,000) P 13,000 P 22,000
Add’l possible loss to Fortuna and
Gener for deficiency of Estrada 12,000 ( 8,000) ( 4,000)
Balances - cash to be distributed P --------- P 5,000 P 18,000
25 B Capital balance of Gener before distribution of cash P 18,000
Share in the cash to be withheld for possible liquidation
expenses - P3,000 x 20%/60% (shared by Fortuna
& Gener) ( 1,000)
Cash to be received by Gener P 17,000
26 D The remaining cash will be distributed according to profit and loss ratio.
Thus the P14,000 will be distributed as follows:
Estrada - P14,000 x 40% = P5,600
Fortuna - P14,000 x 40% = P5,600
Gener - P14,000 x 20% = P2,800
27 C Total capital before drawing and net loss P 135,000
Drawing ( 10,000)
Net loss for the year ( 20,000)
Total liabilities 5,000
Total assets P 110,000
Cash on hand ( 700)
Amount of noncash assets before liquidation P 109,300
28 C Capital balance of Aguila before dist. of net loss P 25,000
Share in net loss (P20,000 x 60%) ( 12,000)
Capital balance of Aguila before liquidation P 13,000
Cash to be received by Aguila 19,000
Share of Aguila in the gain on sale of other assets P 6,000
Percentage share of Aguila ÷ 60%
Total gain on sale of other assets P 10,000
Book value of other assets 109,300
Cash to be realized from sale of other assets P 119,300
AA1 -Chapter 3 – Partnership Liquidation
Suggested Answers page

PAYMENTS
29 D Aguila Balweg Corpuz Aguila Balweg Corpuz
Capital balances P 25,000 P 50,000 P 60,000
Drawing (10,000)
Net loss ( 12,000) ( 5,000) ( 3,000)
Total partners’ interest P 13,000 P 45,000 P 47,000
Profit and loss ratio ÷ 60% ÷ 25% ÷ 15%
Loss absorption bal. P 21,667 P180,000 P313,333
Alloc. I - Cash to Corpuz (133,333) P 20,000
Balances P 21,667 P180,000 P180,000
Alloc. II -Cash to Balweg
and Corpuz (158,333) (158,333) P 39,583 23,750
Balances P 21,667 P 21,667 P 21,667 P 39,583 P43,750
Alloc. III - Based on
P & L ratio
Cash received by Corpuz P 33,000
Cash received from Allocation I ( 20,000)
Cash received from Allocation Ii P 13,000
Fractional share (B – 25% and C -15%) ÷ 15/40
Total cash distributed P 34,667
Fractional share of Balingit x 25/40
Cash received by Balingit P 21,667
30 B Vulnerability
Ranking
Nera - P450,000 / 30% = P150,000 3
Ochoa - P250,000 / 50% = P 50,000 1
Perez - P250,000 / 20% = P125,000 2

31 D Priority Nera Ochoa Perez Perez


Creditors Capital Capital Loan Capital
1st P500,000 100%
next P75,000 100%
next P375,000 60% 26.67% 13.33%
Remainder 30% 50% 20.00%

32 A Nera Ochoa Perez Total


Equities P450,000 P250,000 P250,000 P950,000
Loss to absorb Ochoa ( 150,000) ( 250,000) ( 100,000) ( 500,000)
Balances P300,000 ------ P150,000 P450,000
Loss to absorb Perez ( 225,000) (150,000) ( 375,000)
Balance P 75,000 ------ ---- P 75,000
33 C Reyes (20%) Santos (40%) Torres (40%)
Net capital balances P100,000 P440,000 P310,000
Possible loss of P700,000 ( 140,000) (280,000) ( 280,000)
Balances (P 40,000) P160,000 P 30,000
Possible loss from Reyes debit balance 40,000 ( 20,000) ( 20,000)`
Cash distribution ------ P140,000 P 10,000
AA1 -Chapter 3 – Partnership Liquidation
Suggested Answers page

34 D Roger Sergio Tito Roger Sergio Tito


35 C Capital balances P108,000 P120,000 P129,000
Add Loan 30,000
Total partners’ interest P108,000 P150,000 P129,000
Divided by P & L ratio 30% 50% 20%
Loss absorption capacity P360,000 P300,000 P645,000
Allocation 1 ( 285,000) P57,000
Balances P360,000 P300,000 P360,000
Allocation II ( 60,000) ( 60,000) P18,000 12,000
P300,000 P300,000 P300,000 P18,000 - P69,000
Allocation III – P & L ratio

36 B Roger Sergio Tito


Amount available P72,000
Allocation 1 to Tito 57,000 P57,000
Allocation II – 30%, 20% P15,000 P9,000 6,000
P9,000 P63,000

37 A Roger Sergio Tito


Amount available P120,000
Allocation II – Balance 15,000 P 9,000 P 6,000
Allocation III P105,000 31,500 P52,500 21,000
P40,500 P52,500 P27,000

38 D Jacinto Mapa Magno


Capital balances P400,000 P600,000 P1,000,000
Revaluation of assets 200,000 200,000 200,000
Adjusted capital P600,000 P800,000 P1,200,000
Par of capital stock ÷ P100 ÷ P100 ÷ P100
Shares of stock to be rec’d by partners 6,000 sh. 8,000 sh 12,000 sh

39 C Capital balances P260,000


Adjustment in assets (P20,000 – P10,000 – P3,000) 7,000
Adjusted capital P267,000

40 B Total capital (P94,800 + P214,200) P309,000


Adjustments in assets (P6,600 – P20,000 – P22,000) ( 35,400)
Adjusted capital P273,600
Ordinary Share Capital (720 x 2 x P10) 14,400
Preference Share Capital P259,200

Ordinary shares (P14,400 / P10) 1,440 sh..


Preference shares (P259,200 / P100) 2,592 sh.
AA1 -Chapter 3 – Partnership Liquidation
Suggested Answers page

41 C Roldan Moises
Capital balances before incorporation P94,800 P214,200
Adjustment in assets ( 11,800) ( 23,600)
Adjusted capital P83,000 P190,600
Ordinary Share Capital (720 @P10) 7,200 7,200
Preference Share Capital P75,800 P183,400

Preference shares 758 1,834


Ordinary shares 720 720
AA1 -Chapter 3 (2008 edition))s page 1

Exercise 3 – 7

Other Velasco CAPITAL


Cash Assets Liabilities Loan Viola Velasco Vicente
Balances before liquidation P 80,000 P960,000 P630,000 P 50,000 P470,000 (P140,000) P30,000
Sale of other assets & distribution of loss 200,000 ( 300,000) ( 60,000) ( 20,000) ( 20,000)
Balances P280,000 P660,000 P630,000 P 50,000 P410,000 (P160,000) P10,000
Payment of liabilities ( 280,000) ( 280,000)
Balances --- P660,000 P350,000 P 50,000 P410,000 (P160,000) P10,000
Offset of loan ( 50,000) 50,000
Additional investment by Velasco 90,000 90,000
Balances P 90,000 P660,000 P350,000 ---- P410,000 (P 20,000) P10,000
Asset distributed to Viola ( 200,000) ( 212,000) 6,000 6,000
Balances P 90,000 P460,000 P350,000 ----- P198,000 (P 14,000) P16,000
July sale of assets and distribution of loss 540,000 ( 400,000) 84,000 28,000 28,000
Balances P630,000 P 60,000 P350,000 ----- P282,000 P 14,000 P44,000
Payment of liabilities (350,000) ( 350,000)
Balances P280,000 P 60,000 ------ ---- P282,000 P 14,000 P44,000
Possible loss on remaining other assets ( 60,000) ( 36,000) ( 12,000) ( 12,000)
Cash distribution to partners P280,000 ----- ----- ---- P246,000 P 2,000 P32,000
Chapter 3 – Partnership Liquidation
Suggested Answers page

PROBLEMS
Problem 3-1

1. Elma, Erica and Edna


Statement of Liquidation
January 1 – 31, 2008
Non-cash C A P I T A L
Cash Assets Liabilities Elma Erica Edna
Profit and loss ratio 3/8 3/8 2/8
Balances before liquidation P 80,000 P810,000 P270,000 P60,000 P290,000 P270,000
Sale of non-cash assets and distribution of loss 634,000 (810,000) (66,000) ( 66,000) ( 44,000)
Payment of liquidation expenses ( 24,000) ( 9,000) ( 9,000) ( 6,000)
Balances P690,000 P270,000 (P15,000) P215,000 P220,000
Payment of liabilities (270,000) (270,000)
Balances P420,000 (P15,000) P215,000 P220,000
Additional investment of Elma 15,000 15,000
Balances P435,000 P215,000 P220,000
Payment to partners ( 435,000) ( 215,000) ( 220,000)
Chapter 3 – Partnership Liquidation
Suggested Answers page

2. Elma , Erica and Edna


Statement of Liquidation
January 1 – 31, 2008
Non-cash Note Payable C A P I T A L
Cash Assets to Elma Liabilities Elma Erica Edna
Profit and loss ratio 3/8 3/8 2/8
Balances before liquidation P 80,000 P810,000 P70,000 P200,000 P60,000 P290,000 P270,000
Sale of non-cash assets and distribution of loss 634,000 (810,000) (66,000) ( 66,000) ( 44,000)
Payment of liquidation expenses (24,000) ( 9,000) ( 9,000) ( 6,000)
Balances P690,000 P70,000 P200,000 (P15,000) P215,000 P220,000
Payment of liabilities (200,000) (200,000)
Balances P490,000 P70,000 (P15,000) P215,000 P220,000
Offset of loan against debit balance in the
capital balance of Elma ( 15,000) 15,000
Balances P490,000 P55,000 P215,000 P220,000
Payment to partners (490,000) ( 55,000) ( 215,000) ( 220,000)
3. Elma , Erica and Edna
Statement of Liquidation
January 1 – 31, 2008
NR from Non-cash NP C A P I T A L
Cash Erica Assets to Elma Liabilities Elma Erica Edna
Profit and loss ratio 3/8 3/8 2/8
Balances before liquidation P 80,000 P110,000 P700,000 P70,000 P200,000 P60,000 P290,000 P270,000
Sale of non-cash assets and distribution of 634,000 (700,000) (24,750) ( 24,750) ( 16,500)
loss
Payment of liquidation expenses (24,000) ( 9,000) ( 9,000) ( 6,000)
Balances P690,000 P110,000 P70,000 P200,000 P26,250 P256,250 P247,500
Payment of liabilities (200,000) (200,000)
Balances P490,000 P110,000 P70,000 P26,250 P256,250 P247,500
Offset of receivable against credit balance in
the capital of Erica (110,000) ( 110,000)
Balances P490,000 P70,000 P26,250 P146,250 P247,500
Payment to partners (490,000) ( 70,000) ( 26,250) ( 146,250) ( 247,500)
Chapter 3 – Partnership Liquidation
Suggested Answers page

Problem 3 – 2 (Case 1)
Calma, Daza and Esteban
Statement of Liquidation
January, 2009
Other L O A N C A P I T A L
Cash Assets Liabilities Daza Esteban Calma Daza Esteban
(2/5) (2/5) (1/5)
Balances before liquidation P 20,000 P 340,000 P 112,000 P 5,000 P 8,000 P 95,000 P 60,000 P 80,000
Sale of assets & distribution of loss 250,000 (340,000) (36,000) (36,000) (18,000)
Balances P 270,000 - P 112,000 P 5,000 P 8,000 P 59,000 P 24,000 P 62,000
Payment of liabilities ( 112,000) (112,000)
Payment of to partners P 158,000 - - P 5,000 P 8,000 P 59,000 P 24,000 P 62,000

Problem 3 – 2 (Case 2)
Calma, Daza and Esteban
Statement of Liquidation
January, 2009
Other L O A N C A P I T A L
Cash Assets Liabilities Daza Esteban Calma Daza Esteban
(2/5) (2/5) (1/5)
Balances before liquidation P 20,000 P 340,000 P 112,000 P 5,000 P 8,000 P 95,000 P 60,000 P 80,000
Sale of assets & distribution of loss 185,000 (340,000) (62,000) (62,000) (31,000)
Balances P 205,000 - P 112,000 P 5,000 P 8,000 P 33,000 P( 2,000) P 49,000
Payment of liabilities ( 112,000) (112,000)
Balances P 93,000 - - P 5,000 P 8,000 P 33,000 P( 2,000) P 49,000
Offset of loan against debit balance in the
capital account (2,000) 2,000
Payment to partners P 93,000 - - P 3,000 P 8,000 P 33,000 - P 49,000
Chapter 3 – Partnership Liquidation
Suggested Answers page

Problem 3 – 2 (Case 3)

Calma, Daza and Esteban


Statement of Liquidation
January, 2009
Other L O A N C A P I T A L
Cash Assets Liabilities Daza Esteban Calma Daza Esteban
(2/5) (2/5) (1/5)
Balances before liquidation P 20,000 P 340,000 P 112,000 P 5,000 P 8,000 P 95,000 P 60,000 P 80,000
Sale of assets & distribution of loss 170,000 (340,000) (68,000) (68,000) (34,000)
Balances P 190,000 - P 112,000 P 5,000 P 8,000 P 27,000 P( 8,000) P 46,000
Payment of liabilities ( 112,000) (112,000)
Balances P 78,000 - - P 5,000 P 8,000 P 27,000 P( 8,000) P 46,000
Offset of loan against debit balance in the
capital account (5,000) 5,000
Balances P 78,000 - - - P 8,000 P 27,000 P(3,000) P 46,000
Payment to partners ( 78,000) ( 8,000) ( 25,000) (45,000)
Balances - - - - - P 2,000 P(3,000) P 1,000
Additional investment by Daza 3,000 3,000
Payment to partners P 3,000 - - - - P 2,000 - P 1,000
Chapter 3 – Partnership Liquidation
Suggested Answers page

Problem 3- 2 (Case 4)
Calma, Daza and Esteban
Statement of Liquidation
January, 2009

Other L O A N C A P I T A L
Cash Assets Liabilities Daza Esteban Calma Daza Esteban
(2/5) (2/5) (1/5)
Balances before liquidation P 20,000 P 340,000 P 112,000 P 5,000 P 8,000 P 95,000 P 60,000 P 80,000
Sale of assets & distribution of loss 125,000 (340,000) (86,000) (86,000) (43,000)
Balances P 145,000 - P 112,000 P 5,000 P 8,000 P 9,000 P(26,000) P 37,000
Payment of liabilities ( 112,000) (112,000)
Balances P 33,000 - - P 5,000 P 8,000 P 9,000 P(26,000) P 37,000
Offset of loan against debit balance in the
capital account (5,000) 5,000
Balances P 33,000 - - - P 8,000 P 9,000 P(21,000) P 37,000
Payment to partners ( 33,000) ( 8,000) (25,000)
Balances - - - - - P 9,000 P(21,000) P 12,000
Additional investment by Daza 21,000 21,000
Payment to partners P 21,000 - - - - P 9,000 - P 12,000
Chapter 3 – Partnership Liquidation
Suggested Answers page

Problem 3 – 2 (Case 5)

Calma, Daza and Esteban


Statement of Liquidation
January, 2009
Other L O A N C A P I T A L
Cash Assets Liabilities Daza Esteban Calma Daza Esteban
(2/5) (2/5) (1/5)
Balances before liquidation P 20,000 P 340,000 P 112,000 P 5,000 P 8,000 P 95,000 P 60,000 P 80,000
Sale of assets & distribution of loss 90,000 (340,000) (100,000) (100,000) (50,000)
Balances P 110,000 - P 112,000 P 5,000 P 8,000 P( 5,000) P(40,000) P 30,000
Payment of liabilities ( 110,000) (110,000)
Balances - - P 2,000 P 5,000 P 8,000 P( 5,000) P(40,000) P 30,000
Offset of loan against debit balance in the
capital account (5,000) 5,000
Balances - - P 2,000 - P 8,000 P( 5,000) P(35,000) P 30,000
Additional investment by Calma and Daza 40,000 5,000 35,000
Payment of liabilities, loan and capital P 40,000 - P 2,000 - P 8,000 - - P 30,000
Chapter 3 – Partnership Liquidation
Suggested Answers page

Problem 3-4

1. Estrella, Capital P 144,000 3. a. Cash 463,000


Estrella, Drawing ( 12,000) Eugenio, Capital 52,500
Total interest of Estrella P 132,000 Esteban, Capital 31,500
Cash received by Estrella 111,000 Estrella, Capital 21,000
Share of Estrella in the loss on P 21,000 Other Assets 568,000
liquidation
Fractional share of Estrella  2/10
Total loss on liquidation P 105,000 b. Liabilities 200,000
Cash 200,000

c. Esteban, Loan 40,000


Eugenio, Capital 79,500
Esteban, Capital 102,500
Estrella, Capital 111,000
Cash 333,000

2.
Eugenio , Esteban and Estrella
Statement of Liquidation
January 1 – 31, 2008

CAPITAL
Other Esteban, Eugenio Esteban Estrella
Cash Assets Liabilities Loan (5/10) (3/10) (2/10)
Balances before liquidation P 70,000 P 568,000 P 200,000 P 40,000 P 132,000 P 134,000 P132,000
Sale of other assets & distribution of loss 463,000 ( 568,000) ( 52,500) ( 31,500) ( 21,000)
Balances P 533,000 P 200,000 P 40,000 P 79,500 P 102,500 P 111,000
Payment of liabilities (200,000) ( 200,000)
Balances P 333,000 P 40,000 P 79,500 P 102,500 P 111,000
Payment to partners ( 333,000) ( 40,000) ( 79,500) (102,500) ( 111,000)
Chapter 3 – Partnership Liquidation
Suggested Answers page

Problem 3 - 6

QRS Partnership
Statement of Liquidation
July to September, 2008

Accounts Roman, C A P I T A L
Cash Other Assets Payable Loan Quizon Roman Silva
Balances before liquidation P 150,000 P2,010,000 P1,215,000 P150,000 P300,000 P270,000 P225,000
July: Sale of assets 1,170,000 ( 1,530,000) ( 180,000) ( 108,000) ( 72,000)
Payment of liabilities ( 1,215,000) ( 1,215,000)
Payment of liquidation expenses ( 7,500) ( 3,750) ( 2,250) ( 1,500)
P 97,500 P 480,000 - P150,000 P116,250 P159,750 P151,500
Payment of loan ( 67,500) ( 67,500)
Balances P 30,000 P 480,000 - P 82,500 P116,250 P159,750 P151,500
August: Sale of assets 66,000 ( 105,000) ( 19,500) ( 11,700) ( 7,800)
Payment of liquidation expenses ( 7,500) ( 3,750) ( 2,250) ( 1,500)
P 88,500 P 375,000 - P 82,500 P 93,000 P145,800 P142,200
Payment of loan and capital ( 58,500) ( 41,100) ( 17,400)
Balances P 30,000 P 375,000 - P 41,400 P 93,000 P145,800 P124,800
Sept.: Sale of assets 165,000 ( 375,000) ( 105,000) ( 63,000) ( 42,000)
Payment of liquidation expenses ( 7,500) ( 3,750) ( 2,250) ( 1,500)
P 187,500 - - P 41,400 (P 15,750) P 80,550 P 81,300
Additional loss to Roman & Silva 15,750 ( 9,450) ( 6,300)
Payment to partners P 187,500 - - P 41,400 - P 71,100 P 75,000
Chapter 3 – Partnership Liquidation
Suggested Answers page

Problem 3 - 5
JKLM Trading
Statement of Liquidation
February 1 - March 31, 2008

Other Jocson C A P I T A L
Cash Assets Liabilities Loan Jocson Kaimo Legarda Manabat
Balances before liquidation P 100,320 P 193,530 P 21,360 P 15,000 P 24,120 P 96,480 P 109,020 P 27,870
February:
Sale of assets & distribution of loss 49,320 ( 66,060) ( 3,348) ( 5,022) ( 5,022) ( 3,348)
Payment of liabilities ( 17,750) ( 17,750)
Payment of liquidation expenses ( 8,220) ( 1,644) ( 2,466) ( 2,466) ( 1,644)
Balances P 123,670 P 127,470 P 3,610 P 15,000 P 19,128 P 88,992 P 101,532 P 22,878
Payment to partners (sch. 1) ( 120,060) ( 7,980) (49,770) ( 62,310)
Balances P 3,610 P 127,470 P 3,610 P 7,020 P 19,128 P 39,222 P 39,222 P 22,878
March:
Sale of assets & distribution of gain 48,330 ( 44,850) 696 1,044 1,044 696
Payment of liabilities ( 3,610) ( 3,610)
Payment of liquidation expenses ( 7,380) ( 1,476) ( 2,214) ( 2,214) ( 1,476)
Balances P 40,950 P 82,620 p -------- P 7,020 P 18,348 P 38,052 P 38,052 P 22,098
Payment to partners (sch. 2) ( 40,950) 7,020 ( 1,824) (13,266) ( 13,266) ( 5,574)
Balances, March 31 P --------- P 82,620 P -------- P ------- P 16,524 P 24,786 P 24,786 P 16,524
Chapter 3 – Partnership Liquidation
Suggested Answers page

Problem 3 - 8

Req. 2
Neri, Ordan and Pacia
Statement of Liquidation
January 1 - March 31, 2008

Other Pacia, Sal. Pay. C A P I T A L


Cash Assets Liabilities Loan to Neri Neri Ordan Pacia
Balances before liquidation P 19,000 P 191,000 P 77,000 P 9,000 P 6,000 P 50,000 P 28,000 P 40,000
January:
Collections from customers 112,000 (112,000)
Payment of liquidation expenses ( 4,400) ( 2,200) ( 1,100) ( 1,100)
Payment of liabilities ( 38,000) ( 38,000)
Balances P 88,600 P 79,000 P 39,000 P 9,000 P 6,000 P 47,800 P 26,900 P 38,900
Payment to partners (see schedule) ( 16,000) ( 9,000) ( 7,000)
Balances P 72,600 P 79,000 P 39,000 - P 6,000 P 47,800 P 26,900 P 31,900
February:
Collections from customers 36,000 ( 36,000)
Payment of liquidation expenses ( 2,800) ( 1,400) ( 700) ( 700)
Payment of liabilities & dist. of gain (38,000) ( 39,000) 500 250 250
Balances P 67,800 P 43,000 - - P 6,000 P 46,900 P 26,450 P 31,450
Payment to partners (see schedule) ( 19,800) ( 6,000) ( 1,400) ( 3,700) ( 8,700)
Balances P 48,000 P 43,000 - - - P 45,500 P 22,750 P 22,750
March:
Collections from customers & dist. of 35,000 ( 43,000) ( 4,000) ( 2,000) ( 2,000)
loss
Payment of liquidation expenses ( 4,000) ( 2,000) ( 1,000) ( 1,000)
Payment to partners P( 79,000) - - - - P 39,500 P 19,750 P 19,750
Chapter 3 – Partnership Liquidation
Suggested Answers page

Problem 3 -8
Requirement No 1 Neri, Ordan and Pacia
Cash Priority Program
January 1, 2008

Neri Ordan Pacia Neri Ordan Pacia


Capital balances before liquidation P 50,000 P 28,000 P 40,000
Add Loan balances 6,000 9,000
Total partners’ interest P 56,000 P 28,000 P 49,000
Profit and loss ratio ÷ 50% ÷ 25% ÷ 25%
Loss absorption balance P112,000 P112,000 P196,000
allocation I - Cash to Pacia reducing LAB to an amount reported
for Neri and Ordan. Reduction of P84,000 requires payment
of 25% of P 84,000 ( 84,000) _______ _______ P 21,000
Balances P112,000 P112,000 P112,000 -------- --------- P 21,000
Allocation II - Further cash distributions may be made in the profit and loss raio

SCHEDULE OF CASH DISTRIBUTION:


Amount Neri Ordan Pacia
January: Cash available P 16,000
Allocation I:
Payable to Pacia 16,000 - --------- P 16,000

February: Cash available P 19,800


Allocation I:
Payable to Pacia 5,000 P 5,000
Allocation II
Payable according
to P & L ratio P 14,800 P 7,400 P 3,700 3,700
- P 7,400 P 3,700 P 8,700
Chapter 3 – Partnership Liquidation
Suggested Answers page

Problem 3 - 7

Requirement 3
Tabora, Ureta and Veloso
Statement of Liquidation
January 1 to April 30, 2008

Other L O A N C A P I T A L
Cash Assets Tabora Ureta Veloso Tabora Ureta Veloso
Balances before liquidation - P 338,000 P 45,000 P 30,000 P 13,000 P 120,000 P 90,000 P 40,000
January:
Sale of assets 15,000 ( 15,000)
Payment to partners (15,000) (15,000)
Balances - P 323,000 P 45,000 P15,000 P 13,000 P 120,000 P 90,000 P 40,000
February:
Sale of assets 40,000 ( 40,000)
Payment to partners ( 40,000) ( 21,250) (15,000) ( 3,750)
Balances P 283,000 P 23,750 - P 13,000 P 120,000 P 86,250 P 40,000
March:
Sale of assets 90,000 ( 90,000)
Payment to partners ( 90,000) (23,750) (13,000) ( 23,500) ( 28,350) ( 1,400)
Balances - P 193,000 - - - P 96,500 P 57,900 P 38,600
April:
Sale of assets & distribution of loss 30,000 ( 193,000) ( 81,500) ( 48,900) (32,600)
Balances P 30,000 - - - - P 15,000 P 9,000 P 6,000
Payment to partners (30,000) - - - - P 15,000 P 9,000 P 6,000
Chapter 3 – Partnership Liquidation
Suggested Answers page
AA1 - Chapter 4 – Joint Ventures (2005)
Suggested Answers page 4

Exercise 4-2

Books of Alvin, Managing Partner

Feb. 12 Joint Venture 10,000


Cash 10,000

14 Joint Venture 2,000


Larry 2,000

15 Cash 9,000
Larry 7,500
Joint Venture 16,500

20 Cash 3,000
Joint Venture 3,000

20 Joint Venture 7,500


Income from Joint Venture 4,287.50
Larry 3,212.50
10% commission on net purchases to Alvin
25% commission on own sales

20 Cash 2,287.50
Larry 2,287.50

Books of Larry
Feb. 12 Joint Venture 10,000
Alvin 10,000

14 Joint Venture 2,000


Cash 2,000

15 Cash 7,500
Alvin 9,000
Joint Venture 16,500

20 Alvin 3,000
Joint Venture 3,000

20 Joint Venture 7,500


Alvin 4,287.50
Income from Joint Venture 3,212.50
10% commission on net purchases to Alvin
25% commission on own sales

20 Alvin 2,287.50
Cash 2,287.50
AA1 - Chapter 4 – Joint Ventures (2005)
Suggested Answers page 5

Problem 4-2

Requirement 1
Books of Roland, Managing Partner

1. Joint Venture 40,300,000


Greg 19,500,000
Medel 13,000,000
Land 7,800,000

2. Joint Venture 3,000,000


Cash 3,000,000
Improvements on land

3. Joint Venture Cash 35,400,000


Joint Venture 35,400,000
Sales by venturers.

4. Joint Venture Cash 14,300,000


Joint Venture 14,300,000
Sales by salesmen

5. Joint Venture 684,000


Joint Venture Cash 684,000
Venture expenses

6. Joint Venture 72,000


Income from Joint Venture 72,000
Salaries to Roland as managing partner

Joint Venture 3,540,000


Income from Joint Venture 490,000
Greg 1,280,000
Medel 1,770,000
10% commission on own sales

Joint Venture 2,104,000


Income from Joint Venture 701,334
Greg 701,333
Medel 701,333
Balance of profit divided equally

7. Greg 21,481,333
Medel 15,471,333
Joint Venture Cash 36,952,666
Final cash settlement
AA1 - Chapter 4 – Joint Ventures (2005)
Suggested Answers page 6

Books of Greg
1. Joint Venture 40,300,000
Land 19,500,000
Medel 13,000,000
Roland 7,800,000

2. Joint Venture 3,000,000


Roland 3,000,000
Improvements on land

3. Roland 35,400,000
Joint Venture 35,400,000
Sales by venturers.

4. Roland 14,300,000
Joint Venture 14,300,000
Sales by salesmen

5. Joint Venture 684,000


Roland 684,000
Venture expenses

6. Joint Venture 72,000


Roland 72,000
Salaries to Roland as managing partner

Joint Venture 3,540,000


Roland 490,000
Income from Joint Venture 1,280,000
Medel 1,770,000
10% commission on own sales

Joint Venture 2,104,000


Roland 701,334
Income from Joint Venture 701,333
Medel 701,333
Balance of profit divided equally

7. Cash 21,481,333
Medel 15,471,333
Roland 36,952,666
Final cash settlement
AA1 - Chapter 4 – Joint Ventures (2005)
Suggested Answers page 7

Books of Medel
1. Joint Venture 40,300,000
Greg 19,500,000
Land 13,000,000
Roland 7,800,000

2. Joint Venture 3,000,000


Roland 3,000,000
Improvements on land

3. Roland 35,400,000
Joint Venture 35,400,000
Sales by venturers.

4. Roland 14,300,000
Joint Venture 14,300,000
Sales by salesmen

5. Joint Venture 684,000


Roland 684,000
Venture expenses

6. Joint Venture 72,000


Roland 72,000
Salaries to Roland as managing partner

Joint Venture 3,540,000


Roland 490,000
Greg 1,280,000
Income from Joint Venture 1,770,000
10% commission on own sales

Joint Venture 2,104,000


Roland 701,334
Greg 701,333
Income from Joint Venture 701,333
Balance of profit divided equally

7. Greg 21,481,333
Cash 15,471,333
Roland 36,952,666
Final cash settlement
AA1 - Chapter 4 – Joint Ventures (2005)
Suggested Answers page 8

Problem 4-2

Requirement 2

Books of the Joint Venture


1. Land 40,300,000
Greg, Capital 19,500,000
Medel, Capital 13,000,000
Roland, Capital 7,800,000

2. Land 3,000,000
Roland, Capital 3,000,000

3. Cash 35,400,000
Sales 35,400,000

4. Cash 14,300,000
Sales 14,300,000

5. Expenses 684,000
Cash 684,000

6. Sales 49,700,000
Land 43,300,000
Expenses 684,000
Income Summary 5,716,000

7. Income Summary 72,000


Roland, Capital 72,000

Income Summary 3,540,000


Greg, Capital 1,280,000
Medel, Capital 1,770,000
Roland, Capital 490,000

Income Summary 2,104,000


Greg, Capital 701,333
Medel, Capital 701,333
Roland, Capital 701,334

8. Greg, Capital 21,481,333


Medel, Capital 15,471,333
Roland, Capital 12,063,334
Cash 49,016,000
AA1 - Chapter 4 – Joint Ventures (2005)
Suggested Answers page 9

Books of Greg
1. Investment in Joint Venture 19,500,000
Land 19,500,000

2. Investment in Joint Venture 1,981,333


Income from Joint Venture 1,981,333

3. Cash 21,481,333
Investment in Joint Venture 21,481,333

Books of Medel
1. Investment in Joint Venture 13,000,000
Land 13,000,000

2. Investment in Joint Venture 2,471,333


Income from Joint Venture 2,471,333

3. Cash 15,471,333
Investment in Joint Venture 15,471,333

Books of Roland
1. Investment in Joint Venture 7,800,000
Land 7,800,000

2. Investment in Joint Venture 3,000,000


Cash 3,000,000

3. Investment in Joint Venture 1,262,334


Income from Joint Venture 1,262,334

4. Cash 12,063,334
Investment in Joint Venture 12,063,334

Problem 4-3

Books of Marissa
1. Joint Venture 104,000
Yolly 44,000
Beth 60,000

2. Joint Venture Accounts Receivable 160,000


Joint Venture 160,000

3. Joint Venture Cash 153,000


Joint Venture 7,000
Joint Venture Accounts Receivable 160,000
AA1 - Chapter 4 – Joint Ventures (2005)
Suggested Answers page 10

4. Joint Venture 40,000


Joint Venture Cash 40,000

5. Yolly 10,000
Beth 7,500
Joint Venture 17,500

6. Joint Venture 1,560


Yolly 660
Beth 900
Interest on investment.

7. Joint Venture 8,000


Income from Joint Venture 8,000
Commission on sales.

8. Joint Venture 16,940


Yolly 5,646
Beth 5,647
Income from Joint Venture 5,647
Allocation of the balance.

9. Yolly 40,306
Beth 59,047
Cash 13,647
Joint Venture Cash 113,000

Books of Yolly
1. Joint Venture 104,000
Merchandise Inventory 44,000
Beth 60,000

2. Marissa 160,000
Joint Venture 160,000

3. Joint Venture 7,000


Marissa 7,000

4. Joint Venture 40,000


Marissa 40,000

5. Merchandise Inventory 10,000


Beth 7,500
Joint Venture 17,500

6. Joint Venture 1,560


Income from Joint Venture 660
Beth 900
Interest on investment.
AA1 - Chapter 4 – Joint Ventures (2005)
Suggested Answers page 11

7. Joint Venture 8,000


Marissa 8,000
Commission on sales.

8. Joint Venture 16,940


Income from Joint Venture 5,646
Beth 5,647
Marissa 5,647
Allocation of the balance.

9. Cash 40,306
Beth 59,047
Marissa 99,353

Books of Beth
1. Joint Venture 104,000
Yolly 44,000
Merchandise Inventory 60,000

2. Marissa 160,000
Joint Venture 160,000

3. Joint Venture 7,000


Marissa 7,000

4. Joint Venture 40,000


Marissa 40,000

5. Yolly 10,000
Merchandise Inventory 7,500
Joint Venture 17,500

6. Joint Venture 1,560


Yolly 660
Income from Joint Venture 900
Interest on investment.

7. Joint Venture 8,000


Marissa 8,000
Commission on sales.

8. Joint Venture 16,940


Yolly 5,646
Income from Joint Venture 5,647
Marissa 5,647
Allocation of the balance.

9. Yolly 40,306
Cash 59,047
Marissa 99,353
AA1 - Chapter 4 – Joint Ventures (2005)
Suggested Answers page 12

Requirement 2

Books of the Joint Venture


1. Merchandise 104,000
Yolly, Capital 44,000
Beth, Capital 60,000

2. Accounts Receivable 160,000


Sales 160,000

3. Cash 153,000
Uncollectible Accounts Expense 4,300
Sales Discount 2,700
Accounts Receivable 160,000

4. Expenses 40,000
Cash 40,000

5. Yolly, Capital 10,000


Beth, Capital 7,500
Merchandise 17,500

6. Sales 160,000
Merchandise 86,500
Sales Discounts 2,700
Doubtful Accounts Expense 4,300
Expenses 40,000
Income Summary 26,500

7. Income Summary 26,500


Yolly, Capital 6,306
Beth, Capital 6,547
Marissa, Capital 13,647

8. Yolly, Capital 40,306


Beth, Capital 59,047
Marissa, Capital 13,647
Cash 113,000

Books of Yolly
1. Investment in Joint Venture 44,000
Merchandise Inventory 44,000

2. Merchandise Inventory 10,000


Investment in Joint Venture 10,000

3. Investment in Joint Venture 6,306


Income from Joint Venture 6,306
P660 + P5,646 = P6,306
AA1 - Chapter 4 – Joint Ventures (2005)
Suggested Answers page 13

4. Cash 40,306
Investment in Joint Venture 40,306

Books of Beth
1. Investment in Joint Venture 60,000
Merchandise Inventory 60,000

2. Merchandise Inventory 7,500


Investment in Joint Venture 7,500

3. Investment in Joint Venture 6,547


Income from Joint Venture 6,547
P900 + P5,647 = P6,6,547

4. Cash 59,047
Investment in Joint Venture 59,047

Books of Marissa
1. Investment in Joint Venture 13,647
Income from Joint Venture 13,647
P8,000 + P5,647 = P13,647

2. Cash 13,647
Investment in Joint Venture 13,647

Problem 4-4

1. Merchandise Inventory 10,571.20


Joint Venture 10,571.20

2. Joint Venture 4,785


Income from Joint Venture 4,785
Bonus = 10% (NI – B)
Bonus = 10% (P53,636.20 – B) = P4,785

Joint Venture 47,851.20


Income from Joint Venture 14,355.36
Santi 23,925.60
Romy 9,570.24
Distribution of balance – 30%, 50%, and 20% to
Noel, Santi, and Romy, respectively.

3. Santi 22,863.60
Romy 18,628.24
Cash 41,491.84
Final cash settlement.
AA1 - Chapter 4 – Joint Ventures (2005)
Suggested Answers page 14

Problem 4-5

Books of Leo
1. Joint Venture 4,000
Income from Joint Venture 4,000
Bonus = 20% (NI – B)
Bonus = 20% (P24,000 – B) = P4,000

2. Income from Joint Venture 600


Mandy 300
Joint Venture 300
Interest on deficiency and excess
Leo = P10,000 x 12% x 6/12 = P600
Mandy = P5,000 x 12% x 6/12 = P300

3. Joint Venture 20,300


Income from Joint Venture 8,120
Niel 8,120
Mandy 4,060
Balance of profit divided in the ratio of 4:4:2 to
Leo, Niel, and Mandy, respectively

Books of Mandy
1. Joint Venture 4,000
Leo 4,000

2. Leo 600
Income from Joint Venture 300
Joint Venture 300

3. Joint Venture 20,300


Leo 8,120
Niel 8,120
Income from Joint Venture 4,060

Books of Niel
1. Joint Venture 4,000
Leo 4,000

2. Leo 600
Mandy 300
Joint Venture 300

3. Joint Venture 20,300


Leo 8,120
Income from Joint Venture 8,120
Mandy 4,060
AA1 - Chapter 4 – Joint Ventures (2005)
Suggested Answers page 15

MULTIPLE CHOICE

1. A Total credits in the Joint Venture account P258,100


Less Total debits in the Joint Venture account 197,500
Gain (excess of credit over debit) P 60,600

2. D Merchandise contribution P 85,000


Add Share in the gain (P60,600 x 2/10) 12,120
Final settlement to Minda P 97,120

3. A The account of Melissa has a debit balance, thus, she has to make payment..
The account of Nancy has a debit balance, thus, she has to make payment.
The account of Olivia has a credit balance, thus, she has to receive payment.

4. C P150,000 + P105,000 = P255,000

5. C P120,000 + (135,000/3) = P165,000

6. B Capital of Tan P270,000


Unsold merchandise taken by Tan ( 105,000)
Share on the venture income (P135,000* / 3) 45,000
Amount received by Tan in final settlement P210,000

* Credit balance in the Joint Venture account P150,000


Unsold merchandise taken by Tan 105,000
Venture income P255,000
Salaries to Reyes 120,000
Remainder – divided equally P135,000

7. B 15% (P115,000 –B) = P15,000

8. C. Credit balance in the Joint Venture account P 90,000


Unsold merchandise purchased by Soriente 25,000
Net profit before bonus P115,000
Bonus to Soriente [ 15% (P115,000 – B) 15,000
Net profit after bonus P100,000

9. C P100,000 x 40% = P40,000

10. B Santos Salazar


Account balances (P 5,000) P20,000
Share in venture profit 40,000 35,000
Cash settlement P35,000 P55,000
AA1 - Chapter 4 – Joint Ventures (2005)
Suggested Answers page 16

11. C Sales P240,000


Less Sales discounts 4,050
Net sales P235,950
Cost of sales:
Contributed merchandise P156,000
Less Returned merchandise 26,400 129,600
Gross profit P106,350
Operating expenses (P6,450 + P58,650) 65,100
Net income P 41,250
Less Bonus (P41,250 x 25/125) 8,250
Net income after bonus P 33,000

12. B P41,250 x 25/125 = P8,250

13. D Iona Paula


Merchandise contribution P66,000 P90,000
Merchandise returns ( 15,000) ( 11,400)
Interest on original capital 990 1,350
Balance of profit divided equally 10,220 10,220
Cash settlement P62,210 P90,170

14. A Joint Venture


Purchases 300,000 Sales 559,500
Expenses 34,500
Balance, end 225,000
559,500 559,500

Sales revenue is a credit entry in the Joint Venture account. The total of the
purchases, expenses and the ending balance is equal to total sales revenue. The
ending balance is the sum of the credit balances of Marc and Martin of P120,000
and P105,000.

15. B P236,500 x 50% = P118,250

16. A Investment of Marc P150,000


Cost of unsold goods assumed by Marc ( 4,500)
Share in the joint venture gain:
Credit balance in the JV account P225,000
Unsold goods assumed by the partners 11,500
JV gain P236,500
Share of Marc 50% 118,250
Cash settlement to Marc P263,750
17. B P12,000 – P2,500 = P9,500
18. D Debbie Ellie
Contribution P10,000 P2,000
Less Share on loss (P12,000 – P2,500) 4,750 4,750
P 5,250 (P2,750)
Additional loss to Debbie ( 2,750) 2,750
Cash distribution P 2,500
AA1 - Chapter 4 – Joint Ventures (2005)
Suggested Answers page 17

19. B Valdez Ramos Total


Receipts P789,200 P654,250
Less Investment 300,000 300,000
Revenue P489,200 P354,250 P 943,450
Sale of non-cash assets 600,000
Total revenue P1,543,450
Less Expenses – disbursements (P622,750 + P706,950) 1,329,750
Joint venture profit P 213,750

20.

21. B Debit to Joint Venture account:


Investment of Santos (12,000 shares @ P45) P540,000.00
Investment of Cruz (8,000 shares @ P45) 360,000.00
Manager’s fee [ 1% (176,000 + 240,000 + 133,000 +261,625)] 8,106.25
Miscellaneous expenses 1,500.00
P909,606.25
Credit to Joint Venture account:
Sales (4,000 @ P44) P176,000.00
Sales (6,000 @ P40) 240,000.00
Cash dividend [(12,000 + 8,000 – 4,000 – 6,000) x P2] 20,000.00
Sales (3,500 @ P38) 133,000.00
Sales [(10,000 – 3,500) x 115% = 7,475 shares x P35] 261,625.00
P830,625.00

Net loss of the venture (P909,606.25 – P830,625) P 78,981.25

22. D Investment of Cruz (8,000 shares @ P45) P360,000.00


Less Share in JV net loss (P78,981.25 x 8/20) 31,592.50
Share of Cruz after distribution of proceeds P328,407.50

23. B Loss upon the investment of shares (8,000 shares @ P10) P 80,000.00
Share in JV loss 31,592.50
Loss of Cruz on the disposition of Palawan Oil Co. shares P 111,592.50

Loss on the disposition of the shares of Cruz is the total of the loss upon
investment of the shares (i.e. P45 – P55 = P10 per share) and the share on the net
loss of the dissolved joint venture.

24. A 20,000 shares x P40 MV = P800,000

25. A 20,000 – 4,500 + 15,500 x 120% = 18,600 – 5,000 = 13,600 x P1 = P13,600

26 B Proceeds from sale of shares;


4,500 x P44 P198,000
5,000 x P25 125,000
6,000 x P28 168,000
7,600* x P35 266,000 P757,000
Cost of the shares (see # 1) 800,000
Loss from sale of the shares P 43,000
Expenses (3,000 + 4,700) ( 7,700)
AA1 - Chapter 4 – Joint Ventures (2005)
Suggested Answers page 18

Dividend revenue
Number of shares after stock dividend 18,600
Less shares sold on November 5 5,000
Shares entitled to cash dividend 13,600
Dividend per share x P1 13,600
Net loss P 37,100
x 6/20
Share of Roxas on the venture loss P11,130

* Contributed shares 20,000


Shares sold on Oct. 20 4,500
Remaining shares 15,500
Shares received as stock dividend (20% x 15,500) 3,100
Shares sold on Nov. 5 and 22 (11,000)
Shares sold at P35 7,600

27. D 20,000 – 4,500 = 15,500 x 20% = 3,100

28. C Investment (10,000 shares @ P40) P400,000


Share on the joint venture loss (P37,100 x 1/2 ) 18,550
Share of Silverio on the distribution of proceeds P381,450

29. A Loss upon contribution of the shares [(P40 – P62) x 4,000] P88,000
Share on the JV loss (P37,100 x 4/20) 7,420
Tan’s loss on disposition of his investment in Golden Copper P95,420
CHAPTER 4
SUGGESTED ANSWERS

EXERCISES

Exercise 4 - 1

Paulo, Edwin and Marco


Worksheet Summarizing Joint Venture Transactions
May 12 to 26, 2008

Joint Venture Paulo Edwin Marco


Debit Credit Debit Credit Debit Credit Debit Credit
Construction of stand P 3,000 P 3,000
Purchases 30,000 P 30,000
Additional purchases 60,000 45,000 P10,000 5,000
Sales P153,000 P 80,000 P33,000 P40,000
Stand taken by Edwin 2,000 2,000
Unsold merchandise taken by Marco 4,000* 4,000
P 93,000 P159,000 P 80,000 P 75,000 P35,000 P10,000 P44,000 P 8,000
Net profit 66,000 _______
P159,000 P159,000
Distribution of profit:
Permit paid by Paulo P 1,530
Commission 24,000 P 9,900 P12,000
Balance – shared 50%, 10% and 40% 9,285 1,857 7,428
Share in net income _______ P 34,815 _______ P11,757 _______ P19,428
Totals P 80,000 P109,815 P35,000 P21,757 P44,000 P27,428
Cash settlement 29,815 _______ ______ 13,243 ______ 16,572
P109,815 P109,815 P35,000 P35,000 P44,000 P44,000

* Purchases P90,000 ** If markup is 80% of cost, then sales is 180% of cost.


Cost of sales (P153,000 / 180%**) 85,000
Inventory, end P 5,000
Value upon which participants can purchase inventory x 80%
Value assigned to the inventory taken by Lito P 4,000
AA1 - Chapter 4 (2008 edition) page 2

Exercise 4 – 3

Books of Jolly Books of Bernie Books of Sonny


To set up inventory, end Joint Venture Invty 110,000 Jolly 110,000 Jolly 110,000
(should be set up before Joint Venture 110,000 Joint Venture 110,000 Joint Venture 110,000
recognizing gain or loss)

To recognize gain or loss, Joint Venture 160,000 Joint Venture 160,000 Joint Venture 160,000
shared 4:2:2 Income from JV 80,000 Jolly 80,000 Jolly 80,000
NI=P50,000 + P110,000 Bernie 40,000 Income from JV 40,000 Bernie 40,000
Sonny 40,000 Sonny 40,000 Income from JV 40,000

To record settlement with Sonny 78,000 Sonny 78,000 Cash 62,400


Sonny* JV Cash 62,400 Jolly 72,800 Loss from JV 15,600
Income from JV 10,400 Income from JV 5,200 Bernie 72,000
Bernie 5,200 Jolly 150,000

* Interest of Sonny (P38,000 + P40,000) P78,000


Bonus to Jolly and Bernie (P78,000 x 20%) 15,600
Cash settlement to Sonny P62,400

The bonus to Jolly and Bernie represents a gain to them and a loss to Sonny. The P15,600 bonus shall be shared by Jolly and Bernie in the ratio of 4:2. The, the
sharing is as follows: Jolly – P15,600 x 4/6 = P10,400; Bernie – P15,600 x 2/6 = P5,200.
AA1 - Chapter 4 (2008 edition) page 3

Problem 4-1

Dario, Val, and Rene


Worksheet Summarizing Joint Venture Transactions
August 7 - 10, 2008

Joint Venture Dario Val Rene


Debit Credit Debit Credit Debit Credit Debit Credit
Construction of stand 1,000 P 1,000
Purchases 10,000 10,000
Payment of permit to operate 500 500
Additional purchases 15,000 10,000 5,000
Sales 45,000 15,000 20,000 10,000
Fire extinguishers divided among venturers 3,333 3,333 3,334 10,000
Unsold merchandise taken by Dario 1,250 1,250
26,500 46,250 19,583 21,000 23,333 5,000 13,334 10,500
Net profit 19,759
46,250 46,250
Distribution of profit:
Cleaning of lawn of Rene 500
Commission 6,000 8,000 4,000
Balance -75% to Dario and 25% to Val 938 312
Totals 19,583 27,938 23,333 13,312 13,334 15,000
Cash settlement 8,355 10,020 1,666
27,938 27,938 23,333 23,333 15,000 15,000

Purchases P10,000 + P15,000 P25,000


Cost of Sales P45,000/200% 22,500
Inventory end P 2,500
X 50%
Inventory value taken by Dario P 1,250
CHAPTER 5
SUGGESTED ANSWERS

EXERCISES

Exercise 5 – 1
1. Inventory on Consignment 90,000
Merchandise Inventory 90,000
To record transfer of merchandise to consignee.

2. Consignee Receivable (P24,000 x 130%) 31,200


Consignment Sales Revenue 31,200
To record consignment sales.

3. Cost of Consignment Goods Sold 24,000


Inventory on Consignment 24,000
To record cost of goods sold.

4. Merchandise Inventory 66,000


Inventory on Consignment 66,000
To record return of consigned goods.

5. Commission Expense (P31,200 x 10%) 3,120


Cash 28,080
Consignee Receivable 31,200

or
2. Commission Expense 3,120
Cash 28,080
Consignment Sales Revenue 31,200

3. Cost of Consignment Goods Sold 24,000


Inventory on Consignment 24,000
To record cost of goods sold.

4. Merchandise Inventory 66,000


Inventory on Consignment 66,000
To record return of consigned goods.

Exercise 5 – 2
1. Consignor Books:
Inventory on Consignment 500,000
Finished Goods Inventory 500,000

Inventory on Consignment 40,000


Cash 30,000
Consignee Payable 10,000
AA1 - Chapter 5 (2008 edition) page 2

Commission Expense (P420,000 x 10%) 42,000


Consignee Receivable 378,000
Consignment Sales 420,000

Cost of Consignment Goods Sold 362,880


Inventory on Consignment 362,880

*Cost of goods sold – P420,000 /1.25 = P336,000


Freight costs – P40,000 / P500,000 = 8%
Freight on goods sold – P336,000 x 8% = P26,880
Total costs of goods sold – P336,000 + P26,880 = P362,800

Cash 278,000
Consignee Payable 10,000
Consignee Receivable 288,000

2. Consignee Books:
Memorandum entry to record receipt of goods on consignment.

Consignor Receivable 10,000


Cash 10,000

Cash 420,000
Consignor Payable 420,000

Consignor Payable 42,000


Commission Revenue 42,000

Consignor Payable 288,000


Cash 278,000
Consignor Receivable 10,000

3. Consignor financial statements:

Statement of Financial Position – Assets section


Consignee receivable P 90,000
Inventory on consignment 177,120

Statement of Recognized Income and Expenses


Consignment sales P420,000
Less Cost of consignment sales 362,880 P57,120
Commission expense 42,000
Profit from consignment P15,120

Exercise 5-3
Requirement 1 – Consignment profits calculated separately

Books of Consignor
1. Consignment –Out 7,000
Merchandise Shipment on Consignment 7,000
AA1 - Chapter 5 (2008 edition) page 3

2. Cash 3,500
Consignment-Out 2,500
Consignment-Out 6,000

3. Consignment-Out 1,300
Consignment Income 1,300
Sales (4 sets @P1,500) P6,000
CGS (4 sets @P700) ( 2,800)
Freight-in (4/10 x P1,000) ( 400)
Commission (25% x P6,000) ( 1,500)
Consignment income P1,300

Books of Consignee
1. Received 10 sets of electric fan from Ledesma …………

2. Consignment-In 1,000
Cash 1,000

3. Cash 6,000
Consignment-In 6,000

4. Consignment-In 1,500
Consignment Income 1,500

5. Consignment-In 3,500
Cash 3,500

Requirement 2 – Consignment profits not calculated separately


Books of Consignor
1. Shipped merchandise to Amoranto, a consignee, ………

2. Cash 3,500
Freight 400
Commission Expense 1,500
Merchandise on Consignment 600
Sales 6,000

3. Merchandise on Consignment 4,200


Income Summary 4,200
Unsold merchandise on consignment (6 sets @P700)

Books of Consignee
1. Received 10 sets of electric fan from Ledesma, a consignor.. ………………..

2. Ledesma 1,000
Cash 1,000

3. Cash 6,000
Sales 6,000
AA1 - Chapter 5 (2008 edition) page 4

4. Purchases (P6,000 – P1,500 commission) 4,500


Ledesma 4,500

5. Ledesma 3,500
Cash 3,500

Exercise 5 – 4
1. Sales of laser discs, net of commissions and cartage P181,800
Less: Cost P180,000
Freight and handling 5,400 185,400
Loss on laser discs consignment P 3,600

2. Sales of TV sets, net of commissions and delivery & installation costs P173,250
Less: Cost (15 x P9,000) P135,000
Freight and handling [(15 + 3*)/24** x P10,800] 8,100
Freight on return of defective units 1,080 144,180
Profit on TV sets on consignments P 29,070

* Number of units returned = Cost of returned units / Cost per unit


= P27,000 / P9,000
= 3

** Units shipped to consignee = Total cost of TV sets shipped / Cost per unit
= P216,000 / P9,000
= 24

3. TV sets P54,000 + (P10,800 6/24 = P2,700) = P56,700

Exercise 5 – 5
1. Sales (300 x P1,000) + (100** x P1,100) P410,000*
Cost of sales (400 x P600) 240,000
Gross profit P170,000
Expenses:
Freight (400/500 x P5,500) P 4,400
Safety devices (100/200 x P10,000) 5,000
Commission (P410,000 x 10%) 41,000
Delivery cost 4,500 54,900
Consignment profit P115,100

*Consignee remitt6ance and charges represent only 90% of sales in as much as the 10% commission
of the consignee has not yet been included among the charges (P364,500 + P4,500 = P369,000 /
90% = P410,000)

** The 100 units with safety device sold at P1,100 is computed as follows:
Sales P410,000
Sales of units without safety device (300 x P600) 300,000
Sales of units with safety device P110,000
Units sold (P110,000 / P1,100) 100
AA1 - Chapter 5 (2008 edition) page 5

2. Cost (100 x P600) P60,000


Freight (100/500 x P5,500) 1,100
Safety device (100/200 x P10,000) 5,000
Inventory cost of consigned goods P66,100

PROBLEMS

Problem 5-1
Req. 1. Books of consignee; consignment sales merged with regular sales

April Received 20 sets of VCD player………………

CCM Corp. 1,750


Cash 1,750

Accounts Receivable 24,000


Sales 24,000

Purchases 20,000
CCM Corp. 20,000

Cash 10,000
Accounts Receivable 10,000

CCM Corp. 5,000


Cash 5,000

May Accounts Receivable 18,000


Sales 18,000

Purchases 15,000
CCM Corp. 15,000

Cash 15,000
Accounts Receivable 15,000

CCM Corp. 10,000


Cash 10,000
Req. 2 – Books of consignor; consignment profits calculated separately

April Consignment-Out 36,000


Merchandise Inventory 36,000

Cash 5,000
Consignment – Out 1,750
Receivable – Consignee 13,250
Consignment – Out 20,000

Consignment – Out 4,900


Consignment Income 4,900
AA1 - Chapter 5 (2008 edition) page 6

Sales (8 sets @P2,500) P 20,000


CGS (8 sets @P1,800) ( 14,400)
Freight-in (P1,000 x 8/20) ( 400)
Cartage-in (P750 x 8/20) ( 300)
Consignment profit P 4,900

May Cash 10,000


Receivable – Consignee 5,000
Consignment – Out 15,000

Consignment – Out 3,675


Consignment Income 3,675
Sales (6 sets @P2,500) P15,000
CGS (6 sets @P1,800) ( 10,800)
Freight-in (P1,000 x 6/20) ( 300)
Cartage-in (P750 x 6/20) ( 225)
Consignment profit P 3,675

3. Charge and Profit analysis

April May
Total Sales Inventory Sales Inventory
Charges by consignor:
Cost of consigned goods P36,000 P14,400 P21,600 P10,800 P10,800
Charges by consignee
Freight-in 1,000 400 600 300 300
Cartage-in 750 300 450 225 225
Total P37,750 P15,100 P22,650 P11,325 P11,325
Sales price 20,000 15,000
Consignment profit P 4,900 P 3,675

Problem 5-2
Books of consignor

1. Consignment – Out 50,000


Merchandise Shipment on Consignment 50,000

2. Consignment – Out 750


Cash 750

3. Cash 5,000
Consignment – Out 10,600
Receivable – Consignees 16,400
Consignment – Out 32,000

4. Merchandise Shipment on Consignment 4,000


Consignment – Out 4,000
Returned merchandise (4 sets @P1,000)
AA1 - Chapter 5 (2008 edition) page 7

5. Consignment – Out 5,700


Consignment Income 5,700

Total Sales Inventory


Charges by consignor:
Cost of consigned goods P46,000 P16,000 P30,000
Freight-out 750 300 450
Charges by consignee
Cartage-in 1,000 400 600
Delivery and installation 1,600 1,600
Commission 8,000 8,000
Total P57,350 P26,300 P31,050
Sales price 32,000
Consignment profit P 5,700
*Note: Freight and cartage on sets returned are charged against sales of the period.

Books of consignee
1. Received 50 cordless phones………….

2. Accounts Receivable 32,000


Consignment – In 32,000

3. Cash 15,600
Accounts Receivable 15,600
P32,000 – P16,400 (collectible) = P15,600

4. Returned four (4) defective cordless phones…………….

5. Consignment – In 15,600
Delivery and Installation Expense 1,600
Commission on Consignment 8,000
Cash 5,000
Remittance

Problem 5-3
Correcting entry to bring accounts with Alejo up to date

Consignment Commissions 58,500


Freight on Consignment Shipments (P2,600 x 65/100) 1,690
Prepaid Expenses on Consigned Merchandise (P2,600 x 35/100) 910
Consignment Sales 6,000
Alejo 67,100

Account sales – Alejo


Sales (65 stoves @ P3,600) P234,000
Commission (25% of P234,000) ( 58,500)
Freight ( 2,600)
Amount owed P172,900
Total amount remitted ( 187,900)
Balance-charge against Sunstar P 15,000
AA1 - Chapter 5 (2008 edition) page 8

Entry that should have been made for transactions of Alejo on the books of consignor:

Cash 187,900
Consignment Commission 58,500
Freight on Consignment Shipments 1,690
Prepaid Expenses on Consigned Merchandise 910
Consignment Sales 234,000
Alejo 15,000

Net effect of entries already made with Alejo for transfer of merchandise and remittance

Cash 187,900
Alejo 52,100
Consignment Sales 240,000

Correcting entry to bring accounts with Burgos up to date

Consignment Commissions 13,500


Freight on Consignment Shipments (P2,600 x 65/100) 750
Prepaid Expenses on Consigned Merchandise (P2,600 x 35/100) 1,250
Consignment Sales 42,000
Burgos 57,500

Account sales – Burgos


Sales (15 stoves @ P3,600) P54,000
Commission (25% of P54,000) ( 13,500)
Freight ( 2,000)
Amount owed P38,500
Total amount remitted ( 11,500)
Balance-amount owed Sunstar P27,000

Entry that should have been made for transactions of Burgos on the books of consignor:

Cash 11,500
Consignment Commission 13,500
Freight on Consignment Shipments 750
Prepaid Expenses on Consigned Merchandise 1,250
Burgos 27,000
Consignment Sales 54,000

Net effect of entries already made with Burgos for transfer of merchandise and remittance

Cash 11,500
Burgos 84,500
Consignment Sales 96,000

Problem 5-4
Entries to bring account with Domingo up to date
AA1 - Chapter 5 (2008 edition) page 9

1. Cash 5,100
Operating Expenses 900
Receivables-Consignees 6,000
Remittance for 10 sets less charges

2. Sales 3,000
Receivables – Consignees 3,000
Unsold units previously recognized as sales.

3. Merchandise in Transit 1,600


Cost of Goods Sold 1,600
Merchandise returned and still in transit.

Entries to bring account with Estrella up to date


1. Cash 13,400
Merchandise on Consignment 600
Operating Expenses 1,000
Receivables – Consignees 15,000

2. Sales 9,000
Receivables – Consignees 9,000

3. Merchandise on Consignment 4,800


Cost of Goods Sold 4,800
Unsold units in the hands of consignee

4. Merchandise on Consignment 1,700


Operating Expenses 1,700
Expenditures related to unsold consigned goods
charged to expense

Entries to bring account with Fajardo up to date


1. Operating Expenses 960
Receivables – Consignees 600
Accounts Payable 360

2. Sales 2,400
Receivables – Consignees 2,400

3. Merchandise in Transit 1,280


Receivables – Consignees 1,280

Closing Entries
1. Sales 715,600
Cost of Goods Sold 420,100
Operating Expenses 89,160
Income Summary 206,340

2. Income Tax 72,219


Income Tax Payable 72,219
AA1 - Chapter 5 (2008 edition) page 10

3. Income Summary 72,219


Income Tax 72,219

4. Income Summary 134,121


Retained Earnings 134,121

2.
Moonstar Company
Statement of Financial Position
December 31, 2008

Current assets: P134,000


Cash 62,720
Receivables – Consignees
Inventories:
On hand P112,000
In transit 2,880
On consignment 7,100 121,980
Total Current Assets 318,700
Plant and Equipment 170,000
Total Assets P488,700

Current Liabilities:
Accounts Payable P25,360
Income Tax Payable 72,219
Ordinary Share Capital, P100 par P200,000
Retained Earnings
Balance, beginning P 57,000
Net income for 2008 134,121 191,121
Total Shareholders’ Equity 391,121
Total Liabilities and Shareholders’ Equity P488,700

MULTIPLE CHOICE

1. A
2 C
3. A
4. A P180,000 + P9,000 = P189,000

5. A Receipts (215 x P500) P107,500


Less Shipping charges 2,100
Remittance P105,400

6. B (215 x 40% x P580) + (215 x 60% x P640) – (215 x P500) = P24,860

7. D Receipts ( 7 dozens x 12 x P2,000) P168,000


Charges: Expenses P 3,000
Commissions (15% x P168,000) 25,200 28,200
Remittance P139,800
AA1 - Chapter 5 (2008 edition) page 11

8. C Sales 7 x 12 x P2,000 P168,000


Cost of Sales 7 x 12 x P1,000 P84,000
Freight 7 x P30 210
Expenses 3,000
Commission P168,000 x 15% 25,200 112,410
P 55,590
9. C
10. B Sales ( 4 x P7,000) P28,000
Charges: Commission (20% x P28,000) P5,600
Freight 1,600 7,200
Remittance P20,800

11 D Cost (6 x P4,000) P24,000


Freight (P1,600 x 6/10) 960
Balance of Merchandise on Consignment account P24,960

12 D Sales P28,000
Cost (4 x P4,000) 16,000
Gross profit P12,000
Less: Commission P5,600
Freight (P1,600 x 4/10) 640 6,240
Net profit on consignment P 5,760

13. B Remittance P64,980


Charges:
Delivery expense P 850
Repairs 2,000 2,850
Total P67,380
÷ 85%
Sales P79,800
Cost of sales 52,000*
Gross profit P27,800
Expenses:
Commission (P79,800 x 15%) P11,970
Repairs (P2,000 x 60/100) 1,200
Delivery 850
Shipping cost (P900 x 260/300) 780 14,800
Consignment profit P13,000

*Sales P79,800
Less Sales of units with defects (200 x P300) 60,000
Sales of repaired units P19,800
Selling price of repaired units ÷ P330
Number of repaired units that were sold 60
Units sold without repairs 240
Total number of units sold 300
Unit cost x P200
Cost of sales P52,000
AA1 - Chapter 5 (2008 edition) page 12

14 B Cost (40 x P200) P 8,000


Repairs (P2,000 x 40/100) 800
Shipping cost (P900 x 40/300) 120
Value of inventory on consignment P 8,920

15. D Remittance P54,600


Charges:
Cartage P 600
Advertising 3,600
Delivery and installation 2,400 6,600
Total P61,200
÷ 85%
Sales P72,000

16 C Sales P72,000
Cost and expenses:
Cost (6 x P7,200) P43,200
Freight (P4,800 x 6/10) 2,880
Cartage (P600 x 6/10) 360
Advertising 3,600
Delivery and installation 2,400
Commission (15% x P72,000) 10,800 63,240
Consignment income P 8,760

17. D Remittance P68,250


Consignee charges, excluding the 15% commission
(P4,500 + P3,000 + P750) 8,250
Sum of remittances and charges P76,500
÷ 85%
Sales price of 6 refrigerators P90,000

18 A Sales P90,000
Cost of sales (6 x P9000) 54,000
Gross profit P36,000
Expenses:
Commission (P90,000 x 15%) P13,500
Freight-out (P6,000 x 6/10) 3,600
Marketing expense 4,500
Delivery and installation 3,000
Cartage (P750 x 6/10) 450 25,050
Net profit from the sale of consigned goods P10,950

19 D P90,000 x 15% = P13,500

20 A Sales collected [(2 x P1,500) + (1 x P1,800 x 25%)] P3,450


Commission (P3,450 x 15%) 720
Remittance P2,730
AA1 - Chapter 5 (2008 edition) page 13

21 C Sales P4,800
Cost of sales ( 3 x P800) 2,400
Gross profit P2,400
Expenses:
Trucking (P200 x 3/5) P120
Delivery 170
Commission 720 _____
Profit resulting from consignment P1,390

22 B Sales P21,000
Cost (7 x P2,000) 14,000
Gross profit P 7,000
Expenses:
Advertising P1,000
Commission 4,200
Freight (P600 x 7/10) 420 5,620
Net income on the consignment P 1,380

23 C ( 3 x P2,000) + (P600 x 3/10) = P6,180

24 D Remittance P3,750
Charges by consignor
Cost (P3,840 x 30/48) P2,400
Freight and handling (P1,000 x 30/48) 625
Freight and handling charged by consignee 75 3,100
Net income P 650

25 C T-shirts:
Cost (P3,840 x 18/48) P1,440
Freight and handling (P1,000 x 18/48) 375 P1,815
Baby dresses:
Cost (P2,400 x 4/24) P 400
Freight and handling (P540 x 4/24) 90 490
Cost of the inventory in the hands of consignee P2,305

26. Sales P84,000


Cost of sales (P90,000 x 7/10) 63,000
Gross profit P21,000
Expenses:
Freight (P3,000 x 7/10) P2,100
Commission 8,400
Cartage-in (P1,050 x 7/10) 735 11,235
Profit on consignment of TV sets P 9,765

27. C TV sets (P94,050 x 3/10) P28,215


DVD sets (P84,450 x 2/5) 33,780
Cost of inventory in the hands of consignee P61,995

28. A P3,375 / 15% = P22,500 / P1,500 15 units


AA1 - Chapter 5 (2008 edition) page 14

29 C Sales (15 x P1,500) P22,500


Less: Advertising P2,250
Delivery expense 1,125
Commission 3,375
Advances (150 x P900 x 60% x 15/150) 8,100 14,850
Remittance P 7,650

30 A Sales P22,500
Less Cost (15 x P900) P13,500
Expenses (P2,250 + P1,125 + P3,375) 6,750 20,250
Consignment profit P 2,250

31 D Sales P10,800
Less: Cost (P8,400 x 9/12) P6,300
Freight-out (P720 x 9/12) 540
Delivery 450
Commission 2,160
Advertising 500 3,650
Net income P 850

32. B P10,800 – P450 – P2,160 – P500 = P7,690

33 B P9,120 x 3/12 = P2,280

34. C Net income P 426


Expenses:
Freight (P240 x 6/10) P 144
Advertising 150
Commission 480 774
Gross profit P1,200
Less Sales 2,400
Cost of goods sold P1,200

35 B Cost per unit (P1,200 / 6 units) P200


Unsold units x 4
Total cost of unsold units P800
Inventoriable cost – freight (P240 x 4/10) 96
Total cost P896

36. A Remittance P3,165


Charges: Advertising P120
Delivery 75 195
Total proceeds from sales, net of 20% commission P3,360
÷ 80%
Total sales price of the 7 handbags P4,200

37 C Cost (3 x P300) P 900


Freight (3/10 x P150) 45
Advertising ( 3/10 x P120) 36
Inventory of unsold handbags P 981
AA1 - Chapter 5 (2008 edition) page 15

38 B Sales P4,200
Cost of sales (7 x P300) 2,100
Gross profit P2,100
Expenses:
Freight ( 7/10 x P150) P 105
Commission (4,200 x 20%) 840
Advertising ( P120 x 7/10) 84
Delivery 75 1,104
Net income on the consignment P 996

39. B Sales (80 x P1,500) P120,000


Charges:
Cost of returning defective units P 100
Transportation 350
Insurance 200
Commissions (P120,000 x 20%) 24,000
Reconditioning cost 150 24,800
Balance P 95,200
Advances (80 + 10) x P300 27,000
Remittance P 68,200

40 D Sales P120,000
Cost of sales (80 x P500) 40,000
Gross profit P 80,000
Expenses:
Transportation (90/120 x P1,500) P 1,125
Insurance 900
Cost of returning defective units 100
Transportation of consignee (90/120 x P350) 265
Insurance – consignee (90/120 x P200) 150
Insurance loss (P500 x 10% x 10) 500
Commission 24,000
Reconditioning cost 150 27,190
Profit on consignment P 52,810
CHAPTER 6
SUGGESTED ANSWERS

EXERCISES

Exercise 6-1
2006 sales 2007 sales
Installment Accounts Rec’l, Jan. 1 P 400,000 P 200,000
Less Installment Accounts Rec’l, Dec. 31 100,000 40,000
Collections P 300,000 P 160,000
Gross profit rate (10,000/40,000; 4,400/20,000) 25%__ 22%__
Realized Gross Profit P 75,000 P 35,200

2006 sales 2007 sales


Deferred Gross Profit, beg P 100,000 P 44,000
Less Deferred Gross Profit,end P100,000 x 25%, P40,000 x 22% 25,000 8,800
Realized Gross Profit P 75,000 P 35,200

Exercise 6-2
Deferred Gross Profit – 2006 [(P150,000 - -0- ) x 42%] 63,000
Deferred Gross Profit – 2007 [(P480,000 - P120,000) x 37.5%] 135,000
Deferred Gross Profit – 2008 [(P750,000 - P650,000) x 40%]* 40,000
Realized Gross Profit 238,000
* 66 2/3 ÷ 166 2/3 = 40%
P300,000 ÷ 40% = P750,000

Exercise 6-3
(G) (1) P50,000 - P11,000 P 39,000
(E) (2) P10,500 - (25% of P20,000) = P5,500/P25,000 22%
(F) (3) P50,000 x 22% P 11,000
(H) (4) P1,100/22% P 5,000
(B) (5) P80,000 x 75% P 60,000
(A) (6) P80,000 x 25% P 20,000
(C) (7) P28,200 + P91,800 P120,000
(D) (8) P28,200/P120,000 23.5%
(9) 2006 = P10,000 x 22% = P 2,200
2007 = P50,000 x 25% = 12,500
2008 = P45,000 x 23.5% = 10,575 P 25,275

Exercise 6-4
1. Deferred Gross Profit – 2006 4,500
Deferred Gross Profit – 2007 14,000
Deferred Gross Profit – 2008 69,000
Realized Gross Profit 87,500
2006 2007 2008
Deferred gross profit before adj. P 8,000 P26,000 P105,000
Deferred gross profit after adj.
(Inst. contract rec’l x GP rate) __3,500 _12,000 __36,000
Realized gross profit P 4,500 P14,000 P 69,000
AA1 - Chapter 6 (2008 edition)
page 2

2. Cash collections
2006 sales - P4,500 /35% P 12,857
2007 sales - p14,000/30% 46,667
2008 sales - P69,000/40% 172,500
Total P232,024

2006 2007 2008


Installment Contract Receivable, beg. P 22,857 P86,667 P262,500
Less Installment Contract Receivable, end 10,000 40,000 90,000
Collections P12,857 P46,667 P 172,500

Exercise 6-5
a. Installment Contracts Receivable 250,000
Installment Sales 250,000

b. Cash 120,000
Installment Contracts Receivable 120,000

c. Cost of Installment Sales 200,000


Inventory 200,000
50,000 / 250,000 = 20%

d. Inventory of Repossessed Merchandise 14,500


Deferred Gross Profit (P20,000 x 20%) 4,000
Loss on Repossession 1,500
Installment Contracts Receivable 20,000

e. Expenses 16,000
Cash 16,000

f. Installment Sales 250,000


Cost of Installment Sales 200,000
Deferred Gross Profit 50,000

g. Deferred Gross Profit 24,000


Realized Gross Profit 24,000
P120,000 x 20% = P 24,000

h. Realized Gross Profit 24,000


Expenses 16,000
Loss on Repossession 1,500
Income Summary 6,500

Exercise 6-6
a. Installment Contracts Receivable 600,000
Installment Sales 600,000

b. Cost of Installment Sales 405,000


Shipments on Installment Sales 405,000
AA1 - Chapter 6 (2008 edition)
page 3

c. Cash 360,000
Installment Contracts Receivable 360,000

d. Repossessed Merchandise 24,000


Deferred Gross Profit (P40,000 x 32.5%) 13,000
Loss on Repossession 3,000
Installment Contracts Receivable 40,000

e. Installment Sales 600,000


Cost of Installment Sales 405,000
Deferred Gross Profit 195,000
195,000/600,000 = 32.5%

f. Deferred Gross Profit 117,000


Realized Gross Profit 117,000
P360,000 x 32.5% = P117,000

Exercise 6-7

Requirement 1
a. Cash 1,400
Installment Contracts Receivable 1,240
Interest Revenue 160

b. Repossessed Merchandise 2,000


Deferred Gross Profit (P4,000 x 40%) 1,600
Loss on Defaults 400
Installment Contracts Receivable 4,000

c. Deferred Gross Profit 496


Realized Gross Profit (P1,240 x 40%) 496

Requirement 2
a. Repossessed Merchandise 2,000
Deferred Gross Profit 1,600
Loss on Defaults 3,600

Exercise 6-8
Repossessed Merchandise (P13,500/120%) 11,250
Deferred Gross Profit (P15,000 x 20%/120%) 2,500
Loss on Repossession 1,250
Installment Contracts Receivable 15,000

Exercise 6-9
a. Trade-In Merchandise 180,000
Installment Contracts Receivable 420,000
Installment Sales 600,000
AA1 - Chapter 6 (2008 edition)
page 4

Estimated resale price P280,000


Less: Reconditioning cost P30,000
Gross profit _70,000 __100,000
Estimated realizable value P 180,000

Selling price P720,000


Less Overallowance (P300,000 – P180,000) 120,000
Adjusted selling price P600,000

b. Cost of Installment Sales 500,000


Automobiles 500,000

c. Installment Sales 600,000


Cost of Installment Sales 500,000
Deferred Gross Profit 100,000

d. Deferred Gross Profit 30,000


Realized Gross Profit 30,000
P180,000 x 16 2/3% = P30,000

Exercise 6 - 10
Correct entry
Allowance for Doubtful Installment Contract Rec’l 1,450
Deferred Gross Profit ( P 10,000 x 25/125 ) 2,000
Repossessed Merchandise 6,550
Installment Contract Receivable 10,000

Correcting Entry
Deferred Gross Profit 2,000
Repossessed Merchandise 6,550
Allowance for Doubtful Accounts 8,550

Exercise 6-11
Requirement 1
Oct. 31 Cash 200,000
Installment Contracts Receivable 550,000
Real Estate 600,000
Deferred Gross Profit 150,000
GP rate = P150,000/P750,000 = 20%

Nov. 30 Cash 11,500


Installment Contracts Receivable 6,000
Interest Revenue 5,500
550,000 x 1% = 5,500

Dec. 31 Cash 11,440


Installment Contracts Receivable 6,000
Interest Revenue 5,440
P550,000 – P6,000 = P544,000 x 1% = P5,440
AA1 - Chapter 6 (2008 edition)
page 5

31 Deferred Gross Profit 42,400


Realized Gross Profit 42,400
P200,000 + P6,000 + P6,000 = P212,000 x 20%
Requirement 2
Oct. 31 Cash 200,000
Installment Contracts Receivable 550,000
Real Estate 600,000
Deferred Gross Profit 150,000
GP rate = P150,000/P750,000 = 20%

Nov. 30 Cash 6,000


Installment Contracts Receivable 500
Interest Revenue 5,500
P550,000 x 1% = P5,500

Dec. 31 Cash 6,000


Installment Contracts Receivable 505
Interest Revenue 5,495
P550,000 – P500 = P549,500 x 1% x = P5,495

31 Deferred Gross Profit 40,201


Realized Gross Profit 40,201
P200,000 + P500 + P505 = P201,005 x 20% = P40,201

Exercise 6-12
Recovery of cost Realized gross profit
a. 2008 P400,000 none
2009 25,000 P 75,000
2010 – 2014 100,000/year

b. 2008 none P400,000


2009 none P100,000
2010 P 25,000 P 75,000
2011 – 2014 P100,000/year none

c. 2008 P170,000 P230,000


2009 – 2014 P 42,500 P 57,500

Exercise 6-13
1. Installment payment = P1,260,000/5.6502 = P223,000
2. Journal entries
Jan. 1 Cash 140,000
Notes Receivable (P223,000 x 10) 2,230,000
Real Estate Sales 1,400,000
Discount on Notes Receivable 970,000

Jan. 1 Cost of Real Estate Sales 850,000


Inventory or Real Estate 850,000
AA1 - Chapter 6 (2008 edition)
page 6

1 Real Estate Sales 1,400,000


Cost of Real Estate Sales 850,000
Deferred Gross Profit 550,000
GP rate = P550,000/P1,400,000 = 39.29%

Dec. 31 Cash 223,000


Notes Receivable 223,000

31 Discount on Notes Receivable 151,200


Interest Revenue 151, 200
(P2,230,000 – P970,000) x 12% = P151,200

31 Deferred Gross Profit 83,216


Realized Gross Profit 83,216
[P140,000 + (P223,000- P151,200)] x 39.29%

Exercise 6-14
Requirement 1
a. Land 13,440,000
Land Improvements 3,360,000
Cash 16,800,000

b. Cash 750,000
Installment Contract Receivable 4,250,000
Installment Sales 5,000,000
A – 5 @ P400,000 = P2,000,000
B – 8 @ P300,000 = 2,400,000
C – 3 @ P200,000 = 600,000
Total P5,000,000

c. Cost of Installment Sales 3,000,000


Land 2,400,000
Land Improvements 600,000
Cost of sales = 16,800,000/28,000,000 = 60% x P5,000,000 = P3,000,000
Land = 13,440,000/16,800,000 = 80% x P3,000,000 = P2,400,000
Land imp. = 3,360,000/16,800,000 = 20% x P3,000,000 = P 600,000

d. Cash 1,400,000
Installment Contract Receivable 1,300,000
Interest Revenue 100,000
Requirement 2
a. Installment Sales 5,000,000
Cost of Installment Sales 3,000,000
Deferred Gross Profit 2,000,000
GP rate = 2,000,000/5,000,000 = 40%
b. Deferred Gross Profit 820,000
Realized Gross Profit 820,000
(P750,000 + P1,300,000) x 40% = P820,000
AA1 - Chapter 6 (2008 edition)
page 7

Exercise 6-15
2005 Installment Accounts Receivable 4,700,000
Installment Sales 47,000

Cost of Installment Sales 2,585,000


Inventory 2,585,000

Cash 2,585,000
Installment Accounts Receivable 2,585,000

Installment Sales 4,700,000


Cost of Installment Sales 2,585,000
Deferred Gross Profit 2,115,000

2006 Installment Accounts Receivable 4,500,000


Installment Sales 4,500,000

Cost of Installment Sales 2,610,000


Inventory 2,610,000

Cash 3,885,000
Installment Accounts Receivable – 2005 1,410,000
Installment Accounts Receivable – 2006 2,475,000

Installment Sales 4,500,000


Cost of Installment Sales 2,610,000
Deferred Gross Profit 1,890,000

Deferred Gross Profit 1,410,000


Realized Gross Profit 1,410,000

2007 Installment Accounts Receivable 5,800,000


Installment Sales 5,800,000

Cost of Installment Sales 3,074,000


Inventory 3,074,000

Cash 5,010,000
Installment Accounts Receivable – 2005 470,000
Installment Accounts Receivable – 2006 1,350,000
Installment Accounts Receivable – 2007 3,190,000

Installment Sales 5,800,000


Cost of Installment Sales 3,074,000
Deferred Gross Profit 2,726,000

Deferred Gross Profit – 2005 470,000


Deferred Gross Profit – 2006 P1,350,000 – (P2,610,000-P2,475,000) 1,215,000
Deferred Gross Profit – 2007 116,000
Realized Gross Profit 1,801,000
AA1 - Chapter 6 (2008 edition)
page 8

2008 Installment Accounts Receivable 6,100,000


Installment Sale 6,100,000

Cost of Installment Sales 3,111,000


Inventory 3,111,000

Cash 5,545,000
Installment Accounts Receivable – 2006 450,000
Installment Accounts Receivable – 2007 1,740,000
Installment Accounts Receivable – 2008 3,355,000

Installment Sales 6,100,000


Cost of Installment Sales 3,111,000
Deferred Gross Profit – 2008 2,989,000

Deferred Gross Profit – 2006 450,000


Deferred Gross Profit – 2007 1,740,000
Deferred Gross Profit – 2008 P3,355,000 – P3,111,000 244,000
Realized Gross Profit 2,434,000

Gross Profit Recognized: 2005 2006 2007 2008


Full Accrual; P 2,150,000 P 1,890,000 P 2,726,000 P 2,989,000
Cost Recovery Method 0 1,410,000 1,801,000 2,434,000

Exercise 6-16
2006 2007 2008
Installment sales P 400,000 P 475,000 P 525,000
Cost of Installment sales 248,000 280.250 341,250
Gross profit percentage 38% 41% 35%
Cash collections:
2006 Sales 128,000 232,000 28,000
2007 Sales 114,000 218,500
2008 Sales 162,750
Realized gross profit on installment sales 112,000 80,250

COMPUTATIONS :
1 P 341,250 / . 65 = P 525,000

2 P 400,000 x . 62 = P 248,000

3 1 - (P 280,250 / P 475,000) = 41%

4 Gross profit recognized in 2008 P 80,250


All costs from 2006 sales are recovered
Cash received equals gross profit 28,000
All cost from 2008 sales are not recovered
Cash received goes to recover costs-gross profit 0
Gross profit reported in 2008 from 2006 sales P 52,250
AA1 - Chapter 6 (2008 edition)
page 9

Costs of 2007 sales P 280,250


Costs recovered in 2005 114,000
Costs to be recovered in 2005 166,250
Cash received related to 2007 sales P 218,500

5 Cash collections in 2006 do not exceed costs of sales


Realized gross profit in 2006 = P 0

6 Cash collections for 2006 sales ( P 128,000 + 232,000 ) P 360,000


Costs of 2006 sales 248,000
Realized gross profit in 2007 P 112,000

Exercise 6-17
Requirement 1
2007
Jan. 1 Cash 1,500,000
Notes Receivable (2,197,100 x 10) 21,971,000
Real Estate Sales 15,000,000
Discount on Notes Receivable (2,347,100 - 1,500,000) 8,471,000

1 Cost of Real Estate Sales (240,000 x 30) 7,200,000


Inventory of Real Estate 7,200,000

Dec 31 Real Estate Sales 15,000,000


Cost of Real Estate Sales 7,200,000
Deferred Gross Profit - Real Estate Sales 7,800,000
GP rate = 7,800,000/15,000,000 = 52%

31 Cash 2,197,100
Notes Receivable 2,197,100

31 Discount on Notes Receivable 1,350,000


Deferred Gross Profit - Real Estate Sales 1,350,000

2008
Dec. 31 Cash 2,197,100
Notes Receivable 2,197,100

31 Discount on Notes Receivable 1,265,300


Deferred Gross Profit - Real Estate Sales 1,265,300
(P13,500,000 - P2,197,100 + P1,350,000) x 10%
Requirement 2
Date Cost Collection Unrecovered Cost
Jan. 01, 2007 7,200,000 1,500,000 5,700,000
Dec. 31, 2007 2,197,100 3,502,900
Dec. 31, 2008 2,197,100 1,305,800
AA1 - Chapter 6 (2008 edition)
page 10

Notes receivable (P21,971,000 - P2,197,100 - P2,197,100) P17,576,800


Less: Discount on notes receivable
(P8,471,000-P1,350,000- P1,265,300) P 5,855,700
Deferred gross profit (P7,800,000 + P1,350,000 + P1,265,300) 10,415,300 16,271,000
Unrecovered cost P 1,305,800

Exercise 6-18
Selling Price P 10,000,000
Cost of Land 4,000,000
Gross Profit P 6,000,000

Gross Profit rate ( 600 / 1,000 ) 60%

a) Full accrual method 2008 P 6,000,000


2009 – 2010 none

b) Installment method
Collections Cost Recovery Gross Profit
2008 P 400,000 P 160,000 P 240,000
2009 300,000 120,000 180,000
2010 ___300,000 ___120,000 ___180,000
P1,000,000 P 400,000 P 600,000

c) Cost Recovery method


Collections Cost Recovery Gross Profit
2008 P 400,000 P 400,000 P -
2009 300,000 - 300,000
2010 ___300,000 ____-___ ___300,000
P 1,000,000 P 400,000 P 600,000

PROBLEMS
Problem 6-1
1. 2007 - (P12,000* + P228,000) ÷ (P240,000 + P520,000 + P40,000) 30%
2008 - (P1,500,000 - P975,000) ÷ P1,500,000 35%
*P40,000 – P24,000 – P4,000 = P12,000
2.a. Installment Sales 1,500,000
Cost of Installment Sales 975,000
Deferred Gross Profit – 2008 525,000

b. Deferred Gross Profit – 2007 (520,000 x 30%) 156,000


Deferred Gross Profit – 2008 (740,000 x 35%) 259,000
Realized Gross Profit 415,000

c. Sales 2,120,000
Realized Gross Profit 415,000
Cost of Sales 1,650,000
Gain or Loss on Repossession 4,000
Selling and Administrative Expenses 660,000
AA1 - Chapter 6 (2008 edition)
page 11

Income Summary 221,000


d. Income Tax 77,350
Income Tax Payable 77,350

e. Income Summary 77,350


Income Tax 77,350

d. Income Summary 143,650


Retained Earnings 143,650
Requirement 3
Excellent Co.
Statement of Recognized Income and Expenses
For the Year Ended December 31, 2008

Sales P 2,120,000
Cost of Sales _1,650,000
Gross Profit P 470,000
Realized Gross Profit on Installment Sales __415,000
Total Realized Gross Profit P 885,000
Less Loss on Repossession ____4,000
Realized Gross Profit after Loss on Repossession P 881,000
Selling and Administrative Expenses __660,000
Net Income before Income Tax P 221,000
Income Tax 77,350
Net Income P143,650

Problem 6-2
Requirement 1 Computation of gross profit rates

2006 sales P22,500/P50,0000 45%


2007 sales P96,000/P240,000 40%
2008 sales 100% - (P310,000/P500,000) 38%

Requirement 2 Income Statement


Reliance Sales Corp.
Statement of Recognized income and Expenses
For the Year Ended December 31, 2008

Regular Installment
Sales Sales Total
Sales P 192,000 P 500,000 P 692,000
Cost of Sales:
Inventory, January 1 P 30,000
Purchases 455,000
Repossessed Mdse. __10,000
Cost of Goods Available
for Sale P495,000
Less Inventory, Dec. 31 __35,000 150,000 310,000 460,000
Gross Profit P 42,000 P 190,000 P 232,000
AA1 - Chapter 6 (2008 edition)
page 12

Less Deferred Gross Profit, including DGP on


repossessed merchandise ________ 32,300 32,300
Realized Gross Profit on 2008 sales P 42,000 P 157,700 P 199,700
Add Realized Gross Profit on 2006 & 2007 sales 100,650
Total P 300,350
Less Loss on Repossession [3,000-(3,600+4,000+1,900)] 3,500
Realized gross profit after loss on Repossession P 296,850
Operating Expenses 300,000
Net Loss P 3,150
Analysis of gross profit on installment sales:

2006 2007 2008


Installment sales P500,000
Installment contracts rec’l, beginning P 50,000 P240,000
Installment contracts rec’l, end ( 5,000) ( 20,000) (80,000)
Defaulted installment contracts rec’l ( 8,000) ( 10,000) (5,000)
Collections P 37,000 P210,000 P415,000
Gross profit rate 45% 40% 38%
Realized gross profit P 16,650 P84,000 P157,700

Deferred gross profit P 2,250 P 8,000 P 30,400


Requirement 3 -
Reliance Corp.
Statement of Financial Position
December 31, 2008

Assets Liabilities & Shareholders’ Equity


Cash P 25,000 Accounts Payable P 75,000
Inst. Contracts Rec’l, 2008 80,000 Deferred Gross Profit, 2008 30,400
Inst. Contracts Rec’l, 2007 20,000 Deferred Gross Profit, 2007 8,000
Inst. Contracts Rec’l, 2006 5,000 Deferred Gross Profit, 2006 2,250
Accounts Rec’l 40,000 Ordinary Share Capital 100,000
Inventory 35,000 Retained Earnings 41,350
Other Assets 52,000 ________
Total Liabilities &
Total Assets P257,000 Shareolders’ Equity P257,000

Problem 6-3
1 Schedule of Cost of Goods Sold
.
Inventory, January 1 P 240,000
Purchases, including freight-in 1,250,000
Repossessed Merchandise 70,000
Cost of Goods Available for Sale P1,560,000
Less Inventory, December 31 260,000
Cost of Goods Sold P1,300,000
AA1 - Chapter 6 (2008 edition)
page 13

2. Schedule of Allocation of Cost of Goods Sold


Type of Amount of Amount Based Allocation of
Sale Sales on Cash Sales Ratio CGS
Cash P 300,000 P 300,000 60/400 P 195,000
Charge 600,000 500,000 100/400 325,000
Installment 1,500,000 1,200,000 240/400 780,000
P2,400,000 P2,000,000 P1,300,000
3.
Fuji Products
Statement of Recognized Income and Expenses
For the Year Ended December 31, 2008
Cash Charge Installment
Sales Sales Sales Total
Sales P300,000 P600,000 P 1,500,000 P2,400,000
Cost of Sales 195,000 325,000 780,000 1,300,000
Gross Profit P105,000 P275,000 P 720,000 P1,100,000
Less Deferred Gross Profit, 2008 sales 460,800 460,800
Realized Gross Profit on 2008 sales P105,000 P275,000 P 263,200 P 639,200
Add Realized Gross Profit on 2006 and 2007 sales P 169,500
Total Realized Gross Profit P 808,700
Less Loss on Repossession 51,000
Realized Gross Profit after loss on repossession P 757,700
Operating Expenses, including bad debts 465,000
Net Income before Income Tax P 292,700
Income Tax 102,445
Net Income P 190,255

Problem 6-4
1. Computation of gross profit rates
2006 sales 160,000/400,000 40%
2007 sales 167,200/440,000 38%
2008 sales 163,800/420,000 39%

2. Adjusting entries
a. Installment Sales 420,000
Cost of Installment Sales 256,200
Deferred Gross Profit – 2008 163,800

b. Deferred Gross Profit –2006 27,200


Deferred Gross Profit –2007 58,976
Deferred Gross Profit –2008 70,980
Realized Gross Profit 157,156
2006 - (P110,000 – P28,000 - P9,000 - P5,000) x 40% = P27,200
2007 - (P250,000 – P92,000 - P2,800) x 38% = P58,976
2008 - (P420,000 – P238,000) x 39% = P70,980

3. Correcting entries
a. Deferred Gross Profit – 2006 (9,000 x 40%) 3,600
Deferred Gross Profit - 2007 (2,800 x 38%) 1,064
AA1 - Chapter 6 (2008 edition)
page 14

Operating Expenses 4,664


b. Inventory of Repossessed Merchandise(4,000 - 400 - 600) 3,000
Deferred Gross Profit – 2006 (5,000 x 40%) 2,000
Operating Expenses 5,000
The indicated gain of P600 is ignored and deducted from the
market value of the repossessed merchandise.

4. Closing entries
a. Realized Gross Profit 157,156
Operating Expenses 94,336
Income Summary 62,820

b. Income Tax 21,987


Income Tax Payable 21,987

c. Income Summary 21,987


Income Tax 21,987

d. Income Summary 40,833


Retained Earnings 40,833

Problem 6-5
(1) Sept. 30 Cash 48,000
Installment Contract Receivable 432,000
Piano 300,000
Deferred Gross Profit 180,000
60/160 = 37.5%

(2) Oct. 31 Cash 48,000


Installment Contract Receivable 45,840
Installment Revenue 2,160

Nov. 30 Cash 48,000


Installment Contract Receivable 46,069
Installment Revenue 1,931

Dec. 31 Cash 48,000


Installment Contract Receivable 46,300
Installment Revenue 1,700

Principal Principal
Date Collection Interest Reduction Balance
480,000
Sept. 30 48,000 48,000 432,000
Oct. 31 48,000 432,000 x .005 = 45,840 386,160
2,160
Nov. 30 48,000 386,160 x .005 = 46,069 340,091
1,931
Dec. 31 48,000 340,091 x .005 = 46,300 293,791
1,700
AA1 - Chapter 6 (2008 edition)
page 15

(3) Dec. 31 Deferred Gross Profit 69,828


Realized Gross Profit 69,828
48,000 + 45,840 +46,069 + 46,300 = 186,209 x 37.5% = 69,828
2006
(4) Feb. Repossessed Piano 56,000
Deferred Gross Profit 36,724
Loss on Repossession 5,207
Installment Contract Receivable 97,931
Problem 6-6

(1) Sales 1,260,000 + (2,650,000 ÷1.06) P3,760,000


Cost of Sales:
Inventory, beginning P 580,600
Purchases 2,093,000
P2,673,600
Less: Inventory end 333,000 2,340,600
Gross Profit P1,419,400
÷3,760,000
Gross Profit Rate (1,419,400 / 3,760,000 ) 37.75%

(2 Equivalent Cash Contract Interest Cash


) Installment Sales Price Balance Revenue Collection
Time of 10,000 10,600 - 2,650
Sale (10,600 ÷ 1.06) (10,600 x 25%)
1 7,350 7,950 73.50 530
(10,000 - 2,650) (10,600 - 2,650) (7,350 x 1%) (7,950 ÷ 15)
2 6,893.50 7,420 68.93 530
(7,350 - (530 - 73.50) (7,950 -530) (6,893.50 x 1%) (7,950÷ 15)
3 6,432.43 6,890 64.32 530
(6,893.50 -(530 - 68.93) (7,420 - 530) (6,432.43x 1%) (7,950 ÷15)
4 5,966.75 6,360 59.67 530
(6,432.43-(530 - 64.32) (6,890 - 530) (5,966.75 x 1%) (7,950 ÷ 15)
5 5,446.42 5,830 54.96 530
(5,966.75- (530 - 59.67) (6,360 -530) (5,496.42 x 1%) (7,950 ÷ 15)
6 5,021.38 5,300 50.21 530
(5,446.42- (530 - 54.96) (5,830 -530) (5,021.38 1%) (7,950 ÷ 15)

(3 Installment Sales Price P10,600.00


)
Less: Installment on contract 600.00
10,600 - (10,600 ÷ 1.06)
Installment sales at cash sales price P10.000.00
Less: Downpayment P 2,650.00
Collections P 1,590.00
Less interest 206.70 1,383.30 4,033.60
AA1 - Chapter 6 (2008 edition)
page 16

Balance P5,966.70
Less: DGP 5,966.75 x 37.75% 2,252.45
Repossessed Sales 2,300.00
Repossessed Inventory 1,800.00 P6,352.45
Net gain on defaults P 385.75

(4 Cash Sales (1,260,000 x 37.75%) P 475,650


)
Installment Sales:
Downpayment 662,500 x 37.75% 250,093.75
Installments on defaults 1,383.30 x 37.75% 522.20
Installment Collections P791,820.00
Less Interest (P2,650,000 - P10,600) /1.06 x .03716 92,528.40
Balance P699,291.60
x 37.75% 263,982.60
Realized Gross Profit P990,248.60

Problem 6-7
1. Total installment sales P3,450,000
Less Installment Receivable - Dec. 31 1,594,600
Total Collections in Sales P1,855,400

2. Collections P1,855,400
GPR:
Total Selling price P9,500,000
Total Cost 5,225,000
GP P4,275,000
Total sales ÷9,500,000 45%__
RGP P 834,930

3. Installment Receivable - Dec. 31 P1,594,600


GPR x 45%_
Unrealized GP P 717,570

4. Realized gross profit P 834,930


Interest Income 520,300
Operating Expenses (682,130)
Net Income before Income Tax P 673,100
Income Tax 235,585
Net Income P437,515

Problem 6-8
Sales (Schedule 1) P 8,060,000
Cost of Sales (43% of Sales, Schedule 2) 3,465,800
Gross Profit P 4,594,200
Less Sales Commission 221,000
Gross profit excluding Commission P 4,373,200
AA1 - Chapter 6 (2008 edition)
page 17

Less DGP 4,373,200 / 8,060,000 x 5,370,000 2,913,658


Realized gross profit P 1,459,542
Expenses:
Advertising P730,000
Sales Managers Salaries 900,000
Gen. Operating Expenses (2,360,000 x 1/ 4 ) 590,000 2,220,000
Net loss P 760,458

Schedule 1 – Sales
Total Sales Cash Installment
Price Received NR Balance
A 26 @ 150,000 P3,900,000 P1,650,000 2,250,000
B 32 @ 100,000 3,200,000 800,000 2,400,000
C 12 @ 80,000 960,000 240,000 720,000
P8,060,000 P2,690,000 5,370,000

Schedule 2 - Cost of Sales Rate


Unit Total
No. of lots Sales Price Sales Value
A 80 150,000 P12,000,000
B 100 100,000 10,000,000
C 130 80,000 10,400,000
310 P32,400,000

Cost of Land P 4,800,000


Legal fees, etc. 600,000
Grading 2,250,000
Water & Sewerage 1,849,000
Paving expenses 2,663,000
General operating expenses (2,360,000 x 3/4) 1,770,000
Total cost P13,932,000
Total sales value ÷ 32,400.000
Cost of sales rate 43%

Problem 6 - 9
2007 Inventory 45,200
Cash 45,200

Notes Receivable – 2007 (32,000 + 62,000 + 3,600) 97,600


Discount on Notes Receivable - 2007 (7,167 + 3,600 ) 10,767
Installment Sales 86,833

Cost of Installment Sales (45,200 - 2,000) 43,200


Inventory 43,200

Cash 35,600
Notes Receivable - 2007 (32,000 + 3,600) 35,600
AA1 - Chapter 6 (2008 edition)
page 18

Discount on Notes Receivable - 2007 3,600


Interest Revenue 3,600

Installment Sales 86,833


Cost of Installment Sales 43,200
Deferred Gross Profit - 2007 43,633

Deferred Gross Profit - 2007 16,080


Realized Gross Profit 16,080
43,633/86,833 = 50.25% x 32,000

2008 Inventory 52,020


Cash 52,020

Notes Receivable - 2008 (160,000 + 50,000 + 5,500 - 26,000) 89,500


Discount on Notes Receivable 2008[8,043 + (5,500 - (7,167 - 11,955
5,579)]
Installment Sales 77,545

Cost of Installment Sales (52,020 - 8,000) 44,020


Inventory 44,020

Cash 55,500
Notes Receivable – 2008 (89,500 - 60,000) 29,500
Notes Receivable – 2007 (62,000 - 36,000) 26,000

Discount on Notes Receivable – 2008 (5,500 - 1,588) 3,912


Discount on Notes Receivable – 2007 (7,167 - 5,579) 1,588
Interest Revenue 5,500

Installment Sales 77,545


Cost of Installment Sales 44,020
Deferred Gross Profit - 2008 33,525

Deferred Gross Profit - 2008 (29,500 - 3,912 x 43.33%) 11,062


Deferred Gross Profit - 2007 (26,000 - 1,588 = 24,412 x 50.22%) 11,267
Realized Gross Profit 23,329
33,525/77,545 = 43.23%

Problem 6-10
2005
Jan. 1 Cash 2,000,000
Notes Receivable 5,000,000
Idle Plant 5,000,000
Deferred Gross Profit 2,000,000
AA1 - Chapter 6 (2008 edition)
page 19

2006
July 1 Cash 1,900,000
Notes Receivable 1,000,000
Deferred Gross Profit 900,000

2007
Dec. 31 Cash 2,250,000
Deferred Gross Profit 400,000
Notes Receivable 1,500,000
Interest. Revenue 1,150,000
2,250,000 - (5,000,000 - 2,000,000 - 1,900,000) = 1,150,000

2008
Feb. 1 Cash 2,825,000
Notes Receivable 2,500,000
Interest Revenue 325,000

Feb. 1 Deferred Gross Profit (2,000,000 + 900,000 - 400,000) 2,500,000


Gain on Sale of Idle Plant 2,000,000
Interest Revenue 500,000
(900,000 + 750,000 + 325,000 = 1,975,000 - 1,150,000 - 325,000 = 500,000)

MULTIPLE CHOICE

1. D 2008 2007 2006


Inst. Sales
8,765,625 ÷ 68% P12,890,625
Inst. Rec. beg. P8,387,500 P1,512,500
Inst. Rec. end (9,728,125) (3,025,000) __________
Collections P 3,162,500 P5,362,500 P1,512,500 P10,037,500
GPR x 32% x 30% x 28%
P 1,012,000 P1,608,750 P 423,500 P3,044,250

2 A
3 B
4 C

5 B Deferred gross profit before adjustment P38,000


Deferred gross profit after adjustment
2007 - P16,250 x 30/130 P 3,750
2008 - P90,000 x 33 1/3 /133 1/3 22,500 26,250
Realized gross profit P11,750
Operating expenses 1,500
Net income P10,250

6 B Downpayment (P545,000 x .2) P109,000


Inst. Collections (P545,000 x 8 x .40) 174,400
Collections in year 1 on Year 1 Inst. Sales P283,400
x 35/135
AA1 - Chapter 6 (2008 edition)
page 20

Realized gross profit for Year 1 P 73,474


7 A Installment sales - Year 2 P785,000
Less Collections in Year 2
DP (P785,000 x .20) P157,000
Installment collection (P785,000 x .80 x .40) 251,200 408,200
Balance, end of Year 2 P376,800
x 35/135
Unrealized GP on Year 2 installment sales at the end of Year 2 P 97,689

8 B Inst. Accts. Rec., end of year 3


On year 3 installment sales (P968,000 x .80 x .60) P464,640
On Year 2 installment sales (P785,000 x .80 x .25) 157,000
Total installment accounts receivable, end of Year 3 P621,640

9 B Total installment accounts receivable, end of Year 3 P621,640


x 35/135
Total unrealized gross profit at end of Year 3 P161,166

10 C

11 A 2007 - 100% - (31,250/62,500) = 50% x P25,000 P12,500


2008 - 100% - (45,000/100,000) = 55% x P62,500 34,375
Total P46,875

12 D Sales - regular P187,500


Cost of sales - regular 112,500
Gross profit - regular P 75,000
Realized gross profit (see D1) 46,875
Total gross profit P121,875
Selling expenses 31,250
Net income P 90,625

13 D P610,750 x 60% P366,450


.

14 D P306,520 x 40% P122,608

15 B 2006 sales - P17,400 x 36% P 6,264


2007 sales - P(205,400 - P200 - P25,800) x 39% 69,966
Total P76,230

16 B Selling price P 200


Cost (P200 x 61%) 122
Gain from sale of repossessed merchandise P 78

17 B (P344,460 - P67,440 - P2,200) x 34% P93,438.80

18 B Market value of repossessed ref (P1,700 x 63%) P1,071


Unrecovered cost (P2,200 x 66%) 1,452
Loss on repossession P 381
AA1 - Chapter 6 (2008 edition)
page 21

19 D 2006 sales - 100% - (247/380) = 35% x P24,020 P 8,407.00


2007 sales - see 6-17 93,438.80
2008 sales - 100% - (379,260/602,000 = 37%
37% x (P602,000-P410,090) 71,006.70
Total P172,852.50

20 D ( P100,000 - P12,500 - P6,250) x 50% P40,625

21 B ( 375,000 - 150,000) x 45%* P101,250


*Installment sales P 375,000
Invty. Beg. P 62,500
Purchases 435,000
Repossessions 2,500
Mdse. Avail for sale 500,000
Less Invty. End 75,000
CGS 425,000
Cost of Regular sale ( 312,500 x 70% ) 218,750
Gross profit on Installment sales 206,250
Gross profit rate on Installment sales = 45%
(168,750/375,000)

22 A (3,750 - (6,250 x 50%) P625 loss

23 B 2006 sales - P108,750 x 25% P27,187.50


2007 sales - P120,000 x 27.5% 33,000.00
2008 sales - P 93,750 x 28% 26,250.00
Total P86,437.50

24 A Value assigned to repossessed merchandise:


2006 sales P 9,000
2007 sales 13,500 P22,500
Unrecovered cost
2006 sales - P22,500 x 75% P16,875
2007 sales - P24,000 x 72.5% 17,400 34,275
Loss on repossession P11,775

25 D 2006 sales - P24,000 x 39% P 9,360


2007 sales - (P300,000 - P60,000 - P10,000) x 42% 96,600
2008 sales - (P480,000 - P320,000 - P5,000) x 40% 62,000
Total P167,960

26 B 2007 sales - P4,500 - (P10,000 x 58%) P(1,300)


2008 sales - P3,500 - (P5,000 x 60%) 500
Net gain (loss) on repossession P (800)

27 A P360,000 x 33 1/3% P120,000

28 B Loss = P8,000 - (P15,000 x 60%) P1,000


AA1 - Chapter 6 (2008 edition)
page 22

29 C (P800,000 - P250,000 - P300,000 - P15,000) x 40% P94,000


30 A (P75,810/42%) + P75,810 P256,310

31 D 2006 sales - P35,800 x 32.26% P 11,550


2007 sales - (P155,000 - P42,000) x 40% 45,200
2008 sales - (P256,310 - P100,500) x 42/142 46,085
total P102,835

32 C (P120,000 - P15,000 - P7,750) x 45% P43,762.50

33 C Inventory, Dec. 31, 2007 P 70,000


Purchases 555,000
Repossessions 3,000
Cost of goods available for sale P628,000
Less Inventory, Dec. 31, 2008 95,000
Cost of goods sold P533,000
Less Cost of goods sold on regular sales (P385,000 x 70%) 269,500
Cost of installment sales P263,500
Gross profit rate on installment sales [100% - (263,500/425,000)] 38%
Realized gross profit - (P425,000 - P200,000) x 38% P85,500

34 A. (7,750 x 55%) - 3,000 P1,262.50

35 B Cash sales P 90,000


Charge sales (P180,000/120%) 150,000
Installment sales (P446,400/124%) 360,000
Total sales - cash basis P600,000

36 A Inventory, beginning P52,500


Delivered cost of purchases 393,000
Repossessed merchandise 15,000
Cost of goods available for sale P460,500
Less Inventory, end 70,500
Cost of goods sold P390,000

Cost of installment sales - P390,000 x 360/600 P234,000

37 C 2006 2007 2008


Installment rec’l, beg P 74,000 P123,000
Installment sales P446,400
Installment rec’l, end ( 15,000) ( 45,000) ( 270,000)
Defaulted rec’l ( 18,000) ( 21,000) ---___
Collections P 41,000 P 57,000 P176,400
AA1 - Chapter 6 (2008 edition)
page 23

38 A Sales price of Article “A” P400,000


Less Overvaluation on trade -in
Sales price P110,000
Reconditioning cost ( 8,000)
Normal profit ( 22,000)
Market value of trade-in P 80,000
Allowed trade-in value 120,000 40,000
Adjusted sales price P360,000
Cost of Article “A” 270,000
Gross profit P 90,000
Gross profit rate 25%

Realized GP - (P80,000 + P40,000) x 25% P30,000

39 C Inst. rec’l balance, Dec. 31, 2007 (P360,000 - P120,000) P240,000


Installment payment, Jan. 1 - Mar. 1 (P20,000 x 3) 60,000
Inst. rec’l balance, April 1 P180,000
Cost percentage x 75%
Unrecovered cost P135,000
Market value of repossessed mdse. (P13,500 - P800 - P2,700) 100,000
Loss on repossession P 35,000

40 B Expected loss (125,000 x 4%*) P5,000


Less: Loss on default 2008 P1,500
Recovery from sale of repossessed merchandise 175 1,325
Balance of allowance for defaulted contracts; Dec. 31, 2008 P3,675
*Loss on default in 2007 on 2007 sales P 250
Loss on default in 2008 on 2007 sales 3,750 P 4,000
Recovery on sale of repossession in 2008 P 800
Repossessed Merchandise on hand 200 1,000
Expected loss P 3,000
Sales ÷
P75,000
Rate of loss as a % of sales 4%

41 C Inst. contract rec’l beg. P31,500


Less: Inst. Contract rec’l, end P2,000
Inst. Contract rec’l written off 3,750 5,750
Collections P25,750
Gross profit rate x 40%
Realized gross profit P10,300

42 C Lot A (791,086.20 x 30%) P 237,325.80


Lot B (973,333.30 x 25%) 243,333.30
Lot C and House (P2,080,000 x 40%) 832,000.00
AA1 - Chapter 6 (2008 edition)
page 24

Realized gross profit P1,312,659.10

Schedule of Collection Principal Balance


Lot A
3/31 - Initial balance P1,714,280.00
3/31 - Down payment P514,280.00 1,200,000.00
6/30 - P120,000 - (P1,200,000 x 2.5%) 90,000.00 1,110,000.00
9/30 - P120,000 - (P1,110,000 x 2.5%) 92,250.00 1,017,750.00
12/31- P120,000 - (P1,017,750 x 2.5%) 94,556.20 923,193.80
P791,086.20
Lot B
10/31 – Initial balance P2,400,000.00
10/31 – Down payment P800,000.00 160,000
12/31 – P200,000 – (P1,600,000 x 1.667%) 173,330.30 1,426,666.70
P973,333.30
Lot C
6/30 – Initial balance P4,200,000
6/30 – Down payment P1,800,000.00 2,400,000
12/31- P400,000 – (P2,400,000 x 5%) 280,000.00 2,120,000
P2,080,000.00
Allocation of Cost & GP rates:
Cost SP GP %
Lot A – P1,600,000 x 6/8 P1,200,000 P1,714,280 P514,280 35%
Lot B - P2,400,000 x 6/8 1,800,000 2,400,000 600,000 25%
Lot C: Farm A – P400,000
B – P600,000
House - P1,520,000 2,520,000 4,200,000 1,680,000 40%

43 B Unpaid balance upon default P2,120,000


Less: DGP (P2,120,000 x 40%) 848,000
Unrecovered cost P1,272,000
Market value [P2,520,000 – (P1,520,000 ÷ 20)] 2,444,000
Gain on repossession P1,172,000
44 A 8,000,000 – (8,000,000 x 20%) = 6,400,000 x 3% = P192,000.00
6,400,000 – (642,957.30 – 192,000) x 3% 178,471.30
Collections to be applied on interest P370,471.30
45 D Downpayment P1,600,000.00
1st inst. 642,957.30 - 192,000 450,957.30
2nd inst. 642,957.30 - 192,000 464,486.00
Total collections (P8,000,000 - (2,500,000 + 3,560,000) / P 2,515,443.30
8,000,000]
x 24.25%
Realized gross profit P 609,995.00
CHAPTER 7
SUGGESTED ANSWERS

Exercise 7-1
1. 2006 2007 2008
Contract price P50,000,000 P50,000,000 P50,000,000
Cost incurred to date P 7,500,000 P34,500,000 P40,800,000
Est. cost to complete 30,000,000 8,625,000 -
__________
Total estimated cost 37,500,000 P43,125,000 P40,800,000
Total estimated gross profit P12,500,000 P 6,875,000 P 9,200,000
Percentage of completion 20% 80% 100%

To Date Recognized in prior year/s To be recognized this year


2006 - Recognized revenue P10,000,000 - P10,000,000
Cost of revenue 7,500,000 - 7,500,000
Gross profit P 2,500,000 - P 2,500,000

2007 - Recognized revenue P40,000,000 P10,000,000 P30,000,000


Cost of revenue 34,500,000 7,500,000 27,000,000
Gross profit P 5,500,000 P 2,500,000 P 3,000,000

2008 - Recognized revenue P50,000,000 P40,000,000 P10,000,000


Cost of revenue 40,800,000 34,500,000 6,300,000
Gross profit P 9,200,000 P 5,500,000 P 3,700,000

2. 2006 2007 2008


a. Construction in progress 7,500,000 27,000,000 6,300,000
Cash, Materials, etc. 7,500,000 27,000,000 6,300,000

b. Accounts Receivable 8,000,000 36,000,000 6,000,000


Progress Billings on Const. Contracts 8,000,000 36,000,000 6,000,000
AA1 - Chapter 7 (2008 edition) page 2

c. Cash 5,500,000 33,000,000 11,500,000


Accounts Receivable 5,500,000 33,000,000 11,500,000

d. Cost of LTCC 7,500,000 27,000,000 6,300,000


Construction in Progress 2,500,000 3,000,000 3,700,000
Revenue from LTCC 10,000,000 30,000,000 10,000,000

e. Progress Billings on
Construction Contracts 50,000,000
Construction In Progress 50,000,000

3.
Statement of Financial Position
Current Assets:
Accounts Receivable P5,500,000

Current Liabilities:
Progress Billings on Construction Contracts P44,000,000
Less Construction in Progress 40,000,000 P4,000,000

Exercise 7-2
2006 2007 2008
a. Construction in Progress 32,000,000 43,000,000 15,500,000
Cash, Materials, etc. 32,000,000 43,000,000 15,500,000

b. Accounts Receivable 33,000,000 45,000,000 22,000,000


Progress Billing on Const. Contract 33,000,000 45,000,000 22,000,000

c. Cash 31,000,000 40,000,000 29,000,000


Accounts Receivable 31,000,000 40,000,000 29,000,000

Cost of LTCC 23,000,000 45,250,000 22,250,000


Construction in Progress 2,000,000 4,750,000 2,750,000
Revenue from LTCC 25,000,000 50,000,000 25,000,000
AA1 - Chapter 7 (2008 edition) page 3

e. Progress Billing on Const. Contracts 100,000,000


Construction in Progress 100,000,000

2006 2007 2008


Contract price P100,000,000 P100,000,000 P100,000,000
Cost incurred to date P 32,000,000 P 75,000,000 P 90,500,000
Estimated cost to complete 60,000,000 16,000,000 ___________
Total estimated cost P 92,000,000 P 91,000,000 P 90,500,000
Total estimated gross profit P 8,000,000 P 9,000,000 P 9,500,000
Percentage of completion 25% 75% 100%

To date Recognized in prior year/s To be recognized this year


2006 - Recognized revenue P25,000,000 - P25,000,000
Cost of revenue 23,000,000 - 23,000,000
Gross profit P 2,000,000 - P 2,000,000

2007 - Recognized revenue P75,000,000 P25,000,000 P50,000,000


Cost of revenue 68,250,000 23,000,000 45,250,000
Gross profit P 6,750,000 P 2,000,000 P 4,750,000

2008 - Recognized revenue P100,000,000 P75,000,000 P25,000,000


Cost of revenue 90,500,000 68,250,000 22,250,000
Gross profit P 9,500,000 P 6,750,000 P 2,750,000

Exercise 7-3

1. Contract price P25,000,000


Total estimated cost:
Cost incurred to date P 4,400,000
Estimated cost to complete 15,600,000 20,000,000
Total estimated gross profit P 5,000,000
Percentage of completion ( P 400,000/20,000,000) 22%
Gross profit to be recognized in 2008 P 1,100,000
AA1 - Chapter 7 (2008 edition) page 4

2. Accounts Receivable (P25,000,000 x 30% x 10%) P 750,000


Construction in Progress (P4,400,000 + P1,100,000) P5,500,000
Progress Billings on Construction Contracts (P25,000,000 x 30%) P7,500,000

Exercise 7-4
2006 2007 2008
Contract price P35,000,000 P35,000,000 P35,000,000
Cost incurred to date P17,500,000 P29,250,000 P31,000,000
Estimated cost to complete 10,500,000 3,250,000 -
Total estimated cost P28,000,000 P32,500,000 P31,000,000
Total estimated gross profit P 7,000,000 P 2,500,000 P 4,000,000
Percentage of completion 62.5% 90% 100%

To date Recognized in prior year/s To be recognized this year


2006 - Recognized revenue P21,875,000 - P21,875,000
Cost of revenue 17,500,000 - 17,500,000
Gross profit P 4,375,000 P 4,375,000

2007 - Recognized revenue P31,500,000 P21,875,000 P 9,625,000


Cost of revenue 29,250,000 17,500,000 11,750,000
Gross profit P 2,250,000 P 4,375,000 P(2,125,000)

2008 - Recognized revenue P35,000,000 P31,500,000 P 3,500,000


Cost of revenue 31,000,000 29,250,000 1,750,000
Gross profit P 4,000,000 P 2,250,000 P 1,750,000
2. Journal entries
2006 2007 2008
a. Construction in Progress 17,500,000 11,750,000 1,750,000
Cash, Materials, etc. 17,500,000 11,750,000 1,750,000

b. Accounts Receivable 16,000,000 12,000,000 7,000,000


Progress Billing on
Const. Contracts 16,000,000 12,000,000 7,000,000
AA1 - Chapter 7 (2008 edition) page 5

c. Cash 15,000,000 10,000,000 10,000,000


Accounts Receivable 15,000,000 10,000,000 10,000,000

d. Cost of LTCC 17,500,000 11,750,000 1,750,000


Construction in Progress 4,375,000 1,750,000
Construction in Progress 2,125,000
Rev. from LTCC 21,875,000 9,625,000 3,500,000

e. Progress Billing on Const. Contract 35,000,000


Construction in Progress 35,000,000

3. To date Recognized in prior year/s To be recognized this year


2006 - Recognized revenue P17,500,000 - P17,500,000
Cost of revenue 17,500,000 - 17,500,000
Gross profit - - -
========== ============ ==========

2007 - Recognized revenue P31,500,000 P17,500,000 P14,000,000


Cost of revenue 29,250,000 17,500,000 11,750,000
Gross profit P 2,250,000 ---------------- P 2,250,000

2008 - Recognized revenue P35,000,000 P31,500,000 P3,500,000


Cost of revenue 31,000,000 29,250,000 1,750,000
Gross profit P 4,000,000 P 2,250,000 P1,750,000

Exercise 7-5
Revenue recognized in 2008 (P26,000,000 x 40%) P10,400,000
Gross profit/income recognized in 2008 (P3,120,000 - P1,300,000) 1,820,000
Cost incurred in 2008 P 8,580,000
AA1 - Chapter 7 (2008 edition) page 6

Exercise 7-6
Binondo Project Pasig Project
Revenue (CP x % of work done in 2007) P12,000,000 P1,290,000
Cost of revenue 12,400,000 1,400,000
Gross profit (loss) P (400,000) P( 110,000)

Exercise 7-7
1. Contract revenue/price P10,000,000
Less Total profit 800,000
Total cost incurred P 9,200,000
Less Cost incurred in 2006 and 2008 5,900,000
Cost incurred in 2007 P 3,300,000

2. Gross profit to date, 12.31.07 P 900,000


Cost incurred to date, 12.31.07 (P1,800,000 + P3,300,000) 5,100,000
Revenue to date, 12.31.07 P6,000,000
Percentage-of-completion (6,000,000/10,000,000) 60%

3. Gross profit to date, 12.31.07 P 900,000


Percentage of completion ÷ 60%
Total estimated gross profit P1,500,000

4. Contract price P10,000,000


Less Total estimated gross profit 1,500,000
Total estimated cost P 8,500,000
Less Cost incurred to date 5,100,000
Estimated cost to complete P 3,400,000

Exercise 7-8
Cash 500,000
Notes Receivable 1,000,000
Discount on Notes Receivable 207,540
Unearned Franchise Fees 1,292,460
AA1 - Chapter 7 (2008 edition) page 7

Exercise 7-9
1. Cash 4,000,000
Notes Receivable 3,000,000
Discount on Notes Receivable 513,200
Unearned Franchise Fees 6,486,800

2. Cash 4,000,000
Notes Receivable 3,000,000
Discount on Notes Receivable
(3,000,000-(2.48685 x 1,000,000) 513,200
Revenue from Franchise Fees 6,486,800

3. Cash 4,000,000
Unearned Franchise Fees 4,000,000

4. Cash 4,000,000
Notes Receivable 3,000,000
Discount on Notes Receivable 513,200
Revenue from Franchise Fees 4,000,000
Unearned Franchise Fees (1,000,000 x 2.48685) 2,486,800

Exercise 7-10
2007
July 1 - Cash 1,200,000
Notes Receivable 3,200,000
Discount on Notes Receivable 644,100
Unearned Franchise Fee 3,735,900
P800,000 x 3.1699 = P2,535,900
P3,200,000 - P2,535,900 = P664,100

Sept. 1 - Deferred Franchise Cost 100,000


Cash 100,000

Nov. 15 - Deferred Franchise Cost 60,000


Cash 60,000
AA1 - Chapter 7 (2008 edition) page 8

Dec. 31 - Discount on Notes Receivable 126,795


Interest Revenue 126,795
P2,535,900 x 10% x 6/12 = P126,795

2008
Jan. 10 - Deferred Franchise Cost 100,000
Cash 100,000

15 - Unearned Franchise Fee 3,735,900


Franchise Fee Revenue 3,735,900

15 - Cost of Franchise Fee Revenue 260,000


Deferred Franchise Cost 260,000

July 1 - Cash 800,000


Notes Receivable 800,000

1 - Discount on Notes Receivable 126,795


Interest Revenue 126,795

Problem 7-1
2007 2008
a. Construction in Progress 11,000,000 4,800,000
Cash, Materials, etc. 11,000,000 4,800,000

b. Accounts Receivable 10,800,000 9,200,000


Progress Billing on Const. Contract 10,800,000 9,200,000

c. Cash 10,000,000 10,000,000


Accounts Receivable 10,000,000 10,000,000

d. Cost of LTCC 11,000,000 4,800,000


Construction in Progress 2,750,000 1,450,000
Revenue from LTCC 13,750,000 6,250,000
AA1 - Chapter 7 (2008 edition) page 9

e. Progress Billing on Construction Contracts 20,000,000


Construction in Progress 20,000,000

Problem 7-2
Statement of Recognized Income and Expenses:
Income: 2007 P2,750,000
2008 1,450,000

Statement of Financial Position


Receivable: 2007 P 800,000
2008 -

Inventory - CIP, net of billings


2007 (13,750,000 - 10,800,000) P2,950,000
2008 -

Problem 7-3
Year Income (loss) Recognized Rec’l ending balance CIP Invty. ending balance Cost in excess of billings
2006 1,000,000 380,000 5,000,000 1,200,000
2007 1,000,000 940,000 12,000,000 2,600,000
2008 1,000,000 - - -

2006 2007 2008


Contract price P15,000,000 P15,000,000 P15,000,000
Cost incurred to date P 4,000,000 P10,000,000 P12,000,000
Estimated cost to complete 8,000,000 2,500,000 ----------------
Total estimated cost P12,000,000 P12,500,000 P12,000,000
Total estimated gross profit P 3,000,000 P 2,500,000 P 3,000,000
Percentage of completion 33 1/3% 80% 100%
Gross profit to date P 1,000,000 P 2,000,000 P 3,000,000
Less Gross profit recognized in prior year/s _____-______ 1,000,000 2,000,000
Gross profit to be recognized this year P 1,000,000 P 1,000,000 P 1,000,000
AA1 - Chapter 7 (2008 edition) page 10

Problem 7-4
PROJECT A PROJECT B PROJECT C PROJECT D
2007 2008 2007 2008 2007 2008 2008
Contract price P29,000,000 P29,000,000 P34,000,000 P34,000,000 P17,000,000 P17,000,000 P2,000,000
Cost incurred to date P16,800,000 P26,400,000 P14,400,000 P21,200,000 P 3,200,000 P11,830,000 P 5,600,000
Estimated cost to complete 11,200,000 ------------- 17,600,000 13,000,000 9,600,000 1,170,000 10,400,000
Total estimated cost P28,000,000 P26,400,000 P32,000,000 P34,200,000 P12,800,000 P13,000,000 P16,000,000
Total estimated gross profit (loss) P 1,000,000 P 2,600,000 P 2,000,000 P( 200,000) P 4,200,000 P 4,000,000 P 4,000,000
Percentage of completion 60% 100% 45% 25% 91% 35%
Gross profit (loss) to date P 600,000 P 2,600,000 P 900,000 P( 200,000)* P 1,050,000 P 3,640,000 P 1,400,000
Less gross profit recognized in prior year ------- 600,000 ------ 900,000 ---- 1,050,000 -----
Gross profit - current year P 600,000 P 1,000,000 P 900,000 P(1,100,000) P 1,050,000 P 2,590,000 P 1,400,000

* The entire loss should be recognized immediately

(1) Percentage of completion method

2007 2008
Gross profit P2,550,000 P3,890,000
Operating expenses 1,200,000 1,200,000
Net income P1,350,000 P2,690,000

Problem 7-5
1. (a) 2006 2007 2008 2009
Contract price P120,000,000 P120,000,000 P120,000,000 P120,000,000
Cost incurred to date P 24,000,000 P60,500,000 P90,000,000 P105,000,000
Estimated cost to complete 76,000,000 49,500,000 10,000,000 --------
Total estimated cost P100,000,000 P110,000,000 P100,000,000 P105,000,000
Total estimated gross profit P 20,000,000 P 10,000,000 P 20,000,000 P 15,000,000

Percentage of completion 24% 55% 90% 100%


AA1 - Chapter 7 (2008 edition) page 11

Recognized in To be
recognized
To date prior year in current year
2006-Revenue P28,800,000 ------ P28,800,000
Cost of revenue 24,000,000 ------ 24,000,000
Gross profit P 4,800,000 ------ P 4,800,000
To be
Recognized in recognized
To date prior year in current year
2007-Revenue P66,000,000 P28,800,000 P37,200,000
Cost of revenue 60,500,000 24,000,000 36,500,000
Gross profit P 5,500,000 P 4,800,000 P 700,000

2008-Revenue P108,000,000 P66,000,000 P42,000,000


Cost of revenue 90,000,000 60,500,000 29,500,000
Gross profit P 18,000,000 P 5,500,000 P12,500,000
To be
Recognized in recognized
To date prior year in current year
2009-Revenue P120,000,000 P108,000,000 P 12,000,000
Cost of revenue 105,000,000 90,000,000 15,000,000
Gross profit P 15,000,000 P 18,000,000 P( 3,000,000)

2. 2006 2007 2008 2009


a Construction in Progress 24,000,000 36,500,000 29,500,000 15,000,000
Cash, Materials, etc. 24,000,000 36,500,000 29,500,000 15,000,000

b. Accounts Receivable 26,000,000 31,000,000 34,000,000 29,000,000


Progress Billings on Const. Contract 26,000,000 31,000,000 34,000,000 29,000,000

c. Cash 24,000,000 27,000,000 30,000,000 30,000,000


Accounts Receivable 24,000,000 27,000,000 30,000,000 30,000,000
AA1 - Chapter 7 (2008 edition) page 12

d. Cost of LTCC 24,000,000 36,500,000 29,500,000 15,000,000


Construction in Progress 4,800,000 700,000 12,500,000 3,000,000
Construction in Progress 12,000,000
Revenue from LTCC 28,800,000 37,200,000 42,000,000

e. Progress Billings on Const. Contracts 120,000,000


Construction in Progress 120,000,000

Problem 7-6
2006 2007 2008
Contract price P14,000,000 P14,000,000 P13,000.000
Cost incurred to date P 5,000,000 P11,475,000 P12,295,000
Estimated cost to complete 7,500,000 1,275,000 -------
Total estimated cost P12,500,000 P12,750,000 P12,295,000
Total estimated gross profit P 1,500,000 P 1,250,000 P 705,000
Percentage of completion 40% 90% 100%
Gross profit to date P 600,000 P 1,125,000 P 705,000
Less Gross profit recognized in prior year ------ 600,000 1,125,000
Gross profit - current year P 600,000 P 525,000 P (420,000)

Problem 7-7
2006 2007 2008 Total
1. Recognized revenue P 1,100,000 P1,300,000 (2) P1,100,000 (3) P3,500,000
Cost of revenue 1,000,000 1,250,000 1,150,000 (4) 3,400,000 –(5)
Gross Profit (loss) P 100,000 – (1) P 50,000 P (50,000) P 100,000
2006 2007
2. Contract-price P3,500,000 P3,500,000
Cost incurred to date 1,000,000 2,250,000
Estimated cost to complete P2,250,000 P 950,000
Total estimated cost P3,250,000 P3,200,000
Total estimated gross profit 250,000 300,000
Percentage of completion 30.77% 70.3125%
Gross profit to date P 76,925 P 210,938
Less GP recognized in prior year/s - 76,925
GP to be recognized this year P 76,925 P 134,013
AA1 - Chapter 7 (2008 edition) page 13

Problem 7-8
Franchise A:
The circumstances imply that the full accrual method could be used.
Franchise revenue P3,578,000*
Franchise cost 1,400,000 P2,178,000
Interest revenue (P2,178,000 x 4%) 87,200
Income from Franchise A P2,265,200

*Initial deposit P 1,400,000


PV of four payments [4% for 4 periods
(P600,000 x 3.6299)] 2,178,000
P 3,578,000

Franchise B:
Because of the doubtful collection and only partial completion, the deposit method should be used. No revenue or income would be
recognized in 2008 from the franchise fee. However, because the first payment of P600,000 was made, interest revenue of P87,200 would
be recognized.

Franchise C:
Because of the doubtful collection but substantial completion, either the installment sales or cost recovery method could be used. If the
installment sales method is used, gross profit of P843,600* would be recognized in 2008 plus interest revenue of P87,200.

*Franchise revenue P3,578,000


Franchise cost 2,000,000
Franchise gross profit P1,578,000

Gross profit percentage: P1,578,000 ÷ P3,578,000 44.1%

Collections in 2008:
Initial fee P1,400,000
First payment:
Interest P 87,200
Principal 512,800 512,800
Total P 600,000 P1,912,800
Gross profit recognized – 2008: P1,912,800 x 44.1% = P843,600
AA1 - Chapter 7 (2008 edition) page 14

If the cost recovery method is used, no revenue or income would be recognized, because the P2,000,000 collections are exactly offset by
the P2,000,000 costs.

Problem 7-9
2007
July 1 Cash 7,000,000
Notes Receivable 8,000,000
Unearned Franchise Fee 15,000,000

Aug. 15 Deferred Franchise Cost 800,000


Cash 800,000

Sept. 15 Deferred Franchise Cost 500,000


Cash 500,000

Dec. 31 Interest Receivable 400,000


Interest Revenue 400,000

2008
Jan. 1 Cash 2,400,000
Notes Receivable 2,000,000
Interest Receivable 400,000

15 Deferred Franchise Cost 1,000,000


Cash 1,000,000

31 Unearned Franchise Fee 15,000,000


Cost of Franchise Revenue 2,300,000
Franchise Fee Revenue 15,000,000
Deferred Franchise Cost 2,300,000

July 1 Cash 2,300,000


Notes Receivable 2,000,000
Interest Revenue 300,000
P6,000,000 x 10% x 6/12
AA1 - Chapter 7 (2008 edition) page 15

Dec. 31 Interest Receivable 200,000


Interest Revenue 200,000

Problem 7-10
1. Downpayment made on 1/1/ 07 P 800,000.00
Present value of an ordinary annuity (P240,000 x 3.69590) 887,016.00
Total revenue recorded by Triple Eight P1,687.016.00

2. Cost of acquisition P 1,687,016

3. Cash 800,000.00
Notes Receivable 1,200,000.00
Discount on Notes Receivable 312,984.00
Unearned Franchise Fees 1,687,016.00

4. a. P800,000 cash received from downpayment. (P887,016.00 is recorded as unearned revenue from franchise fees).
b. P800,000 cash received from downpayment
c. None. (P 800,000 is recorded as unearned revenue from Franchise fees).

MULTIPLE CHOICE

1. C 6. B
2. B 7. D
3. D 8. D
4. A 9. D
5. C 10. C

11. D P20,000,000 x (3,000,000/15,000,000) = P4,000,000


AA1 - Chapter 7 (2008 edition) page 16

12. A Contract price P10,500,000


Less Total estimated cost:
Cost incurred to date P3,150,000
Est. cost to complete 6,300,000 9,450,000
Total estimated income P 1,050,000
% of completion (3150/9450) 33 1/3%
Income to be recognized in 2007 P 350,000

13. B Contract price P9,000,000


Total estimated cost 8,100,000
Total estimated income P 900,000
Percentage-of-completion (27/81) 33 1/3%
Income recognized last year P 300,000

14. C Contract price P15,000,000


Total estimated cost (P4,650,000 + P10,850,000) 15,500,000
Total estimated loss – to be recognized in full P 500,000

15 B Contract price P14,000,000


Total estimated cost (P4M + P4M + P2M) 10,000,000
Total estimated gross profit P 4,000,000
Percentage-of-completion (8M/10M) 80%

Gross profit to date P 3,200,000


Less Gross profit recognized in 2006
(P14M –P8M = P6M x 4/8) 3,000,000
Gross profit to be recognized in 2007 P 200,000

16 B Contract price P3,000,000


Total estimated cost 1,800,000
Total estimated gross profit P1,200,000
Percentage-of-completion (600/1,800) 33 1/3%
Gross profit to be recognized in 2007 P 400,000
AA1 - Chapter 7 (2008 edition) page 17

17. B Contract price P12,000,000


Total cost incurred 10,800,000
Gross profit P 1,200,000
Gross profit percentage (1,200/12,000) 10%

18 A Cubao Marikina
Contract price P16,200,000 P25,200,000
Total estimated cost 14,400,000 23,100,000
Total est. gross profit P 1,800,000 P 2,100,000
Percentage-of-completion 83 1/3% 100%
Gross profit to date P 1,500,000 P 2,100,000
Less GP recognized in 2007 750,000 1,872,000
GP to be recognized In 2008 P 750,000 P 228,000

Total GP = P750,000 + P228,000 P 978,000

19. 20,000,000/24,000,000 83.33%

20 Contract price P30,000,000


Total estimated cost 24,000,000
Total estimated gross profit P 6,000,000
Percentage-of-completion 83.33%
GP to date P 5,000,000
GP recognized in prior years
(P30M - P22M = P8M x 50%) 4,000,000
GP to be recognized in 2008 P 1,000,000

21. Total amount billed P843,750


Less Balance of accounts receivable 300,000
Total collections P543,750
Amount deposited 500,000
Cash collected not yet deposited P 43,750

22 P150,000 ÷ 937,500/9,000,000 P1,440,000


AA1 - Chapter 7 (2008 edition) page 18

23 C Mobilization fee (P1.2B x 1%) P 1.2M


Collections on billings (1.2B x 10% x 90%) 10.8M
Total fee received by NNO P12.0M

24 B Contract price P100.00M


Gross profit rate 25%
Total estimated gross profit P25.00M
Percentage-of-completion 50%
Realized gross profit P12.50M

25. B
26 A
27 B
28 C
29

30 A Downpayment P 50,000
First installment payment 50,000
Add’l fee (P1,000,000 x 3%) 30,000
Earned Franchise Fees P130,000

31 C P 100,000 x 1/5 = P 20,000 + 1% of P500,000 = P 25,000


32 A P 1,000,000 + 5% of P8,000,000 = P1,400,000

33 C Downpayment P 100,000
PV of installment payment 199,650
Additional fee ( P 9,000,000 x 5% ) 450,000
Earned franchise fee P 749,650
CHAPTER 8
SUGGESTED ANSWERS

EXERCISES
Exercise 8 - 1

(a) Working Fund - Agency 5,000


Cash 5,000

(b) Accounts Receivable - Agency 50,000


Sales - Agency 50,000

(c) Cash 35,000


Accounts Receivable - Agency 35,000

(d) Expenses - Agency 4,500


Cash 4,500

(e) Expenses - Agency 2,250


Cash 2,250

(f) Cost of Sales - Agency 36,000


Shipments to Agency 36,000

Exercise 8- 2

(a) Working Fund - Makati Agency 10,000


Samples - Makati Agency 60,000
Cash 10,000
Shipments to Makati Agency 60,000

(b) Accounts Receivable - Makati Agency 200,000


Sales - Makati Agency 200,000

(c) Cost of Sales - Makati Agency 116,000


Shipments to Makati Agency 116,000

(d) Salaries and Commission - Makati Agency 20,000


Furniture and Fixtures - Makati Agency 45,000
Cash 65,000

(e) Delivery Expense - Makati Agency 6,200


Office Supplies Expense - Makati Agency 1,000
Cash 7,200

(f) Samples Expense - Makati Agency 25,000


Samples - Makati Agency 25,000
AA1- Chapter 8 (2008 edition) page
2

(g) Office Supplies - Makati Agency 300


Office Supplies Expense 300

(h) Depreciation Expense - Makati Agency 1,250


Accumulated Depr. - Furniture & Fixtures 1,250

(I) Sales - Makati Agency 200,000


Agency Income 30,850
Cost of Sales - Makati Agency 116,000
Salaries and Commission - Makati Agency 20,000
Delivery Expense - Makati Agency 6,200
Samples Expense - Makati Agency 25,000
Office Supplies Expense - Makati Agency 700
Depreciation Expense - Makati Agency 1,250

(j) Agency Income 30,850


Income Summary 30,850

Exercise 8-3
Home Office Books

(a) Cash 20,000


Iloilo Branch 20,000

(b) Iloilo Branch 3,500


Cash 3,500

(c) Expenses 1,200


Iloilo Branch 1,200

(d) Iloilo Branch 32,000


Shipments to Iloilo Branch 32,000

(e) Shipments to Iloilo Branch 15,000


Iloilo Branch 15,000

(f) Furniture and Fixtures 5,500


Cash 5,500

(g) Iloilo Branch 550


Accumulated Depr. - Furniture & Fixtures 550

(h) Branch Income 2,600


Branch 2,600
AA1- Chapter 8 (2008 edition) page
3

Branch Office Books

(a) Home Office 20,000


Cash 20,000

(b) Expenses 3,500


Home office 3,500

(c) Home Office 1,200


Cash 1,200

(d) Shipments from Home Office 32,000


Home Office 32,000

(e) Home Office 15,000


Shipments from Home Office 15,000

(f) Memo entry

(g) Depreciation Expense 550


Home Office 550

(h) Home Office 2,600


Income Summary 2,600

Exercise 8-4
Home Office Books

(a)
(1) Dagupan Branch 300,000
Shipments to Branch 300,000

(5) Cash 150,000


Dagupan Branch 150,000

Branch Office Books

(1) Shipments from Home Office 300,000


Home Office 300,000

(2) Accounts Receivable 390,000


Sales 390,000

(3) Expenses 74,000


Cash 72,000
Accrued Expenses 2,000
AA1- Chapter 8 (2008 edition) page
4

(4) Cash 288,000


Sales Discount 6,000
Accounts Receivable 294,000

(5) Home Office 150,000


Cash 150,000

(6) Furniture and Fixtures 40,000


Cash 40,000

(7) Expenses 8,000


Accumulated Depr. - Furniture & Fixtures 8,000

(b)
Merchandise Inventory 60,000
Sales 390,000
Shipments from Home Office 300,000
Expenses 82,000
Sales Discounts 6,000
Income Summary 62,000

Income Summary 62,000


Home Office 62,000

(c)
Honda Sales, Inc.
Statement of Recognized Income and Expenses - Branch
For the Year Ended December 31,2008

Sales P 390,000
Less: Sales Discount 6,000 P384,000
Cost of sales:
Shipment to Home Office P 300,000
Less: Inventory, end 60,000 240,000
Gross Profit P 144,000
Expenses 82,000
Net Profit P 62,000
(d)
Honda Sales, Inc.
Statement of Financial Position - Branch
December 31,2008

Assets

Cash (-720,00 + 288,000 - 150,000 - 40,000) P 26,000


Accounts Receivable (390,000 – 294,000) 96,000
Merchandise Inventory 60,000
Furniture and Fixture P 40,000
Less: Accumulated Depreciation 8,000 32,000
Total Assets P 214,000
AA1- Chapter 8 (2008 edition) page
5

Liabilities

Accrued Expenses P 2,000


Home Office (300,000 - 150,000 + 62,000) 212,000
Total Liabilities P 214,000

Exercise 8- 5

Home Office Books

(a) Furniture and Fixtures 24,500


Branch 24,500

(b) Branch 24,500


Cash 24,500

(c) Branch 2,450


Accumulated Depr. - Furniture & Fixtures 2,450

(d) Furniture & Fixtures - new 40,000


Accumulated Depr. - Furniture & Fixtures 2,450
Branch Income 7,050
Furniture & Fixtures - old 24,500
Cash 25,000
Branch Office Books

(a) Home Office 24,500


Accounts Payable 24,500

(b) Accounts Payable 24,500


Home Office 24,500

(c) Depreciation Expense 2,450


Home Office 2,450

(d) Memo entry

Exercise 8-6

Jan. 10 Notes Payable 2,500


Home Office 2,500

10 Furniture and Fixtures 10,000


Home Office 10,000

16 Shipments from Home Office 6,500


Home Office 6,500
AA1- Chapter 8 (2008 edition) page
6

16 Home Office 2,000


Cash 2,000

20 Home Office 1,200


Shipments from Home Office 1,200

25 Home Office 150


Accounts Receivable 150

30 Expenses 800
Home office 800

31 Home Office 750


Income Summary 750

Exercise 8 -7
Honda Company
Reconciliation of Home Office and Branch Accounts
December 31,2008

HO Books Branch
Books
Branch Acct. HO Acct.
Unadjusted balances P 8,400 P 9,735
Adjustments;
(a) Merchandise in transit 615
(b) Collection of home office accounts rec’l 2,500
(c) Error in recording the net income of branch
(P1,215 - P1,125) 90
(d) Merchandise returned by branch still in
transit ( 640) _______
Adjusted balances P 10,350 P 10,350

Requirement 2

Home Office Books

(b) Branch 2,500


Accounts Receivable 2,500

(c) Branch 90
Branch Income 90

(d) Shipments to Branch 640


Branch 640

Branch Books

(a) Shipments from Home Office 615


Home Office 615
AA1- Chapter 8 (2008 edition) page
7

Exercise 8-8
Home Office Books

(a) Allowance for Doubtful Accounts 600


Makati Branch 600

(b) Makati Branch 1,250


General and Administrative Expense 1,250

(c) Manila Branch 1,200


Makati Branch 1,200

(d) Makati Branch 850


Allowance for Uncollectible Accounts 850

Branch Books

(a) Home Office 600


Accounts Receivable 600

(b) General and Administrative Expenses 1,250


Home Office 1,250

(c) No entry

(d) Uncollectible Accounts Expense 850


Home office 850

PROBLEMS
Problem 8-1

(a) Samples - Cebu Agency 60,000


Advertising Materials - Cebu Agency 35,000
Shipments to Cebu Agency 60,000
Advertising Materials 35,000

(b) Working Fund - Cebu Agency 30,000


Cash 30,000

(c) Accounts Receivable - Cebu Agency 330,000


Sales - Cebu Agency 330,000

Cost of Sales - Cebu Agency 250,000


Shipments to Cebu Agency 250,000
AA1- Chapter 8 (2008 edition) page
8

(d) Cash 245,000


Sales Discounts - Cebu Agency 5,000
Accounts Receivable - Cebu Agency 250,000

(e) Rent Expense - Cebu Agency 15,000


Delivery Expense - Cebu Agency 3,000
Repairs and Maintenance - Cebu Agency 2,200
Cash 20,200

(f) Salaries and Wages - Cebu Agency 10,200


Commission Expense - Cebu Agency 33,000
Salaries and Wages 10,200
Commission Expense 33,000

(g) Samples Expense - Cebu Agency 15,000


Samples - Cebu Agency 15,000

(g) Advertising Materials Expense - Cebu Agency 10,500


Advertising Materials - Cebu Agency 10,500

(h) Sales - Cebu Agency 330,000


Agency Income 13,900
Cost of Sales - Cebu Agency 250,000
Sales Discounts - Cebu Agency 5,000
Rent Expense - Cebu Agency 15,000
Delivery Expense - Cebu Agency 3,000
Repairs and Maintenance - Cebu Agency 2,200
Salaries and Wages - Cebu Agency 10,200
Commission Expense - Cebu Agency 33,000
Samples Expense - Cebu Agency 15,000
Advertising Materials Expense - Cebu Agency 10,500

(I) Income Summary 13,900


Agency Income 13,900

Problem 8- 2
Home Office Books

(a) Branch 150,000


Cash 150,000

(b) Branch 90,000


Shipments to Branch 90,000

(c) No entry

(d) Branch 90,000


Cash 3,000
Equipment 75,000
Gain on Transfer of Equipment 12,000
AA1- Chapter 8 (2008 edition) page
9

(e) No entry

(f) Expenses 7,500


Cash 7,500

(g) and (h) no entry

(I) Cash 52,500


Branch 52,500

Branch Books

(a) Cash 150,000


Home Office 150,000

(b) Shipments from Home Office 90,000


Home Office 90,000

(c) Purchases 120,000


Cash 120,000

(d) Equipment 90,000


Home Office 90,000

(e) Accounts Receivable 250,000


Sales 250,000

Cash 100,000
Accounts Receivable 100,000

(f) No entry

(g) Depreciation Expense - Equipment 9,000


Accumulated Depr. - Equipment 9,000

(h) Selling and Administrative Expenses 45,000


Cash 45,000

(I) Home Office 52,500


Cash 52,500

Problem 8-3

Requirement 1

a. Accounts Receivable 80,000


Sales 80,000
AA1- Chapter 8 (2008 edition) page
10

b. Purchases 21,000
Accounts Payable 21,000

c. Shipments from Home Office 40,000


Home office 40,000

d. Cash 76,000
Accounts Receivable 76,000

e. Accounts Payable 20,200


Cash 20,200

f. Allowance for Uncollectible Accounts 1,200


Accounts Receivable 1,200

g. Home Office 30,000


Cash 30,000

h. Expenses 24,800
Cash 24,800

I. Expenses 1,600
Home Office 1,600

j. Prepaid Expenses 200


Accrued Expenses 400
Expenses 1,700
Allowance for Doubtful Accounts 1,100
Accumulated Depreciation 1,200

k. Merchandise Inventory, end 38,800


Sales 80,000
Income Summary 3,300
Merchandise Inventory, beg. 33,000
Shipments from Home Office 40,000
Purchases 21,000
Expenses 28,100

l. Home Office 3,300


Income Summary 3,300
AA1- Chapter 8 (2008 edition) page
11

Requirement 2
Volvo Company
Statement of Recognized Income and Expenses - Branch
For the Year Ended December 31, 2008

Sales P80,000
Cost of goods sold:
Inventory, January 1 P33,000
Purchases 21,000
Shipments from home office 40,000
Cost of goods available for sale P94,000
Less Inventory, December 31 38,800 55,200
Gross profit P24,800
Operating expenses 28,100
Net loss P 3,300

Volvo Company
Statement of Financial Position - Branch
December 31, 2008

Assets

Cash (7,000 + 76,000 + 20,200 - 30,000 - 24,800) P 8,000


Accounts Receivable (24,400 + 80,000 - 76,000 -1,200) P27,200
Less Allowance for Uncollectible Accounts 1,600 25,600
Merchandise Inventory 38,800
Prepaid Expenses 900
Furniture and Fixtures P 7,700
Less Accumulated Depreciation 6,600 1,100
Total Assets P74,400
Liabilities

Accounts Payable (4,000 + 21,000 - 20,200) P 4,800


Accrued Expenses 800
Home Office (60,500 + 40,000 - 30,000 + 1,600 - 3,300) 68,800
Total Liabilities P74,400

Volvo Company
Statement of Changes in Home Office Account
For the Year Ended December 31, 2008

Home office account balance, January 1 P 60,500


Add: Shipments from home office P 40,000
Expenses paid by home office 1,600 41,600
Total P102,100
Deduct: Remittance to home office P 30,000
Net loss 3,300 33,300
Home office account balance, December 31 P 68,800
AA1- Chapter 8 (2008 edition) page
12

Requirement 3

c. Davao Branch 40,000


Shipments to Branch 40,000

g. Cash 30,000
Davao Branch 30,000

i. Davao Branch 1,600


Cash 1,600

Branch Income 3,300


Davao Branch 3,300

Problem 8-4

Requirement 1
Branch Books

a. Cash 15,000
Shipments from Home Office 102,000
Accounts Receivable 26,000
Home Office 143,000

Home Office 9,000


Cash 9,000

b. Accounts Receivable 62,000


Sales 62,000

c. Cash 26,000
Accounts Receivable 26,000

d. Purchases 30,000
Accounts Payable 30,000

e. Accounts Payable 14,500


Cash 14,500

f. Expenses 12,500
Cash 12,500

g. Cash 16,000
Home Office 1,500
Accounts Receivable 17,500

h. Shipments from Home Office 12,500


Home Office 12,500
AA1- Chapter 8 (2008 edition) page
13

I. Home Office 10,000


Cash 10,000

Home Office Books

a. Bacolod Branch 143,000


Cash 15,000
Shipments to Branch 102,000
Accounts Receivable 26,000

Store Furniture and Fixtures 9,000


Bacolod Branch 9,000

b. Accounts Receivable 346,000


Sales 346,000

c. Cash 400,000
Accounts Receivable 400,000

d. Purchases 316,000
Accounts Payable 316,000

e. Accounts Payable 362,000


Cash 362,000

f. Expenses 89,500
Accrued Expenses 2,500
Cash 92,000

g. Allowance for Uncollectible Accounts 1,500


Bacolod Branch 1,500

h. Bacolod Branch 12,500


Shipments to Branch 12,500

I. Cash 10,000
Bacolod Branch 10,000
AA1- Chapter 8 (2008 edition) page
14

Requirement 2
Jazz Company
Statement of Recognized Income and Expenses - Bacolod Branch
For the Month Ended January 31, 2008

Sales P 62,000
Cost of goods Sold:
Shipments from Home Office (102,000 + 12,500 + 6,000) P120,500
Purchases 30,000
Cost of Goods Available for Sale P150,500
Less Merchandise Inventory, December 31 (9,800 + 600) 104,000 46,500
Gross Profit P 15,500
Expenses (12,500 + 4,750 + 350+ 3,500) 21,100
Net Loss P 5,600

Jazz Company
Statement of Financial Position - Bacolod Branch
January 31, 2008

Assets
Cash (15,000 - 9,000 + 26,000 - 14,500 - 12,500 + 16,000 - 10,000) P 11,000
Accounts Receivable (62,000 + 26,000 - 26,000 – 17,500) 44,500
Merchandise Inventory (98,000 + 6,000) 104,000
Total Assets P159,500
Liabilities
Accounts Payable (30,000 - 14,500) P 15,500
Accrued Expenses 3,500
Home Office (143,000-9,000-1,500+12,500-10,000+6,000+4,750+350 -5,600) 140,500
Total Liabilities P159,500

Jazz Company
Statement of Recognized Income and Expenses - Home Office
For the Month Ended January 31, 2008

Sales P346,000
Cost of Goods Sold:
Merchandise Inventory, January 1 P460,000
Purchases 316,000
Cost of Goods Available for Sale P776,000
Less Shipments to Branch (102,000 + 12,500 + 6,000) 120,500
Cost of Goods Available for Own Sale P655,500
Less Merchandise Inventory, December 31 445,000 210,500
Gross Profit P135,500
Expenses (89,500 - 4,750 + 1,000 + 7,500) 93,250
Net Income from Own Operations P 42,250
Less Branch Net Loss 5,600
Net profit P 36,650
Income Tax 12,828
Net Profit P23,822
AA1- Chapter 8 (2008 edition) page
15

Jazz Company
Statement of Financial Position - Home Office
January 31, 2008

Assets

Cash (150,000 - 15,000 + 400,000 – 362,000 - 92,000 + 10,000) P 91,000


Accounts Receivable (420,000 - 26,000 + 346,000 - 400,000) P340,000
Less Allowance for Uncollectible Accounts (12,000 - 1,500) 10,500 329,500
Merchandise Inventory 445,000
Branch (143,000 - 9,000 - 1,500 + 12,500 - 10,000
+ 6,000 + 4,750+ 350 - 5,600) 140,500
Store Furniture and Fixtures (150,000 + 9,000) P159,000
Less Accumulated Depreciation (46,000 + 1,350) 47,350 111,650
Total Assets P1,117,650

Liabilities and Shareholders’ Equity

Accounts Payable (337,500 + 316,000 - 362,000) P 291,500


Accrued Expenses 7,500
Income Tax Payable 12,828
Ordinary Share Capital (282,000 + 36,650) 500,000
Retained Earnings (282,000 + 23,822 305,822
Total Liabilities and Shareholders’ Equity P1,117,650

Requirement 3
Jazz Company
Combined Statement of Recognized Income and Expenses for Home Office and Branch
For the Month Ended January 31, 2008

Sales P408,000
Cost of Goods Sold:
Merchandise Inventory, January 1 P460,000
Purchases 346,000
Cost of Goods Available for Sale P806,000
Less Merchandise Inventory, December 31 549,000 257,000
Gross Profit P151,000
Expenses 114,350
Net Profit P 36,650
Income Tax 12,828
Net profit P 23,822
AA1- Chapter 8 (2008 edition) page
16

Jazz Company
Combined Statement of Financial Position for Home Office and Branch
January 31, 2008

Assets

Cash P 102,000
Accounts Receivable P384,500
Less Allowance for Uncollectible Accounts 10,500 374,000
Merchandise Inventory 549,000
Store Furniture and Fixtures P159,000
Less Accumulated Depreciation 47,350 111,650
Total Assets P1,136,650

Liabilities and Shareholders’ Equity

Accounts Payable P 307,000


Accrued Expenses 11,000
Income Tax Payable 12,828
Ordinary Share Capital 500,000
Retained Earnings 305,822
Total Liabilities and Shareholders’ Equity P1,136,650

Requirement 4
Branch Books

a. Shipments from Home Office 6,000


Home Office 6,000

b. Expenses 4,750
Home Office 4,750

c. Expenses 350
Home Office 350
P22,500 + P9,000 = P31,500/7.5 yrs x 1/12

d. Expenses 3,500
Accrued Expenses 3,500

e. Sales 62,000
Merchandise Inventory, end 104,000
Income Summary 5,600
Shipments from Home Office 120,500
Purchases 30,000
Expenses 21,100

f. Home Office 5,600


Income Summary 5,600
AA1- Chapter 8 (2008 edition) page
17

Home Office Books

a. Bacolod Branch 6,000


Shipments to Branch 6,000

b. Bacolod Branch 4,750


Expenses 4,750

c. Expenses 1,000
Bacolod Branch 350
Accumulated Depreciation 1,350
( 150,000 - 30,000 = 120,000 x 10% x 1/12 = 1,000 )
d. Expenses 7,500
Accrued Expenses 7,500

e. Branch Income 5,600


Bacolod Branch 5,600

f. Sales 346,000
Shipments to Branch 120,500
Merchandise Inventory, end 445,000
Income Summary 36,650
Merchandise Inventory, beg. 460,000
Purchases 316,000
Expenses 93,250
Branch Income 5,600

Income Tax 12,828


Income Tax Payable 12,828

Income Summary 12,828


Income Tax 12,828

g. Income Summary 23,822


Retained Earnings 23,822
AA1- Chapter 8 (2008 edition) page
18

Problem 8-5

Requirement 1
Feroza Company
Working Paper for Combined Financial Statement
For the Year Ended December 31,2008

Adjustments Combined Income Combined Balance


Eliminations Statement Sheet
Debits HO BR Dr. Cr. Dr. Cr. Dr. Cr.
Cash 63,000 21,900 84,900
NR 10,500 10,500
AR 120,600 55,950 176,550
Inventories 143,700 36,300 180,000
F&E 72,150 72,150
Branch 124,050 a.124,050
Cur.
CGS 300,750 128,700 429,450
OE 104,250 32,850 137,100
939,000 275,700
Credits
AP 61,500 61,500
CS 300,000 300,000
HO Cur. 124,050 a.124,050
RE 37,500 37,500
Sales 540,000 151,650 691,650
939,000 275,700 124,050 124,050 566,550 691,650
Net Profit 125,100 125,100
691,650 691,650 524,100 524,100

Requirement 2

a. Sales 151,650
Income Summary 9,900
Cost of Goods Sold 128,700
Operating Expenses 32,850

b. Home Office 9,900


Income Summary 9,900

Requirement 3

a. Branch Income 9,900


Branch 9,900

b. Income Summary 9,900


Branch Income 9,900
AA1- Chapter 8 (2008 edition) page
19

Problem 8-6

Requirement 1
Isuzu Company
Reconciliation of Home Office and Branch Accounts
January 31, 2008

Home Office Branch Books


Books
Branch Acct. Home Office Acct.
Unadjusted balances P77,150 P56,450
Add (deduct);
Advertising expense charged to branch 600
Merchandise shipment in transit 4,400
Merchandise shipment for P16,560 recorded
as P16,650 ( 90)
Collection of home office account 750
Understatement of 1994 depreciation ( 540)
Remittance to home office in transit (16,000) _______
Adjusted balances P61,360 P61,360

Requirement 2

Home Office Books

a. Cash 16,000
Retained Earnings 540
Accounts Receivable 750
Iloilo Branch 15,790

Branch Books

a. Advertising Expense 600


Shipments from Home Office 4,310
Home Office 4,910

Problem 8-7

Requirement 1

a. Shipments from Home Office 57,600


Operating Expenses 8,100
Home Office Current 65,700
AA1- Chapter 8 (2008 edition) page
20

b. Sales 778,200
Merchandise Inventory, end 122,180
Income Summary 116,990
Merchandise Inventory, beg. 47,800
Shipments from Home Office 680,800
Operating Expenses 54,790

c. Income Summary 116,990


Home Office Current 116,990

Requirement 2

a. Freight-Out 470
Branch Current 470

b. Cash 19,200
Branch Current 19,200

c. Branch Current 116,990


Branch Income 116,990

Requirement 3
Ford Company
Reconciliation of Current Account
December 31,2008

Branch Home Office


Acct. Acct.
Balances before adjustment P 206,344 P120,974

Shipment in transit 57,600


Advertising charged to branch 4,200
Rent charged to branch 3,900
Error in charging freight (470)
Remittance in transit (19,200)
P 186,674 P 186,674
AA1- Chapter 8 (2008 edition) page
21

Problem 8-8

Mitsubishi Trading Company


Reconciliation of Home Office and Branch Accounts
December 31, 2008

Branch Acct. Home Office Acct.


Unadjusted balances P225,770 P220,485
Add (deduct):
Error in recording cost of equipment 3,150
Insurance premium recorded twice by branch ( 675)
Freight for P1,125 recorded as P1,215 ( 90)
Discount from home office not recorded ( 800)
Share of branch in advertising not recorded 700
Error in recording remittance ( 3,000) ________
Adjusted balances P222,770 P222,770

Requirement 3

a. Office Equipment 3,150


Advertising Expense 700
Insurance Expense 675
Freight 90
Discount from Home Office 800
Home Office 2,285

MULTIPLE CHOICE
1. B

2. A

3. A Sales P400,000
Cost of sales ( 400,0000 - 70,000) 330,000
Gross profit 70,000
Expenses [30,000 + 10,000 + (10,000 - 6,000) + 5,000] 49,000
Net profit P 21,000

4. A Sales P46,500
x 70%
Cost of sales w/o freight P32,550
Add freight 1,100
Cost of sales w/ freight P33,650
AA1- Chapter 8 (2008 edition) page
22

5. Sales P46,500
Less Sales Discount (39,690 / 98%) - 39,690 810 P45,690
Cost of sales 33,650
Gross Profit P12,040
Expenses:
Selling P 2,820
Administrative (46,500 x 5%) 2,325
Samples Expenses 1,900 7,045
Net Profit P 4,995

6. 17,500 + 8,000 +9,250 + (50,000 x 60% x 1/6)

7. C Sales P176,000
Cost of sales 105,000
Gross Profit P 71,000
Expenses 39,750
Net Income P 31,250

8. D P 87,00 / 125% = P70,000


9. B Sales P 87,500
Cost of sales 70,000
Gross Profit P 17,500
Expenses (350 + 250) 6,000
Net Income P 11,500

10 A

11 C

12 B

13 A Sales P74,000
Cost of sales
Shipments P67,680
Less Inventory, end 9,180 58,500
Gross Profit P15,500
Expenses 6,820
Net Profit P 8,680

14 A P17,500 + 8,680 P 26,180


15 A

16 D 203,500 – (186,120 – 25,245 – 18,755) = 23,870

17 D Home Office Current P 48,125


Branch Income 23,870
Correct branch account - current P 71,995
AA1- Chapter 8 (2008 edition) page
23

18 B Sales P112,500
Cost of Sales:
Shipments from home office P120,000
Less Inventory, Dec. 31 30,000 90,000
Gross profit P 22,500
Expenses 8,100
Net Profit P 14,400

19 C P90,000 + P14,400 P104,400

20 A P1,500 + 43,800 + 37,170 P 82,470

21 D Net Sales (198,720 - 3,600) P 195,120


Cost of sales
Beg. Inventory P 37,170
Shipments 136,000
Goods available for sale P 173,170
End. Inventory 41,370 131,800
Gross Profit P 63,320
Expenses (57,930 + 1,920) 59,850
Net Profit P 3,470

22 C BR. Acct. HO Acct.


Balances before adjustment P150,000 P117,420
Adjustments:
1. Shipments in transit 37,500
2. HO AR collected by branch 10,500
3. Supplies returned (4,500)
4. Error in recording Br. net income (1,080)
5. Cash to Branch in transit 25,000 25,000
P179,920 P 179,920
23 D P 179,920

24 C BR Acct. HO. Acct.


Balances P43,500 P41,900
Error in recording allowance 60
Advances taken by Pres. (550)
Share in advertising expense 900
P42,950 P42,950
25 A Sales
Cost of sales P 100,000
Inventory, beg. P21,000
Merchandise from Home Office 61,000
Merchandise available for sale P82,000
Less Inventory, end 19,000 63,000
Gross profit P37,000
Operating Expenses 21,000
Net profit of Branch A P16,000
AA1- Chapter 8 (2008 edition) page
24

26 C Branch A Branch B
Imprest branch fund P 2,000 P 1,500
Accounts Receivable, Jan.1 55,000 43,500
Inventory, Jan.1 21,000 19,000
Home Office account P 78,000 P 64,000

27 B

28 C Imprest branch fund P 1,500


Accounts Receivable, Dec. 31 53,000
Inventory, Dec. 31 12,000
Balance of Branch account - current P 67,000

29 D Sales P 80,000
Cost of sales
Inventory, Jan.1 P 19,000
Merchandise from Home office 47,000
Merchandise available for sale P 66,000
Less Inventory, Dec.31 12,000 54,000
Gross profit P 26,000
Operating Expenses 14,300
Net profit of Branch B P 11,700

30 D HO Account. Branch
Acct.
Beg. Balances P 30,670 P 30,670
1. Branch remittances (55,000) (47,800)
2. Shipment to branch 138,000 160,000
3. Home office expense paid by branch (5,700)
4. Branch receivable collected by branch (8,900)
P 107,970 P 133,970

31 D Branch Acct. HO Acct.


Unadjusted balances P 133,970 P 107,970
1. Remittance in transit (7,200)
2. Shipment in transit 22,000
3. Home office expense paid by branch (5,700)
4. Branch receivable collected by branch (8,900)
5. Branch net profit 6,500 6,500
P 127,570 P 127,570

32 A Marketing Expense of another branch charged to Butuan P (10,000)


Butuan’s remittance credited to Davao ( 65,700)
Net adjustment in Home Office Banch account P (75,700)
AA1- Chapter 8 (2008 edition) page
25

33 C Fixed account not recorded by Butuan P (53,960)


Inventory transfer recorded twice by Butuan 75,000
Error in recording debit memo 4,650 ( 90)
Net adjustment in Branch Books P 20,950

34 D.
Branch Account Home Office
Account
Unadjusted balances P165,920 P111,170

35 B. Net adjustment in Branch Account (75,700) (20,950)


Adjusted balances P 90,220 P 90,220
CHAPTER 9
SUGGESTED ANSWERS

EXERCISES

Exercise 9 - 1
Books of Branch R
Home Office 15,000
Cash 15,000

Books of Branch S
Cash 15,000
Home Office 15,000

Books of the Home Office


Branch S 15,000
Branch R 15,000

Exercise 9 - 2
Books of Branch No. 1
Home Office 1,950
Shipments from Home Office 1,600
Freight-In 350

Books of Branch No. 5


Shipments from Home Office 1,600
Freight-In 400
Cash 350
Home Office 1,650

Books of the Home Office


Branch No. 5 1,650
Excess Freight 300
Branch No. 1 1,950
Exercise 9 – 3
Home Office Books
1. no entry

2. Branch 360,000
Shipments to Branch 240,000
Allowance for Markup in Branch Inventory 120,000
120,000/240,000 = 50%

3. no entry

4. Branch 134,000
Advertising Expense 40,000
Depreciation Expense 70,000
Utility Expense 24,000
AA1- Chapter 9 (2008 edition)
page 2

5. no entry

Cash 360,000
Branch 360,000

6. no entry

7. Branch 58,000
Branch Income 58,000

8. Allowance for Markup in Branch Inventory 100,000


Branch Income 100,000
P300,000 x 50/150 = P100,000

9. Branch Income 158,000


Income Summary 158,000

Branch Books
1. Purchases 160,000
Accounts Payable 160,000

2. Shipments from Home Office 360,000


Home Office 360,000

3. Accounts Receivable 652,000


Sales 652,000

4. Advertising Expense 40,000


Depreciation Expense 70,000
Utility Expense 24,000
Home Office 134,000

5. Cash 470,000
Accounts Receivable 470,000

Home Office 360,000


Cash 360,000

6. Merchandise Inventory 60,000


Sales 652,000
Purchases 160,000
Shipments from Home Office 360,000
Advertising Expense 40,000
Depreciation Expense 70,000
Utility Expense 24,000
Income Summary 58,000

7. Income Summary 58,000


Home Office 58,000
AA1- Chapter 9 (2008 edition)
page 3

Exercise 9 - 4
a. Merchandise inventory, beg. P150,000
Less Merchandise from home office at billed price
Markup on merchandise shipped to branch P 36,000
Markup on current shipment (P96,000 – P80,000) 16,000
Markup on beginning inventory P 20,000
x 120/20 120,000
Merchandise purchased from outsiders P 30,000

b. Allowance for Intercompany Inventory Profit 22,000


Branch Income 22,000
Bal. of allowance before adjustment P36,000
Adjusted balance of allowance acct
(P84,000 x 20/120) 14,000
Realized markup P22,000

Exercise 9 – 5
Home Office Books
1. Branch 450,000
Shipments to Branch 300,000
Allowance for Markup in Branch Inventory 150,000
150,000/300,000 = 50%

2. no entry

3. Allowance for Markup in Branch Inventory 116,000


Branch Income 116,000
Realized markup on beginning inventory
P600,000 x 55% = P330,000 x 25/125* P 66,000
Realized markup on current shipments
P450,000 x 1/3 = P150,000 x 50/150 50,000
Total P116,000
*(600,000 – 480,000) / 480,000 = 25%

Branch Books
1. Shipments from Home Office 450,000
Home Office 450,000

2. Accounts Receivable 870,000


Sales 870,000
P590,000 + P280,000 = P870,000

3. no entry

Exercise 9 – 6
1. Branch 820,000
Cash 80,000
Shipments to Branch 240,000
AA1- Chapter 9 (2008 edition)
page 4

Land 300,000
Allowance for Markup in Branch Inventory 120,000
Allowance on Transfer of Land 80,000
120,000/240,000 = 50%

2. Branch 560,000
Shipments to Branch 400,000
Allowance for Markup in Branch Inventory 160,000
160,000/400,000 = 40%

3. Branch 130,000
Branch Income 130,000

4. Allowance for Markup in Branch Inventory 200,000


Branch Income 200,000
Realized markup on 1st inventory transfer P120,000
Realized markup on 2nd inventory transfer
(P640,000 – P360,000) x 40/140 80,000
Total P200,000

5. Allowance on Transfer of Land 80,000


Branch Income 80,000

6. Branch Income 410,000


Income Summary 410,000

Exercise 9 - 7
Required balance of allowance (markup on branch ending inventory)
P9,600 x 20/120 P 1,600
Adjustment for realized markup 36,400
Balance of allowance before adjustment P38,000
Allowance on current shipment (P160,000 x 20%) 32,000
Allowance on branch beginning inventory P 6,000
Markup rate ÷ 20%
Branch beginning inventory, at cost P30,000

Exercise 9 - 8
a. Merchandise available for sale at billed price (P16,200 + P20,250) P36,450
Merchandise available for sale at cost (P36,450/135%) 27,000
Unrealized intercompany inventory profit balance before adjustment P 9,450

b. Unrealized Intercompany Inventory Profit 4,550


Branch Income 4,550
Balance before adjustment P 9,450
Adjusted balance (P18,900 x 35/135) 4,900
Realized markup P 4,550
AA1- Chapter 9 (2008 edition)
page 5

Home Office Books


c. Shipments to Branch 400
Unrealized Intercompany Inventory Profit 140
Branch 540

Branch Books
Home Office 540
Shipments from Home Office 540

Exercise 9 – 9
1. P20,000 ÷ 25/125 = P100,000

2. Allowance for Markup in Branch Inventory 74,000


Branch Income 74,000
P100,000 + P350,000 – P80,000 = P370,000 x 25/125 = P74,000

Exercise 9 - 10
Separate cost of goods sold of the home office:
Inventory, beginning P 252,000
Purchases 2,800,000
Shipments to branch ( 600,000)
Cost of goods available for sale P2,452,000
Less Inventory, end 240,000 P2,212,000
Separate cost of goods sold of the branch:
Inventory, beginning
From outside purchases P 12,000
From home office (P36,000 / 120%) 30,000
Total P 42,000
Purchases 96,000
Shipments from home office (P720,000 / 120%) 600,000
Cost of goods available for sale P 738,000
Less Inventory, end:
From outside purchases P10,000
From home office (P42,000 / 120%) 35,000 45,000 693,000
Combined cost of goods sold P2,905,000

Exercise 9– 11
1. Total Resold On Hand
Shipments from home office P450,000 P360,000 P90,000
Shipments to branch 375,000 300,000* 75,000**
Markup P 75,000 P 60,000 P15,000

* 75,000 / 375,000 = 20%


** 90,000 / 120% = P75,000

2. Cost of Goods Sold 420,000


Inventory 420,000
P60,000 + P450,000 – P90,000 = P420,000
AA1- Chapter 9 (2008 edition)
page 6

3. Billed Price Cost Markup


Inventory, beginning P 60,000 P 50,000 P 10,000
Shipments 450,000 375,000 75,000
Total P510,000 P425,000 P 85,000
Inventory, end 90,000 75,000 15,000
Cost of goods sold P420,000 P350,000 P 70,000

Exercise 9 - 12
a. Merchandise Inventory, January 1 P26,400
Add Shipments from Home Office 20,000
Cost of Goods Available for Sale P46,400
Cost of Goods Sold
Sales, net of Sales Returns (P15,000 - P2,000) P13,000
Sales rate 125% 10,400
Merchandise destroyed by fire at billed price P36,000
÷ 120%
Merchandise destroyed by fire at cost P30,000
b. Home Office Books
Branch Loss from Fire 30,000
Allowance for Markup in Branch Inventory 6,000
Branch 36,000
Branch Books
Home Office 36,000
Merchandise Inventory 36,000

Exercise 9 – 13
1. Branch Income 50,000
Cost of Goods Sold P230,000 x 15/115 30,000
Branch 20,000

2. Home Office 520,000


Branch 520,000

3. Allowance for Markup in Branch Inventory 9,000


Inventory 9,000

PROBLEMS
Problem 9 – 1
Billed Price Cost Markup
Beginning inventory:
Acquired from vendors P100,000 P100,000 -
Acquired from home office 40,000 32,000 P 8,000
Purchases from vendors 240,000 240,000 -
Shipments from Home Office P180,000 + P30,000 210,000 168,000 42,000
Total inventory available for sale P590,000 P540,000 P50,000
Less Ending inventory:
Acquired from vendors 40,000 40,000 -
Acquired from home office P60,000 + P30,000 90,000 72,000 18,000
Cost of goods sold P460,000 P428,000 P32,000
AA1- Chapter 9 (2008 edition)
page 7

Problem 9 – 2
Home Office Books
a. Dagupan Branch 10,000
Cash 10,000

b. Dagupan Branch 25,000


Baguio Branch 40,000
Shipments to Branch 65,000

c. Furniture and Fixtures 17,500


Cash 17,500

d. Expenses 800
Dagupan Branch 800

e. Baguio Branch 29,400


Sales Discounts 600
Accounts Receivable 30,000

f. Baguio Branch 15,000


Dagupan Branch 15,000

g. Shipments to Branch 2,500


Dagupan Branch 2,500

h. Dagupan Branch 1,800


Baguio Branch 1,800

i. Dagupan Branch 20,100


Shipments to Branch 20,000
Cash 100

j. Baguio Branch 20,110


Excess Freight 35
Dagupan Branch 20,145

Dagupan Branch Books


a. Cash 10,000
Home Office 10,000

b. Shipments from Home Office 25,000


Home Office 25,000

c. no entry

d. Home Office 800


Cash 800

e. no entry
AA1- Chapter 9 (2008 edition)
page 8

f. Home Office 15,000


Cash 15,000

g. Home Office 2,500


Shipments from Home Office 2,500

h. Expenses 1,800
Home Office 1,800

i. Shipments from Home Office 20,000


Freight-In 100
Home Office 20,100

j. Home Office 20,145


Shipments from Home Office 20,000
Freight-In 100
Cash 45

Baguio Branch Books


a. no entry

b. Shipments from Home Office 40,000


Home Office 40,000

c. no entry

d. no entry

e. Cash 29,400
Home Office 29,400

f. Cash 15,000
Home Office 15,000

g. no entry

h. Home Office 1,800


Cash 1,800

i. no entry

j. Shipments from Home Office 20,000


Freight-In 110
Home Office 20,110

Problem 9 – 3
Requirement 1
Home Office Books
1. Baguio Branch 20,000
Cash 20,000
AA1- Chapter 9 (2008 edition)
page 9

2. Baguio Branch 259,000


Shipments to Branch 185,000
Allowance for Markup in Branch Inventory 74,000

3. Cash 245,000
Baguio Branch 245,000

4. Baguio Branch 7,000


Cash 7,000

5 – 7 - no entry

Baguio Branch Books


1. Cash 18,000
Home Office 18,000

2. Shipments from Home Office 257,600


Home Office 257,600

3. Home Office 247,400


Cash 247,400

Accounts Receivable 2,400


Home Office 2,400

4. Expenses 7,000
Home Office 7,000

5. Cash 247,400
Accounts Receivable 40,600
Sales 288,000

6. Expenses 21,000
Cash 21,000

7. Merchandise Inventory, end P30,100 + P1,400 31,500


Sales 288,000
Income Summary 15,000
Merchandise Inventory, beginning P257,600 + P1,400 17,500
Shipments from Home Office 259,000
Expenses 28,000

Income Summary 15,000


Home Office 15,000

Requirement 2
Baguio Branch 15,000
Branch Income 15,000

Allowance for Markup in Branch Inventory 70,000


Branch Income 70,000
(P17,500 + P259,000 – P31,500) x 40/140
AA1- Chapter 9 (2008 edition)
page 10

Branch Income 85,000


Income Summary 85,000

Requirement 3

Shipments from Home Office 1,400


Home Office 1,400

Cash 2,000
Home Office 2,000

Problem 9 - 4
Requirement 1
Triple D Bookstore
Statement of Recognized Income and Expenses - Quezon City Branch
For the Year Ended December 31, 2008
Sales P192,690
Cost of Goods Sold:
Merchandise Inventory, beginning P 31,500
Shipments from Home Office 128,000
Cost of Goods Available for Sale P159,500
Less Merchandise Inventory, end 22,750 136,750
Gross Profit P 55,940
Operating Expenses:
Advertising and Promotion P 6,400
Depreciation 2,400
Uncollectible Accounts Expense 1,250
Others 36,600 46,680
Net income P 9,260

Requirement 2
Branch 9,260
Branch Income 9,260

Allowance for Markup in Branch Inventory 27,350


Branch Income 27,350
P136,750 x 25/125

Branch Income 36,610


Income Summary 36,610

Problem 9 - 5
Branch Books
a. Sales 78,000
Merchandise Inventory, end 12,000
Income Summary 10,000
Merchandise Inventory, beginning 10,000
Shipments from Home Office 80,000
Selling Expenses 4,000
Administrative Expenses 6,000
AA1- Chapter 9 (2008 edition)
page 11

b. Home Office 10,000


Income Summary 10,000

Home Office Books


a. Sales 310,000
Shipments to Branch 64,000
Merchandise Inventory, end 30,000
Merchandise Inventory, beginning 25,000
Purchases 300,000
Selling Expenses 20,000
Administrative Expenses 30,000
Income Summary 29,000

b. Branch Income 10,000


Branch 10,000

c. Allowance for Overvaluation in Branch Inventory 15,600


Branch Income 15,600
P18,000 - (P12,000 x 25/125) = P15,600

d. Branch Income 5,600


Income Summary 5,600

e. Income Tax 12,110


Income Tax Payable 12,110

f. Income Summary 12,110


Income Tax 12,110

g. Income Summary 22,490


Retained Earnings 22,490

Problem 9 - 6
Requirement 1
Triple F Products Inc. - Branch
Trial Balance
December 31, 2008

Debit Credit
Cash 12,800
Accounts Receivable 48,160
Merchandise Inventory 27,280
Accounts Payable 2,040
Home Office 68,900
Sales 256,000
Cost of Sales 191,620
Operating Expenses 47,080 _______
326,940 326,940
AA1- Chapter 9 (2008 edition)
page 12

Requirement 2
Home Office Books
a. Sales 640,600
Income Summary 41,360
Cost of Sales 452,840
Operating Expenses 146,400

b. Branch 17,300
Branch Income 17,300

c. Allowance for Overvaluation of Branch Inventory 17,420


Branch Income 17,420
P191,620 x 10/110
P21,100 ÷ (P202,400 + P29,700 – P21,100) = 10%

d. Branch Income 34,720


Income Summary 34,720

e. Income Tax 26,628


Income Tax Payable 26,628

f. Income Summary 26,628


Income Tax 26,628

g. Income Summary 76,080


Retained Earnings 76,080

Branch Books
a. Merchandise Inventory P202,400 – P189,200 13,200
Home Office 13,200

b. Sales 256,000
Income Summary 17,300
Cost of Sales 191,620
Operating Expenses 47,080

c. Income Summary 17,300


Home Office 17,300
Requirement 3

Combined net income (P41,360 + P34,720 – P26,628) P49,452

Combined Merchandise Inventory:


Home Office P156,640
Branch [ P27,280 + P13,200) ÷110% 36,800 P193,440
AA1- Chapter 9 (2008 edition)
page 13

Problem 9 - 7

Triple G Company
Combined Statement of Recognized Income and Expenses
for Home Office and Branch
For the Year Ended December 31, 2008

Sales P325,000
Cost of goods sold:
Merchandise inventory, beginning P107,500
Purchases 215,000
Cost of goods available for sale P322,500
Less Merchandise inventory, end 81,300 241,200
Gross profit P 83,800
Operating expenses 50,000
Net income before Income Tax P 33,800
Income Tax 11,830
Net Income P 21,970

Inventory: Beginning Ending


Home Office P 80,000 P55,000
Branch P7,500 + (P24,000/120%) 27,500 P5,500 + (P26,000/125%) 26,300
Total P107,500 P81,300
** P37,500 – P30,000 = P7,500/ P30,000 = 25%
Requirement 2
Davao Branch Books
a. Sales 75,000
Merchandise Inventory, end 31,500
Income Summary 12,500
Shipments from Home Office 37,500
Purchases 15,000
Expenses 10,000
Merchandise Inventory, beginning 31,500

b. Income Summary 12,500


Home Office 12,500
Requirement 3
Home Office Books
a. Davao Branch 12,500
Branch Income 12,500

b. Allowance for Markup in Branch Inventory 6,300


Branch Income 6,300
Markup on branch beginning inventory
(P24,000 x 20/120) P 4,000
Markup on shipments 7,500
Allowance balance before adjustments P11,500
Markup on branch ending inventory
(P26,000 x 25/125) 5,200
Realized markup P 6,300
AA1- Chapter 9 (2008 edition)
page 14

c. Sales 250,000
Shipments to Branch 30,000
Merchandise Inventory, end 55,000
Income Summary 15,000
Purchases 200,000
Expenses 40,000
Merchandise Inventory, beginning 80,000

d. Branch Income 18,800


Income Summary 18,800

e. Income Tax 11,830


Income Tax Payable 11,830

f. Income Summary 11,830


Income Tax 11,830

g. Income Summary 21,970


Retained Earnings 21,970

Problem 9 – 8

Requirement 2
a. Plant Assets 4,000
Branch 4,000

b. Home Office 2,000


Accounts Receivable 2,000

c. Cash 5,000
Branch 5,000

d. Expenses 1,000
Home Office 1,000

e. Shipments from Home Office 3,000


Home Office 3,000

f. Retained Earnings 2,500


Inventory 2,500
P15,000 x 20/120

g. Home Office 11,000


Branch 11,000

h. Sales 48,000
Shipments from Home Office 48,000
AA1- Chapter 9 (2008 edition)
page 15
AA1- Chapter 9 (2008 edition)
page 16
AA1- Chapter 9 (2008 edition)
page 17

MULTIPLE CHOICE

1. B 3. C 5. D 7. D
2. B 4. D 6. B 8. D

9 A

10 C
11 C
12 D P13,200 + P350 = P13,550
13 C P11,000 + P350 = P11,350

14 A P12,000 x 20/120 = P2,000

15 D Inventory, beginning (P165,000 / 125%) P132,000


Shipments (P110,000 / 125%) 88,000
Merchandise available for sale from home office at cost P220,000
Cost of merchandise sold from home at cost
Sales, net of returns and allowances P165,250
Less Sales from merchandise purchased from
outsiders (P7,500 x 120%) 9,000
Sales from merchandise from home office P156,250
÷ 125%
Cost of sales at billed price P125,000
Billed price rate ÷ 125% 100,000
Inventory destroyed by fire P120,000

16 D Balance of allowance before adjustment P370,000


Required balance of allowance (P1,170,000 x 20/120) 195,000
Realized markup P175,000

17 C Sales P141,000
Cost of goods sold (P120,000 x 3/4 x 125%) 112.500
Gross profit P 28,500
Operating expenses 27,000
Net income reported by the branch P 1,500

18 A P50,400/120% P42,000

19 B P90,000 + P36,000 – P2,520 – P50,400/120% = P60,900

20 B Net income (loss) reported by branch (P 7,800)


Realized markup
(P90,000 + P36,000 – P2,520 - P50,400 = P73,080) x 20/120 12,180
True net income of the branch P 4,380

21 B Net income reported by branch P 4,800


Realized markup [(P3,960 + P17,600 – P4,840) x 10/110] 1,520
Actual branch income P 6,320
AA1- Chapter 9 (2008 edition)
page 18

22 B Branch ending inventory, at cost (P4,840 / 110%) P 4,400


Home office ending inventory 11,200
Ending inventory to be reported in the combined balance sheet P15,600

23 D Net income reported by the branch P 5,000


Realized markup[P280,000 - (P50,000 – P6,600)] x 40/140 67,600
True net income of the branch P 72,600

24 B Branch inventory from home office (P43,400 /140%) P31,000


Branch inventory from outside purchases 6,600
Total cost of branch inventory, end P37,600

25 D P60,000 - P7,500 P52,500

26 A P60,000 - (P7,500 x 120/20) P15,000

27 B Sales P 292,500
Cost of goods sold (P180,000 + P45,000 - P60,000) (165,000)
Operating expenses ( 72,000)
Realized markup [(P180,000 x 20/120) - P7,500 22,500
True net income of the branch P 78,000

28 A Unadjusted balance of allowance account P99,900


Markup on 2008shipments from home office
(P390,000 – P300,000) 90,000
Markup on beginning inventory P 9,900
x 130/30 P 42,900
Total merchandise inventory beg 54,600
Merchandise from outside purchases P 11,700

29 D Sales P540,000
Cost of goods sold
(P54,600 + P390,000 + P144,600 - P48,750) (540,450)
Operating expenses ( 51,000)
Realized markup [P99,900 - (P39,000 x 30/130)] 90,900
True net income of the branch P 39,450

30 C P39,000 x 30/130 = P9,000

31 D Sales (net of discount of P1,480) P115,520


Cost of goods sold (P104,000 - P12,500) ( 91,500)
Operating expenses ( 20,000)
Net income reported by branch P 4,020

32 C Net income reported by branch P 4,020


Realized markup (P91,500 x 25/125) 18,300
True net income of the branch P 22,320
AA1- Chapter 9 (2008 edition)
page 19

33 C Sales P 37,400
Cost of goods sold (P5,000 + P2,000 + P26,400 – P4,500) ( 28,900)
Operating expenses ( 3,000)
Realized markup [P2,800 – (P3,960 x 10/110)] 2,440
True profit of Cebu branch P 7,940

34 D Sales P110,000
Cost of goods sold
(P16,000 + P80,000 – P24,000 – P20,000) ( 52,000)
Operating expenses ( 10,000)
Net income of the home office P 48,000
Net income of the branch 7,940
Combined net income of the home office and branch P 55,940

35 C Sales P155,000
Cost of sales:
Inventory, beginning P 23,000
Purchases 190,000
Goods available for sale P213,000
Shipments to branch (P110,000/110%) 100,000
Goods available for own sale P113,000
Less Inventory, end 30,000 83,000
Gross profit P 72,000
Expenses 52,000
Net income P 20,000

36 A Sales P140,000
Cost of sales:
Inventory, beginning (P11,550 – P1,000) P 10,550
Shipments from HO, including freight-in 105,750
Goods available for sale P116,300
Less Inventory, end
[(P10,400 + P5,000)/110%] + P520 + P250 14,770 101,530
Gross profit P 38,470
Expenses 28,400
True branch net income P 10,470

37 D (P10,400 + P5,000) x 10/110 P1,400

38 B Unadjusted balance of allowance account P 57,500


Markup on 2008 shipments (P200,000 x 25%) 50,000
Markup on beginning inventory P 7,500
x 125/25
Branch beginning inventory at billed price P 37,500
AA1- Chapter 9 (2008 edition)
page 20

39 C Sales P400,000
Cost of goods sold (P37,500 + P250,000 - P40,000) (247,500)
Operating expenses (100,000)
Net income reported by branch P 52,500

40 C Net income reported by branch P 52,500


Realized markup (P247,500 x 25/125) 49,500
True net income of the branch P102,000

41 B Beginning inventory P 8,000


Purchases 30,000
Shipments from home office 93,750
Ending inventory ( 10,350)
Cost of goods sold reported by branch P 121,400
Realized markup [P19,750 - (P6,000 x 25/125)* ( 18,550)
Cost of goods sold at cost P 102,850

*P93,750 – P75,000 = P18,750/P75,000 = 25%

42 C The amount of the realized markup of P18,550

43 C Adjusted net income P156,000


Reported net income 60,000
Realized markup P 96,000
Cost of sales at cost
(P70,000 + P350,000 – P84,000) – P96,000 ÷ P240,000
140%

44 C P84,000 x 40/140 P24,000

45 A 72,500 ÷ (75,000 + 444,000 -84,000 – 72,500) 20%

46 D P444,000 / 120% P370,000

47 D Sales P 600,000
Cost of goods sold (P75,000 + P444,000 - P84,000) (435,000)
Operating expenses (200,000)
Realized markup [P72,500 - (P84,000 x 20/120)] 58,500
Adjusted profit of the branch P 23,500

48 C P84,000 x 20/120 = P14,000

49 A P84,000 – P14,000 = P70,000

50 B Home office inventory (P160,500 - P10,500) P 150,000


Branch inventory (P108,000/120%) 90,000
Inventories reported in the combined balance sheet P 240,000
AA1-Chapter 9 (2008 edition) page1

Problem 9 – 8
Triple J Wholesale Company
Work Sheet for Combined Financial Statements
For the Year Ended December 31, 2008

Adjustments and
Trial Balance Eliminations Income Statement Balance Sheet
Debits Home Office Branch Debit Credit Debit Credit Debit Credit
Cash 36,000 8,000 (c ) 5,000 49,000
Accounts Receivable 35,000 12,000 (b) 2,000 45,000
Inventory 70,000 15,000 (f) 2,500 82,500 74,500 74,500
Plant Assets, net 90,000 (a) 4,000 94,000
Branch 20,000 (a) 4,000
(c ) 5,000
(g) 11,000
Purchases 290,000 24,000 314,000
Shipments from Home Office 45,000 (e) 3,000 (h) 48,000
Expenses 44,000 16,000 (d) 1,000 61,000
Income Tax (i) 36,400 36,400
585,000 120,000
Credits
Accounts Payable 36,000 13,500 49,500
Accrued Expenses 14,000 2,500 16,500
Income Tax Payable (i) 36,400 36,400
Home Office 9,000 (b) 2,000 (d) 1,000
(g) 11,000 (e) 3,000
Ordinary Share Capital 50,000 50,000
Retained Earnings 45,000 (f) 2,500 42,500

Sales 440,000 95,000 (h) 48,000 487,000


585,000 120,000 112,900 112,900 493,900 561,500
Net income 67,600 67,600
561,500 561,500 262,500 262,500

Inventory, beg: Inventory, end:


Home Office P55,000 Home Office P70,000
Branch P2,000 + P21,000/120% 19,500 Branch 12,500
P74,500 P82,50
AA1-Chapter 9 (2008 edition) page2

Problem 9 – 9
Triple M Company
Work Sheet for Combined Financial Statements
For the Year Ended December 31, 2008
Adjustments and Combined
Trial Balance Eliminations Cost of Goods Sold Income Statement Balance Sheet
Debits HO Branch Debit Credit Debit Credit Debit Credit Debit Cred8t
Cash 17,000 200 a. 1,700 20,700
b. 1,800
Inventory 23,000 11,550 e. 1,000 33,550 44,770 44,770
Sundry Assets 200,000 48,450 248,450
Investment in Branch 60,000 a. 1,700
g. 58,300
Purchases 190,000 190,000
Shipment from Home Office 105,000 c. 5,000 f. 110,000
Freight-in from Home Office 5,500 d. 250 5,750
Sundry Expenses 42,000 24,300
Income Tax h. 15,460 15,460
532,000 195,000 66,300
Credits
Sundry Liabilities 35,000 3,500 d. 250 38,750
Income Tax Payable h. 15,460 15,460
Ordinary Share Capital 200,000 200,000
Retained Earnings 31,000 31,000
Home Office Equity 51,500 g. 58,300 b. 1,800
c. 5,000
Sales 155,000 140,000 295,000
Shipments to Branch 110,000 f. 110,000
Allowance for Markup in BI 1,000 e. 1,000
532,000 195,000 193,510 193,510 229,300 44,770
Cost of Goods Sold 184,530 184,530
229,300 229,300 266,290 295,000
Net income 28,710 28,710
295,000 295,000 313,920 313,920
Merchandise inventory, end:
Home office P30,000
Branch [((P15,400 / 110%) + (P15,400 x 5%)] = P14,000 + P770 14,770
Total P44,770
AA1-Chapter 9 (2008 edition) page3
AA1 - Chapter 9 (2008 edition) page 19

Problem 9 – 10
Triple N Commercial
Working Paper for Combined Financial Statements for Home Office and Branch
For the Year Ended December 31, 2008

Adjusted Branch Home Office Combined


Trial Balance Adjustments Trial Balance Eliminations Income Statement Income Statement Balance Sheet
Debits HO Branch HO Branch HO Branch Debit Credit Debit Credit Debit Credit Debit Credit
Cash 50,100 1,260 a. 5,320 b. 2,100 55,420 3,360 58,780
Accounts Receivable (net) 350,000 135,660 c. ( 2,500) 350,000 133,160 483,160
Inventory – Home office 64,400 64,400 64,400 70,000
Branch 32,340 32,340 a. 2,940 29,400 32,200 102,200
Fixed Assets (net) 210,000 210,000 210,000
Branch Current 163,120 a. (5,320) 157,800 c. 157,800
Purchases 532,000 532,000 532,000
Shipments from Home Office 294,000 d. 14,000 308,000 b. 28,000 280,000
Sundry Expenses 119,980 83,440 119,980 83,440 83,440 119,980
Income Tax f. 23,667 e. 10,976 23,667 10,976 10,976 23,667
1,489,600 546,700 23,667 24,576 1,513,267 571,276
Credits
Accounts Payable 30,500 10,500 30,500 10,500 41,000
Mortgage Payable 67,500 67,500 67,500
Income Tax Payable f. 23,667 e. 10,976 23,667 10,976 34,643
Home Office Current 144,200 b. 2,100
c. ( 2,500)
d. 14,000 157,800 c. 157,800
Sales 434,000 392,000 434,000 392,000 392,000 434,000
Shipments to Branch 308,000 308,000 b. 28,000 280,000
Allowance for Overvaluation 2,940 2,940 a. 2,940
Ordinary Share Capital 600,000 600,000 600,000
Retained Earnings 46,660 46,660 46,660
1,489,600 546,700 23,667 24,576 1,513,267 571,276 188,740 188,740 403,816 424,200 740,047 784,000 854,140 789,803
Branch net income 20,384 20,384
Home Office net income 43,953 43,953
424,200 424,200 784,000 784,000 854,140 854,140

Inventory, end:
Home Office P 70,000
Branch [(P21,420 + P14,000) / 110%] 32,200
Total P102,200
AA1 - Chapter 9 (2008 edition) page 19

You might also like